You are on page 1of 112

1

2
3
4
5
6
7
8
9
10
11
12
13
14
15
16
SOLVED EXAMPLES
Ex.1. Let us consider the following subsets of R 2
(i) The set of complex Z ∋ Z < 1 is open and bounded.

(ii) The set of complex Z ∋ Z ≤ 1 is closed, perfect and bounded.


(iii) A non empty finite set is closed and bounded.
(iv) The set of all integers are closed.
1
(v) The set consisting of the numbers ( n = 1, 2 , ...) . This set has a limit point and bounded.
n
(vi) The segment (a, b) is bounded.
1
Ex.2. Find the infimum and supremum of a function f : (0,1) → R defined by f ( x ) =
x
1
Sol: Consider a function f : (0,1) → R ∋ f ( x ) = . The range of f is (1, ∞) .
x
∴ f is not bounded above on (0,1) , but it is bounded below. f does not have a maximum or a
minimum on (0,1) . However, inf f = 0 . If we replace (0,1) by (0,1] then the range of f is[1, ∞) . So f
is bounded below and has a minimum which is also the infimum equal to 0. But f is not bounded above.


1
Ex.3.Consider the power series ∑ 2n n x n
n =1

(i) For which values of x ∈ R does the series converge ( absolutely? conditionally?)
n x x
Sol: (i) The ratio of the absolute value of successive terms is → as n → ∞ .
n +1 2 2
Hence the series converges absolutely for x < 2 . If x = +2 the series is the harmonic series,

1
∑n which diverges by comparison or the integral test. If x = −2 the series converges, but only
conditionally, by the Leibnitz test.

⎧⎪3 x − 2 if x < 1
Ex.4.Determine the function f ( x) = ⎨ 3 is differentiable at x = 1 . Find the derivative.
⎪⎩ x if x ≥ 1

Sol: f (1) = 13 = 1
The limit from the right is
f ( x) − (1) x3 − 1 ( x − 1)( x 2 + x + 1)
lim+ = lim+ = lim+
x →1 x −1 x →1 x − 1 x →1 x −1

= xlim( x 2 + x + 1) = 3
→1+

The limit from the left is

17
Ex.2 Analysis

f ( x) − f (1) 3x − 2 − 1
lim = xlim = xlim 3=3
x →1− x −1 →1− x −1 →1−

Since, the limt from the left equals the limit from the right, the limit exists and is equal to 3. So f ′(1) = 3 .

⎧ 1 1 1⎫
Ex.5. Find lim ⎨ + + .... + ⎬
x →∞ n + 1 n + 2 2n⎭

Sol: Divide the interval [1, 2] into n equal pieces. The right hand Riemann sum for the function

1 1⎧ n n 1⎫
f ( x) = is ⎨ + + .... ⎬ and
x n ⎩ n +1 n + 2 2⎭
2
1
∴ f(x) is continuous, hence integrable the sum converges to ∫ x dx = log 2
1


nx
Ex.6. Is the series ∑ 1 + n4 x2
n =1
converges uniformly on [a, ∞) for a > 0

nx 1 1
= . 1 n 1 1
Sol: 1 + n4 x 2 x 1 2 + n4 ≤ a . 4 = a . 3
x n n
∞ ∞
1 nx
As ∑ n3n =1
converges by the p - test, ∑ 1 + n4 x2
n =1
converges by the Comparison test.


sin x
Ex.7. Is ∫
π x
dx converges

sin x
Sol: We approximate from below by a step function φ . For n > 0 an odd integer sin(nπ 2 ) = 1
x

π 1 sin x 1
and x − nπ < ⇒ sin x > ⇒ ≥
2 6 2 x (n + 1)π

⎧ 1 nπ π nπ π
⎪ for n odd − ≤x≤ +
Define
φ ( x) = ⎨ (n +1)π 2 6 2 6
⎪0 otherwise


sin x
∞ ∞
2π 1
Then, ∫ dx ≥ ∫ φ ( x) dx ≥ ∑ =∞
π x π k =1 6 (2k + 1 + 1)π


sin x
∴ ∫
π x
dx is diverges

n
1
Ex. 8. Let f n : R → R be the function defined by f n ( x) = ∑ 2
k =1 2k + k cos( kx )

18
Analysis Ex.3

(i) Prove that f n converges uniformly to a continuous function f : R → R


(ii) Prove that f is differentiable
1 1
Sol: (i) Note that 0 < 2
≤ 2 for all x ∈ R and all k ≥ 1 .
2k + k cos(kx) k

1
∴ The series ∑ k2
k =1
converges it follows from the Weierstrass M - test that the sequence f n

converges uniformly on R to a fuction f.


∴ Every f n is continuous, so is f.

d 1 sin(kx)
(ii) =
dx x (2k + k cos( kx )) (2k + k cos(kx)) 2
2

1
and that the absolute value of the term on the right hand side is bounded by for all x ∈ R and
k2
all k ≥ 1 . It follows from the Weierstrass M- test that the sequence f n′ converges uniformly to a

function g : R → R . ∵ Every f n′ is continuous, so is g. It follows from this that f is differentiable


with derivative g ( x) at x.

Ex.9.Is the following functions Riemann integrable on [ 0, 1 ]?


⎧ x, x ∈ Q
f ( x) = ⎨
⎩0 x ∉ Q
Sol: No, on any interval [a, b] ⊂ [0,1] . inf{ f ( x) : a ≤ x ≤ b} = 0 and sup{ f ( x) : a ≤ x ≤ b} = b > 0 .
∴ All lower sums L( f , P) = 0 and all upper sums are the same as for the integrable function
1
g ( x) = x , hence inf{U ( f , P) : P} = . Therefore f is not integrable.
2

Ex.10. Define the sequence {an }n = 0 of numbers as follows. Let a0 = 2 for n > 0 , define an by

1 an −1
an = +
2 2
(i) Show that 1 < an < an −1 for all n.

(ii) Show that the sequence {an }n = 0 is convergent and find its limit.

Sol:
(i) By induction assume an −1 > 1 . Then an is the sum of 1/2 and a number greater than 1/2 and
1 an −1
thus is greater than 1. The inequality an < an −1 can be rewritten as < or an −1 > 1 which
2 2
again holds by the induction hypothesis.
(ii) By completeness of the reals, a decreasing sequence of numbers bounded below has a limt L.

19
Ex.4 Analysis

1 an −1 1 L
Taking limits in both sides of the recursion an = + as n → ∞ gives L = +
2 2 2 2
which is solved by L = 1 .

e − kx
Ex.11. Consider the series f ( x) = ∑
k =1 k

(i) For what values of x ∈ R does it converge?


k −x
Sol: (i) The ratio of the absolute value of the successive terms is e which tends to e− x
k +1
as k → ∞ . Hence the series converges absolutely for x > 0 and diverges for x < 0 .
For x = 0 the series is the harmonic series which diverges.

⎛1⎞
Ex.12. Consider the sequence f n ( x ) = x2 + ⎜ ⎟
⎝n⎠

(i) Prove the sequence f n ( x) converges uniformly to f ( x) = x on R.

(ii) Prove f n′ ( x) converges pointwise on R and that for x ≠ 0 , nlim f n′( x) = f ′( x)


→∞

Sol:
(i) Both f n and f are even functions. Let g ( x) = f n ( x) − f ( x) . For x > 0 we have

x
g ′( x) = −1 < 0
⎛1⎞ 1
2
x +⎜ ⎟ hence g is decreasing on (0, ∞) . Thus f n ( x) − f ( x) ≤ n,
⎝n⎠

So f n ( x) converges uniformly to f.

x x x
lim = = = f ′( x )
(ii) For x ≠ 0 n →∞
x + 1
2
( n) x + lim 1
2
( n) x

So f n ( x) converges pointwise to f ( x) for x ≠ 0 .


1
Ex.13. Show that the function f ( x) = is uniformly continuous on R.
1 + x2

2
Sol: Let ε > 0 be given. Without loss of generality, we may assume that ε < 2 . Then for M = −1
ε

1 1 ε
We have 2
≤ 2
< Hence for all x , y > M
1+ x 1+ M 2

1 1 1 1 2
We have 2
− 2
≤ 2
+ 2
< <ε .
1+ x 1+ y 1+ x 1+ y 1+ M 2
Now , since f is continuous on R, it is continuous and thus uniformly continuous on [- M - 1, M + 1].

20
Analysis Ex.5

For above ε > 0 , ∃ δ1 > 0 ∋ for all x, y ∈ [ − M − 1, M + 1] , we have f ( x) − f ( y ) < ε . Then for

δ = min{1, δ1} > 0 , when x, y ∈ R ∋ x − y < δ we have f ( x) − f ( y ) < ε .

1
Therefore f ( x) = is uniformly continuous on R.
1 + x2
Ex.14. Find the radius of convergence R of each of the power series. Discuss the convergence of the
power series at the points x − t = R .

x 2 n −1 ∞
n( x − 1) n
(i) ∑ (2n − 1)!
n =1
(ii) ∑
2n
n =1

Sol: (i)

x 2( n +1) −1
(2(n + 1) − 1)! x2
lim = lim =0
n →∞ x 2 n −1 n →∞ (2n + 1)(2n)

(2n − 1)!
So, by the ratio test, the series converges absolutely for all x, and so R = ∞ .
(ii)

(n + 1)( x − 1) n +1
2n +1 n +1 x −1 x −1
lim n = lim =
n →∞ n( x − 1) n →∞ n 2 2
n
2

So, by the ratio test, the series converges absolutely if x − 1 < 2 and diverges if x − 1 > 2 .

Thus, R = 2 . If x − 1 = 2 then the power series diverges.

Ex.15. Prove the following series converge conditionally


1 2 1 1 2 ∞ (1− n )
(ii) ∑ (−1) n
n
(i) 1 + − + + − + .... n
2 3 4 5 6 n =1

Sol: (i) Let {an } be the sequence 1,1, −2,1,1, −2,.....



1
Let bn =
n
. Note that the sequence of partial sums of ∑ an
n =1
is bounded and {bn } is a

decreasing sequence with limit 0.



1 2 1 1 2
By Dirichlet’s Test 1 + − + + − + .... = ∑ an bn converges.
2 3 4 5 6 n =1

(ii) Let an = (−1)n n−1 and Let bn = n n .Note that


1
∑ an
n =1
converges by Alternating series Test and

that {bn } is a bounded monotone sequence for n sufficiently large. We apply Abel’s Test

21
Ex.6 Analysis

∞ ∞ 1 ∞

∑ (−1)n n(1− n) / n = ∑ (−1)n n −1n n = ∑ anbn converges.


n =1 n =1 n =1

Ex.16. (i) Prove that f ( x ) = x is uniformly continuous on [0, ∞)


(ii) Prove that f ( x ) = x3 is not uniformly continuous on R.

Sol: (i) Given ε > 0 let δ = ε 2 . x− y ≤ x+ y .

2
Hence, if x − y < δ = ε 2 then we have x− y ≤ x− y x + y = x− y < ε2

Hence, x − y < ε . This shows that f ( x) = x is uniformly continuous on [0, ∞) .

(ii) Pick ε = 1 . Given any δ > 0 let x > 0 ∋


3δ x 2
2
( )
> 1 . Then d x + δ 2 , x < δ but we have

( ( , f ( x) ) = ( x + δ )
3δ x 2 3δ 2 x δ 3 3δ x 2
2)
3
d f x +δ − x3 = + 2 + 3 ≥ >1
2 2 2 2 2

This shows that f ( x) = x3 is not uniformly continuous.

Ex.17. Is the set Q( 2) = {a + b 2 : a, b ∈ Q} countable or uncountable?

Sol: {
The set of rational numbers is countable. Hence for any given a, the set a + b 2 : b ∈ Q is also }
countable. The set Q ( 2 ) is the union over all rational numbers a of the countable set
{a + b 2 : b ∈ Q} . Since the countable union of countable set is countable. We conclude that
Q ( 2 ) is countable.

⎧1 − x 2 if 0 ≤ x < 1

f ( x ) = ⎨ (1 + x )
Ex.18. Let
⎪ if 1 ≤ x < 2 . Show that f is not continuous in an interval [0, 2] .
⎩ 2

Sol: Consider that xlim f ( x ) = 0 and lim+ f ( x ) = 3 ∴ lim f ( x) does not exists. Hence f is not
→1− x →1 2 x →1

continuous at x = 1 . Hence f is not continuous on [0, 2] .

Ex.19. Determine which of the following series converge.


∞ ∞
1 1
(i) ∑ (ii) ∑
n=2 n log n n = 4 [ n(log n)(log log n)]

22
Analysis Ex.7

1 1 eω
Sol: (i) The integral ∫ x log x
dx is equalent to ∫
log u
du and
∫ω d ω . Which do not have elementary

antiderivatives (i.e, the antiderivative is not expressible as an algebraic combination of polynomi-


als, exponentials, logarithms, trig functions and inverse trig functions). So using the integral test
would be problematic. So, using comparison test n > log n for all n > 0 .

1 1 1 1 1
Hence, log n > and so > = > 0.
n n log n n n n

1 1
Since, ∑n diverges. ∴ ∑ diverges.
n log n

1
(ii) We can integrate ∫ x log x log(log x) dx = log(log(log x)) + c
∞ ∞
1 1
Notice that ∑ n log n log(log n) > ∫ x log x log(log x) dx = lim log log log b − log log log 4 = ∞
n=4
b →∞
4


1
Hence, ∑ n log n log(log n)
n=4
diverges.

Ex.20. Is the following functions uniformly continuous?


(i) f ( x) = 3x + 11 on R (ii) f ( x) = x 2 on [0,3]

ε
Sol: (i) Given ε > 0 , let δ = ε 3 . Then x − y < δ =
3
⇒ 3 x − y = 3x − 3 y = 3x + 11 − (3 y + 11) = f ( x) − f ( y) < ε . So, the function f ( x) = 3 x + 11 on
R is uniformly continuous.
ε ε ε
(ii) Given ε > 0 , let δ = Then x − y < δ = ⇒ x 2 − y 2 = x − y x + y ≤ x − y .6 < (6) = ε .
6 6 6
So the function is uniformly continuous on [0,3] .

Ex.21. Find the interior of each set

(i) [0,3] ∪ (3,5) {


(ii) r ∈ Q : 0 < r < 2 } {
(iii) r ∈ Q : r ≥ 2 }
Sol: (i) [0,3] ∪ (3,5) . The interior is (0,5) .

{ }
(ii) r ∈ Q : 0 < r < 2 . The interior is φ . Since the irrationals are dense in R.

(iii) {r ∈ Q : }
r ≥ 2 . The interior is φ .

Ex.22. Determine which of the following subsets of R is compact.


(i) [1,3) (ii) { x ∈ Q : 0 ≤ x ≤ 2}

23
Ex.8 Analysis

⎛ 1⎞
Sol: (i) Let f = { An } , where An = ⎜ 0,3 − ⎟ for n ∈ N . Then f is open cover for [1,3) which has no finite
⎝ n⎠
subcover. So [1,3) is not compact.
(ii) Because the rationals are dense in R. We can construct an infinite collection of rational points that
1
get close to 2 , but never reach it . Let x1 = 2 , ∵ x1 & 2 are real numbers, there is a rational

number between them, call it x2 . Now, since x2 ∈ R , there is some rational number

between x2 and 2 , call it x3 . Continue to get x1 < x2 < x3 ...... < xn < ..... 2 .
Now Let An = (1, xn ) . Then J = { An } is an open cover of our set, but it has no finit subcover..
{x ∈ Q : 0 ≤ x ≤ 2} is not compact.

Ex.23. Determine whether each series converges conditionally, converges absolutely or diverges.

(−1) n (−2)n
(i) ∑ (ii) ∑
n =1 log n n2

( −1) n 1 1 1
Sol: (i) an = so an = log n . Notice that log n < n , So log n > n . Thus by the comparison
log n

( −1) n
test, ∑ an diverges. On the other hand lim an = 0 , so ∑ log n
n =1
converges conditionally..

(−2)n 2n 2n log 2 2n log 2


(ii) an = and lim an = lim == lim = lim = ∞ .Thus the series diverges.
n2 n2 2n 2

Ex.24. Find the radius of convergence R and the interval of convergence C for each series.
n2 n
(i) ∑ 2n x
n 2 2n +1 2n 2
Sol: R = lim n 2
= lim = 2.
2 (n + 1) ( n + 1) 2

n2
At x = −2 our series is ∑ 2n (−2)n = ∑ (−1)n n2 which diverges.
n2
At x = 2 our series is ∑ 2n (2)n = ∑ n2 Which diverges. Thus C = (−2, 2) .

24
OBJECTIVE TYPE QUESTIONS (PART-B)

sin( nt ) (c) 1/g is not continuous on a, b


1. fn ∑n
n2
(d) 1/g is continuous on a, b
n −n 2
gn ∑(−1) 2 5. Let g be defined as
n

(a) f n converges absoluted but g n is not ⎧⎪2n ( x − n) 1 if n − 2−n ≤ x ≤ n


⎪⎪
converges g ( x) ⎪2n (n − x ) 1 if n ≤ x ≤ n 2−n

(b) g n converges absoluted but f n is not ⎪⎪
⎪⎪0 otherwise
converges ⎩

(c) both f n and g n are converges absoluted. Then


∞ ∞
(d) both f n and g n are diverges.
(a) ∫ g converges but ∑ g 1
diverges
2. Let M be a compact subset of a complete metric 1

space ( X , d ). Then ∞ ∞

(b) ∫g diverges but ∑g converges


(a) x → dist ( x, M ) inf d ( x, y ) is a
y∈M
1 1

continuous function on X ∞ ∞

(b) x → dist ( x, M ) d ( x, y ) is not a (c) ∫ g and ∑ g both are converges


1
1
continuous function on X
∞ ∞
(c) x → dist ( x, M ) d ( x, y ) is a continuous
function on X
(d) ∫ g and ∑ g both are diverges.
1
1

(d) x → dist ( x, M ) inf d ( x, y ) is not a


y∈M
6. The closed unit ball B x ∈ ∞ : x ∞ ≤ 1 is
continuous function on X
(a) closed
3. Let h and j be differentiable functions on an open (b) bounded
interval I. Suppose that a, b ∈ I satisfy a b (c) compact
(d) closed and bounded but not compact
and h(a) h(b) 0. Then
(a) h ′( x ) 7. Let xn , yn , zn be bounded sequence
h( x ) j ′( x) 0 for some x ∈ (a, b)
such that every n, xn ≤ yn ≤ zn and such that
(b) j ( x) h( x) j ′( x) for some x ∈ [ a, b)
(c) h ′( x ) j ′ ( x ) for all x ∈ [a, b] lim sup zn ≤ lim inf xn . Then

(d) j ′( x ) h ′( x) j ( x) 0 for some x ∈ [a, b] (a) lim xn lim yn lim zn


(b) lim xn ≤ lim yn
4. Suppose that g is integrable on a, b and that
(c) lim xn ≤ lim zn
there exists k 0 ∋ g ( x) ≥ k for all x ∈ a, b . (d) lim xn lim yn lim zn
Then
8. If xn is a decreasing and ∑ xn converges then
(a) 1/g is integrable on a, b
lim nxn 0 which of the following imply true?
(b) 1/g is not integrable on a, b

25
OB.2 Analysis

1 ⎪⎧1 x ∈ Q
(a) ∑ converges if 0 ≤ S ≤ 1 g ( x) ⎨⎪
n ns ⎪⎪⎩0 x ∉ Q
1 (a) g ( x) is bounded variation
(b) ∑ diverges if 0 ≤ S ≤ 1
n ns
(b) g ( x) is not a bounded variation
1
(c) ∑ converges if S ≤ 1 (c) g ′ ( x ) exists
n ns
(d) None of these
1
(d) ∑ diverges if S ≥ 1 ∞
1
n ns 14. The series ∑ (−1)
n 1
n−1

2n 3
tan θ
9. lim (1− 5cot θ ) (a) convergent
θ→π
2
(b) absolutely convergent
(a) 0 (b) 1 (c) conditionally convergent
(c) e−5 (d) log 5 (d) divergent
15. Find the sets of points of discontinuity for the
1 n ⎛ πk ⎞⎟
10. n→∞ ∑ cos ⎜⎜⎜ ⎟⎟
lim function f :[−1,1] → R defined by
nk 0 ⎝ 2n ⎠
⎧⎪⎪ x if x is irrational
(a) 2π (b) 1 (c) 0 (d) 2 π f ( x) ⎨
⎪⎪⎩0 if x is rational

11. Let C ′( R ) denote the set of all continuously (a) [−1,1] (b) (−1,1)
differentiable real valued functions defined on
(c) [−1,1] \ 0 (d) φ
the real line. Define

A f ∈ C ′(R) f (0) 1, 16. Let f : 0,1 → R be twice differentiable with

f ′′ ( x ) 0 for all x ∈ 0,1 . If f (0) 0 and


f (1) 1, f ′( x ) ≤ 1 for all x ∈ R
2
f (1) 1. Then
where f ′ denotes the derivative of the function
(a) f (d ) d for some d ∈ 0,1 iff f ′(1) 1
f. Pick out the true statement.
(a) A is an infinite set (b) f (d ) d for some point d ∈ 0,1 iff
(b) A is an empty set
(c) A is a finite and non empty set f ′(1) 1
(d) None of these
(c) f (d ) ≠ d iff f ′(1) 1
12. The function g ( x) x with domain [−1,1] (d) f ′(1) 1 iff f (d ) d for all d ∈ 0,1
is
(a) uniformly continuous but does not satisfy a 17. An absolutely continuous function is Lipschitz
Lipschitz condition. only if
(b) uniformly continuous and satisfy a Lipschitz (a) g is bounded
condition.
(b) g ′ is bounded
(c) continuous and compact
(d) not uniformly continuous (c) g is continuous but not bounded

13. A function g : 0,1 → R (d) None of these

26
Analysis OB.3

18. Find the sets of points of discontinuity for the (a) g is not a constant
function f :[0, ∞) → R defined by (b) g is not differentiable
(c) g is constant
⎧⎪( x) if x is even (d) g ′( y ) ≠ 0
f ( x) ⎪

⎪⎪1− ( x) if x is odd
⎩ 23. Let f n be a sequence of functions defined on
where x is the largest integer less than or equal 0,1 . Determine nlim n 2 x (1− x 2 ) n
→∞

to x and ( x) x− x . (a) 1 (b) 0


(c) ∞ (d) 2
(a) [0, ∞) (b) [−1,1]
24. Let f n be a sequence of functions defined on
(c) φ (d) [−1,1] \ 0
0,1 . Evaluate nlim x (1 − x 2 ) n
19. Pick out the true statements →∞

(a) sin x − sin y ≤ x − y for all x, y ∈ R (a) 0 (b) 1


(c) log 2 (d) e
(b) sin 2 x − 2 y ≤ x − y for all x, y ∈ R
25. Let g be a bounded function on a, b , so that
2 2
(c) sin x − sin y ≥ x − y for all x, y ∈ R
there exists M 0 ∋ g ( x) ≤ M for all x ∈ B.
3 3
(d) sin x − sin y ≥ x − y for all x, y ∈ R Then

20. Let g be a continuous function from R to R. (a) U ( g , P) − L( g , P) ≤ 2 M ⎡⎣⎢U ( g 2 , P ) − L ( g 2 , P )⎤⎦⎥

(a) x : g ( x) 0 is a open subset of R. for all partitions P of [a, b)

(b) ⎡⎣⎢U ( g , P ) − L ( g , P )⎤⎦⎥


2 2
(b) x : g ( x) c is a closed subset of R. U ( g , P) − L( g , P)

(c) x : g ( x) c is a closed subset of R. for all partitions P of (a, b]

(d) x : g ( x) 0 is a closed subset of R. (c) U ( g , P) − L( g , P) ≥ M 2 ⎡⎣⎢U ( g 2 , P) − L( g 2 , P)⎤⎦⎥


for all partitions P of [a, b)
21. Let g be a function of bounded variation on a, b .
(d) U ( g 2 , P ) − L ( g 2 , P ) ≤ 2M U ( g , P) − L( g , P)
Then
b for all partitions P of a, b
(a) ∫ g ′ ≤ Vab g where V b g is the variation of g.
a 26. Find the sets of points of discontinuity for the
a
function f :[−1,1] → R defined by
b

(b) ∫ g ′ ≥ V g where Vab g is the variation of g.


b
a ⎧⎪⎪1 if x is irrational
f ( x) ⎨
a
⎪⎪⎩0 if x is rational
b
(a) (−1,1) (b) [−1,1]
(c) ∫ g ′ ≠ Vab g
a (c) φ (d) [−1, ∞)
b
27. Let f n be a sequence of functions defined on
(d) ∫ g′ 0
a
0,1 Find nlim nx (1 − x 2 ) n
→∞

22. Suppose that g : R → R has the property that (a) 1 (b) ∞


g ( x) − g ( y ) ≤ x − y 2
for all x, y ∈ R (c) 0 (d) e− 1 2

27
OB.4 Analysis

28. Let g : (−1,1) → R be a differentiable function ⎛ ⎞


2
⎛ ⎞⎟
such that there exists a limit (b) ⎜⎜⎜ sup g ( x) ⎟⎟⎟ ≤ 4 ⎜⎜ sup g ( x) ⎟
⎝ x∈ a ,∞ ⎠⎟ ⎝⎜ x∈ a ,∞ ⎠⎟⎟
g ( x)
lim L∈R sup g ′( x) ≤ sup g ′′( x)
x→ 0 x2 (c) x∈ a ,∞ x∈ a ,∞

(a) second derivative g ′′(0) exists and equals L 2


⎛ ⎞ ⎛ ⎞⎟
(b) second derivative g ′′(0) does not exists (d) ⎜⎜⎜ sup g ′′( x) ⎟⎟⎟ ≤ 4 ⎜⎜ sup g ( x) g ′( x) ⎟
⎝ x∈ a ,∞ ⎠⎟ ⎝⎜ x∈ a ,∞ ⎠⎟⎟
(c) second derivative g ′′(0) exists and equals to
32. Suppose that g : R → R is a differentiable at a
2L
(d) second derivative g ′′(0) exists and equal to and that g (a ) 0. Then which of the following
0. imply true.
(a) If h( x) g ( x) , h is differentiable at a then
29. If hn ≤ g ∈ L ′ Ω then hn are uniformly
g ′(a ) 0
integrable on Ω.
(b) If h( x) g ( x), h is differentiable at a iff
(a) There exist a family of functions hn that is
uniformly integrable but there is no integrable g ′( a ) 0
function g so that hn ≤ g. (c) If h( x) g ( x), and g ′( a ) 0 then h is not
(b) There exist a family of functions hn that is differentiable at a
not integrable but there is a integrable (d) if h is not differentiable at a then g ′( a ) 0 g ′′ ( 0 )

function g so that hn ≤ g.
G ′
f ′

g ( x) h( x )
lim n s in
n → ∞

∑ X n

(c) Does not exist any family of function.


33. What is the radius of convergence of the
(d) hn is uniformly integrable and hn ≤ g .

zn
30. Let f : 0,1 → R be a continuous function. following series ∑
n 0 n!

1 n
⎛K ⎞ (a) 1 (b) -1
Then nlim ∑ f ⎜⎜ ⎟⎟⎟ ? (c) ∞ (d) 0
→∞ nK 0 ⎜⎝ n ⎠

1
34. Let K ∈ 0, ∞ be a real number. Define
(a) 0 (b) ∫ f (t )dt
⎧⎪t K sin 1 t ≠ 0
0 ⎪ t
f K (t ) ⎨
⎪⎪0 t 0
1
⎪⎩
(c) 1 (d) ∫ f 1 (t )dt
0
A K ∈ 0, ∞ f K is differentiable
31. Let g be a twice differentiable function on the
then A ?
interval (a, ∞)
(a) 0,∞ (b) 1,∞
2
⎛ ⎞ ⎛ ⎞
(a) ⎜⎜⎜ sup g ′( x) ⎟⎟⎟ ≤ 4 ⎜⎜ sup g ( x) ⎟⎟⎟ (c) −∞, ∞ (d) 0,∞
⎝ x∈ a ,∞ ⎠⎟ ⎝⎜ x∈ a ,∞ ⎠⎟
35. Suppose that f : a, b → R is Riemann
⎛ ⎞
⎜⎜ sup g ′′( x ) ⎟⎟⎟
⎜⎝ x∈ a ,∞ ⎠⎟ integrable on a, b

28
Analysis OB.5

b
b−a n ⎛ b − a ⎞⎟ sin 2 x − sin 2 y ≤ K x − y for all real numbers x
(a) ∫ f ( x)dx lim
n →∞

n i 1
f ⎜⎜a
⎜⎝
i
n ⎠⎟
⎟ and y?
a
1
b n (a) (b) 2
lim ∑ f a 2
(b) ∫ f ( x)dx
n →∞
i 1
i (b − a )
(c) 1 (d) -1
a

x 2 sin 1
b n
⎛a b⎞ 40. Consider the functions K ( x)
(c) ∫ f ( x)dx ≤ lim ∑ f ⎜⎜ i ⎟⎟⎟ x and
n →∞
i 1
⎜⎝ b n⎠
a
h( x ) x 2 sin 1
b
b−a n x2
(d) ∫ f ( x)dx ≥ lim
n →∞
∑ f (b)
n i 1 (a) h( x) is of bounded variation on [−1,1]
a

36. Let f(x) be continuous with (b) K ( x) is of bounded variation on [−1,1]


x (c) both h( x) and K ( x) are bounded variation
2

∫ f (t )dt K e( x−1) Find f and the constant K


on [−1,1]
1

(d) K ( x) is not a bounded variation on [−1,1]


(a) f ( x) 2( x −1)e x , K 1

(b) f ( x)
2 41. Let g k be a sequence of functions of bounded
2( x −1)e( x−1) , K −1
variation on [a, b]. If V [ g k ; a, b] ≤ M ∞
(c) f ( x) 2 xe( x−1) , K 0
3 for all k and g k → g pointwise on [a, b].
( 2 π e n !)
(d) f ( x) 4e( x−1) , K 1
(a) g is not a bounded variation
37. Let a smooth function f have the properties (b) g is of bounded variation and
f (0) 4, f (1) 0, f (3) 6. Then which of V [ g ; a, b] ≥ M
the following is true? (c) g is of bounded variation and V [ g ; a, b] ≤ M
(a) at some point 0 c 3 one has f ′′(c ) 0 (d) g is of bounded variation and V [ g ; a, b] M
(b) at some point 0 c 1 one has f ′′(c ) 0
42. In the plane R 2 is provided with the Lebesgue
(c) at 0 c 3 one has f ′′(c ) 0 measure, what is the measure of the set
(d) at 0 c 1 one has f ′′ ( c ) 0 A x, y ∈ R 2 x 2 y2 1
(a) 1 (b) 0
38. Suppose that f is real valued, bounded on a, b
(c) ( x, y) (d) ∞
and f 3 is Riemann integrable on a, b . Then
43. Let f be a real valued function on R. Consider the
(a) f 2 is Riemann integrable.
functions w j ( x ) sup f (u ) − f (v ) :
2
(b) f is not Riemann integrable
⎡ 1 1⎤
1 1 u, v ∈ ⎢ x − , x ⎥ where j is a positive integer
⎢ j j ⎥⎦
(c) ∫ f ( x)dx lim ∫ f ( x)dx
c→0

0 c and x ∈ R. Define next
(d) f 2 is discontinuous ⎧⎪ 1⎫
Aj , n ⎨ x ∈ R : w j ( x) ⎪⎬ ,
⎪⎩⎪ n 1, 2,... and
39. What is the least value of K 0∋ n ⎪⎭⎪

29
OB.6 Analysis

∞ 47. A real valued function on an interval a, b is


An ∪A
j 1
j ,n n 1, 2,...
said to be a function of bounded variation if
∃ M 0 ∋ for any finite set of points
Now let C x ∈ R : f is continuous at x
a a0 a1 a2 ... an b we have
Express C interms of the set An
n−1
∞ ∞
∑ f (ai ) − f (ai 1 ) M. Which of the
(a) C ∪A
n 1
n (b) C ∩A
n 1
n i 0

following statements are necessarily true?


(c) C An (d) C 0 (a) Any monotone function on 0,1 is not of
44. Let f be a continuous real valued function on R bounded variation.
x
d (b) If f : R → R is continuously differentiable,
and n a positive integer find dx ∫ (2 x − t ) f (t )dt
n

0
then its restriction to the interval −n, n is
x
bounded variation on that interval, for any
(a) 2n ∫ (2 x − t ) f (t )dt
n−1
x n f ( x) positive integer n.
0
(c) Any continuous function on 0,1 is of
x
bounded variation
∫ (2 x − t )
n
(b) f (t )dt f ( x)
0 (d) f : R → R is not differentiable
x
⎛1⎞
(c) ∫ (2 x − t )
n −1
f (t ) dt x n f ( x) 48. g ( x) x 2 sin ⎜⎜ ⎟⎟⎟ for x ≠ 0 and g (0) 0
⎜⎝ x ⎠
0
which of the following imply true?
x

(d) ∫ (2 x − t )
n
f (t )dt (a) g is differentiable at x 0 and g ′(0) 0
0
(b) g ′ is continuous at x 0
45. Suppose that ( f n ) is a sequence of decreasing (c) g is not differentiable at x 0
functions f n : 1,3 → R which converges (d) g ′ is continuous for all x.
pointwise to 0. Then
49. Which of the following is true?
(a) ( f n ) converges uniformly to -1 2
(a) 1 x 2 ≥ e x for all x ∈ R
(b) ( f n ) does not converges uniformly 2
(b) 1 x 2 ≤ e x for all x ∈ R
(c) ( f n ) converges uniformly to 0
(c) x 2 ex
2
1 for all x ∈ R
(d) ( f n ) converges to 1
2
(d) e x ≤ x 1 for all x ∈ R
46. Suppose that g : a, b → R and that g ′′ exists
everywhere 50. Let g : 0,1 → R be a continuous function with
(a) g ′′ ( x ) ≥ 0 for all x ∈ a, b iff g is convex continuous second derivative g ′′ and
(b) g ′′ ( x ) ≤ 0 for all x ∈ a, b iff g is compact g (0) g ′(1) 0.
1
(c) g ′′( x ) 0 for all x ∈ a, b iff g is integrable
(a) If ∫ g ′( x)g ′′( x)dx 0 then g ≡ 0
(d) g ′( x) ≥ 0 for all x ∈ a, b then g is bounded 0

30
Analysis OB.7

1 (a) convergent
(b) conditionally convergent
(b) If g ≡ 0 then ∫ g ( x)g ′′( x) 0
(c) absolutely convergent
0
(d) divergent
1

(c) If ∫ g ( x)g ′′( x)dx 0 then g ≡ 0 56. Let rn be an enumeration of the set Q, then
0

1
there exists a subsequence rns such that
(d) If g ≠ 0 then ∫ g ( x)g ′′( x) 0
(a) slim rn
→∞ s
∞ (b) slim rn
→∞ s
−∞
0

51. If g : R → R satisfies (c) slim rn 0 (d) slim rn0 1


→∞ s →∞ s

d g ( x), g ( y ) ≤ d ( x, y)2 for all x, y ∈ R . Then 57. Suppose that g ( x) is a continuous function on
(a) g is constant (b)g is polynomial the interval a, b ∋ g ( x) 0 for all x.
(c) g is nonconstant (d)None of these
a b
52. Let f be a differentiable function of one variable
∫ g 1 ( x)dx ∫ g ( x) g ( x)dx
1
(a) 0 (b) 0
and let g be the function of two variables given b a

by g ( x, y ) f (ax by ) where a, b are fixed non b b


zero numbers. Write down a partial differential
equation satisfied by the function g
(c) ∫ g ( x) dx 0 (d) ∫ dx 0
a a

∂g ∂g ∂g ∂g ∞
n
(a) b ∂x a
∂y (b) a ∂x b
∂y 58. The series ∑ (−1)
n

n 1 n 2
∂g ∂g ∂g ∂g (a) convergent
(c) b ∂x −a (d) ∂x (b) conditionally convergent
∂y ∂y
(c) absolutely convergent
(d) divergent
53. Let g be a continuous function on a, b and

n log n
assume that g (a ) 0 g (b). Let 59. The series ∑ (−1)
n 1
n

en
W x ∈ a, b , g ( x) 0 and let (a) convergent
w sup W .
(b) conditionally convergent
(c) absolutely convergent
(a) g ( w) ≠ 0 (b) g ( w) 0 (d) divergent
(c) g ( w) 0 (d) g ( w) 0 60. Let xn be a sequence of nonnegative numbers
54. Let K be a fixed positive integer. Find RK the and
radius of convergence of the power series (a) If lim xn 0 then lim xn 0

⎛n 1 ⎞⎟
n2
(b) If lim xn 0 then lim xn ≠ 0
∑ ⎜⎜ ⎟ z Kn
⎜⎝ n ⎠⎟
(c) If lim xn ≠ 0 then lim xn ≠ 0
(a) 1 (b) 0
(c) e 1 K (d) ∞ (d) If lim xn 0 then lim xn ≠ 0


1 61. Suppose that there exists N o ∋ an ≤ bn for all
55. The series ∑ (−1) n

2n 3
is
n 1 n No

31
OB.8 Analysis

(a) if lim an ∞ then lim bn ∞ 1 5


(a) (b)
(b) if lim an −∞ then lim bn ∞ 15 112
(c) 1 (d) 103
(c) if lim an ∞ then lim bn −∞
67. If f is an increasing real-valued function of a real
(d) if lim an ≠ −∞ then lim bn −∞ variable then
(a) f has at most countable number of
62. The sequence of functions sin(nx), n ≥ 1 is
discontinuities.
(a) equicontinuous in c 0,1 (b) f does not have discontinuities
(b) bounded in c 0,1 (c) f has at most countable number of continuities
(c) compact in c 0,1 (d) None of these
(d) not a equicontinuous in c 0,1 68. The minimum value of the function
63. Let hn be a function ∋ hn : (0,1) → R. Then f ( x, y ) x2 5 y 2 − 6 x 10 y 6 is
(a) If hn → h and hn′ → f then h is continuous (a) 10 (b) 6
(c) 5 (d) -8
on (0,1) and h1 f
69. The interval of convergence of the series
(b) If hn → h then h is not continuous on (0,1)
2 3 4
x 1 x 1 x 1
(c) If f n → f then hn′ does not exists ( x 1) − − ... is
4 9 16
(d) If hn → h and hn′ → f then h is not (a) −2 ≤ x ≤ 0 (b) −1 x 0
continuous on (0,1) and h ′ f (c) x ≤ 0 (d) −2 ≤ x ≤ 1
64. Suppose G : R → C. There is a constant M such 70. Which of the following series is diverges
that G ( x) − G ( y ) ≤ M x − y for all x, y ∈ R 1 n
(a) ∑ (b) ∑ 1 2 50 2
n
iff n!
(a) G is continuous almost everywhere ∞
1 log n
(c) ∑ ∑
(b) G is absolutely continuous and
2n n (d)
n
G ′ ≥ M almost everywhere n 2

(c) G is absolutely continuous and G ′ ≤ M 71. The function g ( x) x 3 is not uniformly


almost everywhere continuous on
(d) G is continuous and G ′ does not exist. (a) 0,1 (b) 0,1
x
sin xy (c) R (d) 0,3
65. Let x 0. Define f ( x) ∫ y
dy Evaluate
0 72. A differentiable real valued function on R with
f ′( x ) as a function of x. bounded derivative is
(a) continuous
sin x sin( x 2 ) (b) uniformly continuous
(a) (b) 2 (c) bounded
x x
(d) not continuous
(c) cos x (d) 1
n2
66. Find the coefficient of x7 in the Taylor series 73. Evaluate nlim
→∞ n!
expansion of the function f ( x) sin −1 x around (a) 0 (b) 1
0 in the interval −1 x 1 (c) e (d) ∞

32
Analysis OB.9

74. Find nlim


→∞
(1 a ... a n ) for a 1 (a) 0 (b) 1 2

1 (c) 1 (d) π 2
(a) 1 (b)
1− a
(c) 0 (d) ∞ 80. Let f : R → R be a continuously differentiable
75. Determine the interval of convergence of the function ∋ f ′(0) 0. Define x and y ∈ R

( x −1) n
series ∑ (−1)
n 1
n−1

n
g ( x, y ) f x2 y2 .

Pick out the true statements


(a) (0,1) (b) (0, 2]
2
(c) (−∞, ∞) (d) [1, 2] (a) g is differentiable only on R \ 0, 0
(b) g is differentiable function on R 2
76. What is the cardinality of the following set
iff f (0) 0
A f ∈ C1 0,1 : f (0) 0, f (1) 1,
(c) g is differentiable function on R 2
1
f (t ) ≤ 1 for all t ∈ 0,1 (d) None of these
(a) ∞ (b) 0 ⎪⎧ nx ⎪⎫
(c) −∞ (d) 1 81. The sequence of functions ⎨ ⎬
⎪⎩⎪1 xn⎪⎭⎪ over the
⎡ n⎤
⎢ ⎥
⎢ ⎥
1 ⎣ 2⎦ ⎛ K π ⎞⎟ ⎡n⎤ interval [0, 2] is
77. Evaluate lim ∑ cos ⎜⎜ ⎟
⎟ where ⎢⎢ ⎥⎥ denotes (a) convergent
n→∞ n
K 1
⎜⎝ n ⎠ ⎣ 2⎦
(b) uniformly convergent
the largest integer not exceeding n 2 . (c) not uniformly convergent
(d) absolutely convergent
(a) 1 π (b) π 2 ∞
sin x
(c) 2π (d) 0
82. The series ∑n
n 1
2
1
over R is

(a) convergent
1
78. Let f ( x) consider its Taylor expansion (b) uniformly convergent
1 x2 (c) absolutely convergent
about a point a ∈ R , given by (d) divergent

83. Let g n be a sequence of functions of X which
f ( x) ∑ a ( x − a)
n
n
. What is the radius of
n 0 converges pointwise on X. Let G ⊂ X be finite
convergence of this series?
(a) g n converges uniformly on G
(a) 1 (b) a2 1
(b) g n does not converges
(c) ∞ (d) a
79. Let f :[−1,1] → R be continuous. Assume that (c) g n converges but not uniformly converges
on G
1 1
(d) None of the above
∫ f (t )dt 1. Evaluate lim ∫ f (t ) cos 2 ntdt
n →∞
−1 −1 84. Pick out the sequences f n which are uniformly
convergent

33
OB.10 Analysis

sin x 88. Let g n be a sequence of real valued measurable


(a) f n ( x) on R
n functions on X ∋ for each n ∈ N
(b) f n ( x) x n on 0,1 ⎛⎪⎧ 1 ⎪⎫⎪⎟⎞ 1
μ ⎜⎜⎨ x ∈ X : g n ( x) − g n 1 ( x) ⎬⎟
⎜⎝⎪⎩⎪ 2 n ⎪⎭⎪⎠⎟⎟ 2n
(c) f n ( x) nxe nx on 0,∞
(d) All the above (a) g n is converges but not pointwise

85. Let an be a sequence of positive real numbers convergent


(b) g n is pointwise convergent μ almost
a
∋ lim n 1 r 1 find lim an everywhere on X.
n →∞ a n→∞
n
(c) g n diverges
(a) 1 (b) 0
(c) ∞ (d) a (d) g n is not a cauchy sequence
n

n 1 89. The space C a, b equipped with L1 norm i
86. The series ∑ n 5 is 1
n 1 n 4
b

(a) convergent (b)uniformly convergent defined by g 1 ∫ g ( x) dx is


(c) absolutelyconvergent (d) divergent a

87. L(x) is Lipschitz, if there is a constant M ∋ for (a) compact


(b) convergent sequence
any L( x) − L( y ) M ( x − y ) for any x and y..
(c) uniformly convergent sequence
Which of the following imply true? (d) incomplete
(a) If the function G ( x ) is absolutely continuous 90. Consider the sequence of continuous functions
on a closed interval a, b then the function 1 2 cos 2 (nx)
given by g n ( x)
L G( x) is also absolutely continuous on n
(a) g n ( x) converges to the constant 0 function
a, b
on R
(b) If the function L G( x) is absolutely (b) g n ( x) converges uniformly to the constant
continuous on a, b then the function G ( x) 0 function on R
is absolutely continuous on a closed interval (c) g n ( x) converges to 0,1
a, b (d) g n ( x) does not converges

(c) If the function G ( x) is continuous on a, b n


1
91. Evaluate nlim
→∞

then the function L G( x) is absolutely K 1 4n − K 2
2

continuous on a, b (a) π (b) π 6

(d) The function L G( x) is absolutely (c) 2 π (d) 0

92. Evaluate nlim n sin(2π en !)


continuous on a, b then the function G ( x) →∞

(a) 4 π (b) 8 π
is continuous on a, b (c) 2 π (d) ∞

34
Analysis OB.11

93. Which of the followig functions are uniformly (c) Z


continuous? (d) Every finite subset of R
(a) f ( x) sin 2 x, x ∈ R 1
⎪⎧⎛ 1 ⎞⎛ 2 ⎞⎟ ⎜⎛ n ⎞⎟⎪⎫⎪ n

98. Evaluate nlim ⎨⎪⎜⎜⎜1 ⎟⎟⎟⎜⎜⎜1 ⎟...⎜1 ⎟⎬


(b) f ( x) 1 ,
x x ∈ (0,1) →∞ ⎪⎝
⎩⎪ n ⎠⎝ n ⎠⎟ ⎝⎜ n ⎠⎟⎪⎭⎪

(c) f ( x) x 2 , (a) 0 (b) 1


x∈R
(d) None of these (c) 4 e (d) ∞

1 ⎛1⎞
94. The series ∑ tan ⎜⎜ ⎟⎟⎟ is ⎛ 1
⎜⎜ 4 n2 ⎞⎟
⎜⎝ n ⎠ ⎟
n 1 n 99. Evaluate nlim
→∞ ⎜ 3 3
... 3
n ⎠⎟⎟ 3
⎝1 n 8 n n
(a) convergent
(b) uniformly convergent (a) 1 (b) 0
(c) absolutely convergent 1
(d) divergent (c) log 2 (d) ∞
3
95. Let f and g be differentiable functions
100. Find the points in R where the following function
(a) If f ′( x ) g ′( x) for all x, then is differentiable
f ( x) g ( x) k
⎧⎪tan −1 x if x ≤ 1
⎪⎪
(b) If f ′( x ) g ′( x ) for all x, then f ( x) k f ( x) ⎨π x −1
⎪⎪ Sgn( x) if x 1
(c) If f ′( x) 0 for all x, then g ( x) x ⎪⎪⎩ 4 2
(d) If g ′( x ) f ( x ) for all x, then where sgn (x) equals 1 if x 0,
g ( x) k g ′( x ) −1 if x 0 and is equal to zero if x 0 and
tan −1 ( x) takes its values in the range
96. Let  ∈ R and let f n be a real sequence
− π , π for real numbers x.
(a) If f n → a then the sequence . f n 2 2
converges to .a (a) R (b) Q
(c) N (d) R \ −1
(b) If f n → a then the sequence . f n does
not converges to .a 101. Let f , f n : 0,1 → R be continuous functions.
(c) If f n converges to zero then the sequence Complete the following sentence ∋ both
f n converges to . statements (i) and (ii) below are true. Let f n → f
.....................
(d) If the sequence .xn converges to .a then
1 1
fn → a lim ∫ f n ( x)dx
(i) n →∞ ∫ f ( x)dx
0 0
97. Which of the following sets is not a discrete
subset of R
(ii) nlim lim f n ( x)
→∞ n→ 0
lim lim f n ( x )
x → 0 n →∞
⎧⎪ 1 ⎫⎪
(a) S ⎨ n∈z ⎬ (a) continuous on 0,1
⎪⎩⎪ n ⎪⎭⎪

⎧⎪ 1 ⎫⎪ (b) uniformly on 0,1


(b) T ⎨ n∈z ⎬∪ 0
⎪⎩⎪ n ⎪⎭⎪ (c) uniformly on 0,1
(d) None of the above

35
OB.12 Analysis


n sin nx
102. Evaluate ∫∫ max x, y dxdy
108. The series ∑ , x ∈ 0, π
0,1 0,1 n 1 en
(a) 1 (b) 0 (a) uniformly convergent
(b) not uniformly convergent
(c) 2 3 (d) log 2 (c) absolutely covergent
(d) None of the above
h
1
h→ 0 h ∫
103. Let f ∈ C −1,1 Evaluate lim f (t )dt xn
−h 109. The sequence f n ( x) , x ∈ 0, 2
1 xn
(a) 2 (b) 2 f (0) (a) uniformly convergent
(c) f (0) 1
(d) 0 (b) not uniformly convergent
(c) absolutely covergent
1 n 1 ⎛⎜ K ⎞⎟ (d) None of the above
104. Let f ∈ C1 −1,1 . Evaluate nlim
→∞
∑f ⎜ ⎟
n K 1 ⎜⎝ 3n ⎠⎟ x
1− e xy
⎡ ⎤ 110.Compute F ′( x) where F ( x) ∫ y dy, x 0
(a) 3 ⎢ f 13 − f (0)⎥ −x
⎣ ⎦
2 2

⎡ (a) e x2 (b) 2(e x − e x ) / x


(b) 3 ⎢ f 1 3 f (0)⎤⎥
⎣ ⎦ (c) cos x 2 (d) 0
(c) f
1 1 1
f (0)
3 111. Let a 0 and let K ∈ N.
(d) 0 ⎛
n
K
j ⎞⎟
π
lim
Evaluate n→∞ a −nK
∏ ⎜⎜a
j 1⎝
⎜ ⎟
n ⎠⎟
→∞ ∫
105. Let f ∈ C −π, π Evaluate nlim f (t ) cos nt dt
K ( K 1)
−π (a) e 2a (b) log K ( K 1)
(a) 0 (b) -1
(c) 2π (d) ∞ (c) 0 (d) a n

π
112. nlim sin ⎡⎢ 2nπ 1 sin 2nπ 1 ⎤
⎣ 2nπ 2nπ ⎥⎦ is
→∞ ∫
106. Let f ∈ C −π, π Evaluate nlim f (t ) cos2 nt dt →∞

−π (a) 0 (b) ∞
π (c) sin1 (d) log(sin n)
1
(a) 0 (b) 2 ∫ f (t )dt
−π 1 1
113. nlim (n 1)(n 2)...(n n) n
is

→∞n
(c) ∫ f (t )dt (d) cos2 nt (a) 0 (b) ∞
0
(c) 4 e (d) log 4
nx
107. The sequence f n ( x ) , x ∈ 0, ∞ 114. Find the points where the following function is
1 nx differentiable
(a) uniformly convergent
(b) not uniformly convergent ⎧⎪tan −1 x if x ≤ 1
⎪⎪
(c) absolutely covergent f ( x) ⎪⎨
π x −1
(d) None of the above ⎪⎪ if x 1
⎪⎪⎩ 4 x 2

36
Analysis OB.13

(a) R \ −1 (b) 0,∞ ∞

(c) ∑ an converges
(c) −∞, ∞ (d) C n N 1


115. For no distinct p, q is
(d) ∑a n
2
converges
(a) Lp ⊃ Lq (b) Lq L p n 1

(c) Lp ⊂ Lq (d) Lp 0 119. Suppose that g : R → R is continuous on R and

116. Let ( X , d ) be a metric space that g (r ) 0 for every rational number r. Then
(a) a subset M ⊂ X is unbounded if the (a) g ( x) 0 for all x ∈ R
function d is bounded on M M (b) g ( x) 0 for all x ∈ R
(b) a subset M ⊂ X is bounded if the function
(c) g ( x) 0 for all x ∈ R
d is bounded on M M
(c) a subset M ⊂ X is bounded if the function (d) g ( x) ≠ 0 for all x ∈ R
d is unbounded on M M 120. Which of the following sequences / series of
(d) a subset M ⊂ X is unbounded if the functions are uniformly convergent on 0,1 ?
function d is unbounded on M.
(a) f n ( x) n 2 x(1− x 2 ) n
117. Let g n : R → R be defined by
2n
(b) f n ( x) cos π n ! x
1
g n ( x)
⎛ 2 ⎞⎟
n3 ⎜⎜ x − 1 1 and let g ( x) 0 ∞ cos m 6 x
⎝ n ⎠⎟⎟ (c) ∑ m 1 m3
(a) g n ( x) → g ( x) for each x then g n converges (d) All the above
uniformly to g.
(b) g n ( x) → g ( x) for each x then g n converges 121. Let f ∈ C ′ 0,1 . For a partition
to zero ( P) : 0 x0 x1 x2 ... xn 1 define
(c) g n ( x) → g ( x) for each x but g n doesn’t n

converge uniformly to g. S ( P) ∑
i 1
f ( xi ) − f ( xi−1 ) . Compute the
(d) g n ( x) → 0 for each x then g n converge to
supremum of S(P) taken over all possible
zero. partitions P
118. Let an be a sequence of non negative real 1 1

∞ (a) ∫ f ′(t ) dt (b) ∫ f (t )dt


numbers and suppose that ∑a
n 1
n diverges. Then
(c) 0
0

(d) 1
0

122. Let Dn be the open disc of radius n with centre


∞ at the point ( n, 0) ∈ R 2 . Then there exist a
(a) ∑ an converges
n 1 function f : R 2 → R of the form f(x, y)=ax+by


(b) ∑n 1
an diverges
∋ ∪D n ( x, y ) f ( x, y ) 0 . The value of a
n 1

and b is

37
OB.14 Analysis

(a) a 1, b 0 (b) a 0, b 1 2
129. Let f ( x) x x x for x ∈ R, where x
(c) a −1, b 0 (d) a 0, b −1 denotes the largest integer not exceeding x. what
is the set of all values taken by the function f?

K2
123. Find the sum of the series ∑ K!
K 1
(a) Q (b) R
(c) R \ 0 (d) R \ −1
(a) 2 n (b) 2e
130. Let n be a positive integer. Let
(c) ( K 1) n (d) K −n
1
124. The radius of convergence of the power series f ( x) xn 2
sin
if x ≠ 0 and let f (0) 0.
x

For what value of n will f be twice differentiable

K 1
log K x k is but with its second derivative discontinuous at
(a) K (b) 1 x 0.
(c) 0 (d) ∞ (a) 1 (b) 2
(c) 0 (d) 3
125. What are the values of α ∈ R for which the
131. What is the coefficient of x8 in the expansion of
∞ n
(−1)
following series is convergent ∑
n 1 nα
x 2 (cos x 2 ) around x 0
(a) 1 (b) 0
(a) α ≥ 0 (b) 0 α ∞ (c) -1 (d) 2
(c) α ≥ 1 (d) α 1
132. Let g : R → R be a differentiable function
126. nlim n2 n − n2 1 ∋ g ′(x) ≤ M for all x ∈ R. For what values of
→∞

(a) 1 (b) 0 ε will the function f ( x) x ε g ( x) be


necessarily one to one?
(c) 1 2 (d) ∞
1 1
(a) ε (b) ε M
n
1 M
127. nlim ∑ is
→∞
k 1 n2 k2 (c) ε M (d) ε 1
(a) 1 (b) 0
x2
t2
(c) 2 (d) log(1 2) 133. If f ( x) ∫e dx, x 1. then f ′( x)
x

128. Let f : R → R be a given function. Which of the


(a) 2 xe x4 − e x2 (b) e x2 e2 x
following is uniformly continuous
(c) 2 xe2 x (d) 0
ex
(a) f ( x ) on (0, 2) 134. Pick out the true statments.
x
(i) f ( x) ≥ 0 for x ≥ 0 and f ( x) ≤ 0 for
1
(b) f ( x) on (0,1) x≤0
x2
(c) for all x and y ∈ R (ii) f is a decreasing function on the entire real
line
1
f ( x) f ( y) ≤ x − y 2 (a) (i) and (ii) both are true
(b) (i) is true but (ii) is not

g ( x − n) (c) (ii) is true but (i) is not
(d) f ( x) ∑ n 1 2n
,x∈ R
(d) both (i) and (ii) are false

38
Analysis OB.15

135. Let f : −1, 2 → R be given by (a) continuous at x 1 , n∈ N


2
f ( x) 2 x 3 − x 4 −10. What is the value of x 1 , n∈ N
(b) uniformly continuous at x 2
where f assumes its minimum value
(a) 1 (b) -1 (c) continuous everywhere
(c) 2 (d) 0 (d) discontinuous
n

∑ Ke
1 Kx
142. Evaluate , x∈R\ 0
136. ∫ log xdx ? K 1
0

(a) 0 (b) 1 ne( n 2) x


− (n 1)e( n 1) x
ex
(c) -1 (d) ∞ (a) e x −1
2

137. Let X , d be a metric space and let E ⊂ X . For


ne ( n 2) x

x ∈ X define d ( x, E ) inf d ( x, y )
y∈E
(b) e x −1
2

Pick out the true statements


(a) d ( x, E) − d ( y, E) ≤ d ( x, y) for all x, y ∈ X ex
(c) x
e −1
(b) d ( x, E ) d ( x, y0 ) for some y0 ∈ E
(c) d ( x, E ) d ( x0 , y0 ) ne( n 2) x
ex
(d) e −1x
(d) All the above

∑X
1
143. If converges absolutely then
→∞ ∫
138. Find nlim sin n xdx n

−1
(a) ∑ X n2 converges
(a) 0 (b) 1
(c) -1 (d) π / 2 (b) ∑ X n2 diverges

⎛3 3 (c) ∑ X n2 neither converges nor diverges


3 3 3⎞
lim ⎜⎜ −
139. Find n→∞ − ... (−1) n ⎟⎟⎟
n
⎜⎝ 2 4 8 16 2 ⎠ (d) ∑ X n−2 converges
(a) 1 (b) -1
(c) 0 (d) ∞ 144. Let ∑ an ( x − t ) n and ∑ bn ( x − t ) n have raddi of

140. Let [ x] denote the largest integer less than, or converges M 1 and M 2 respectively. If
equal to x ∈ R. The function x −t M where M min M1 , M 2 then
2
f ( x) [ x ]sin π x, x 0 is (a) ∑ an ( x − t ) n converges and is equal to
(a) continuous at x n, n ∈ N ∑ an ( x − t ) n ∑ bn ( x − t ) n
(b) uniformly continuous at x n, n ∈ N (b) ∑ an bn ( x − t ) n converges absolutely
(c) continuous everywhere
and is equal to ∑ an ( x − t ) n ∑ bn ( x − t ) n
(d) discontinuous at x n, n ∈ N , n ≠ K 2
(c) ∑ an ( x − t ) n diverges and is equal to
141. Let [ x] denote the largest integer less than or
∑ an ( x − t ) n
equal to x ∈ R, The function
(d) None of the above
, x ≥ 1 2 is
[ x]
f ( x) [ x] x − [ x]

39
OB.16 Analysis

145. Let M be a metric space such that M is a finite set. (c) a n f (a ) − a n−1 f ′( a )
Then
(a) Every subset of M is closed (d) a n f ′( a )
(b) Every subset of M is open
(c) Every subset of M is both open and closed 151. Let f : R → R be a function which is
(d) Every subset of M is not closed differentiable at x a Evaluate
146. Let k ∞ be the set of all real sequences ⎡ K j ⎤
lim n ⎢⎢ ∑ f a − Kf (a)⎥⎥
an ∋ an ≤ 1 for every n. Then n→∞ n
⎣j 1 ⎦
an − bn K2
(a) ∑ 2 n converges (a)
2
f ′( a ) (b) ( K 1) f ′(a )

an K ( K 1)
(b) ∑1 an
converges (c)
2
f ′( a ) (d) f ′ ( a )

an − bn 152. Let xn

be a sequence of real numbers. Pick
(c) ∑ 2 n diverges n 1

out the cases which imply that the sequence is



(−1) n cauchy.
(d) ∑
n 0 n 1
diverges
1
(a) xn − xn 1 ≥ for all n
2n
147. let xn ∈ 2 , g :  2 → R by g yn ∑x n yn .
(b) xn − xn ≤ 1 for all n
n 1 n
(a) g is continuous
(b) g is not continuous (c) xn − xn ≤ 1
1
n 2 for all n
(c) g is unbounded
(d) g is constant (d) xn − xn ≥ 1
1
n 2 for all n
148. Any compact metric space M is
(a) not separable 1
(b) complete 153. Let hn ( x ) and g n ( x) nx (1− x) n
1 n2 x 2
(c) continuous
(d) closed for x ∈ 0,1 .
149. If g is a continuous, periodic function from R to a (a) hn converges
metric spece M, then
(a) g is not continuous on R (b) g n does not converges
(b) g is uniformly continuous on R
(c) g is contraction mapping from R to R (c) hn and g n converges pointwise but not
(d) M is separable
uniformly on 0,1
150. Let f : R → R be a function which is
(d) hn and g n converges pointwise and
differentiable at x a . Evaluate
also uniformly continuous on 0,1
a n f ( x ) − x n f (a )
lim
x→a x−a 154. Suppose that g 1 ( x ) exists and is bounded. Then
(a) a n f ′( a ) − nf (a ) (a) g is continuous
(b) g is uniformly continuous
(b) a n f ′( a ) − na n−1 f (a ) (c) g is not continuous
(d) g is constant

40
Analysis OB.17


1 1

∑ (−1)
x2
n
sin 1 is
→∞ ∫
160. nlim
155. The series n e n
dx
n 1 n
−1

(a) convergent (a) 2 (b) 1


(b) absolutely convergent (c) 0 (d) ∞
(c) conditionally convergent
(d) divergent nx 2 3
3

161. Evaluate n→∞ ∫ x3 nx dx


lim

n −1
156. The series ∑ (−1)
n 1
n

(n 1)(n 2) is (a) 4 (b) 2


(c) 1 (d) 0
(a) convergent
(b) absolutely convergent 162. The function f ( x) x sin x is
(c) conditionally convergent (a) continuously differentiable
(d) divergent (b) differentiable but not continuously
differentiable
157. The function f ( x) x sin 1 (c) not differentiable
x if x ≠ 0 and
(d) None of the above
f (0) 0 is
163. Sum of the following infinite series
(a) continuously differentiable
(b) differentiable but not continuously 1 5 5.8 5.8.11
...
differentiable 6 6.12 6.12.18 6.12.18.24
(c) not differentiable
(d) None of the above 1 13
(a) 5 1 2 −1 (b) 4 −1
2
158. The function f ( x) x 3 2 is
(c) 2 −1 (d) 0
(a) continuously differentiable
(b) differentiable but not continuously 5 cot x
cos ecx

differentiable 164. lim (1 3x2 ) x


x→ 0
(c) not differentiable
(d) None of the above (a) e6 (b) log 6
(c) 1 (d) 0
159. Let gn be a sequence of real valued

continuous functions defined on 0,1 and 165. The series ∑
n 1
3
n3 1 − n is

assume that g n → g uniformly on 0,1 . Then (a) convergent


1 1
(b) absolutely convergent
lim ∫ g n ( x ) dx (c) conditionally convergent
(a) n →∞
1
∫ g ( x ) dx
(d) divergent
0
n

1 3 5 7
1
166. The series ... is
lim ∫ g n ( x)dx 0 1.2.3 2.3.4 3.4.5 4.5.6
(b) n →∞
1
n
(a) convergent
(b) uniformly convergent
1 1
(c) conditionally convergent
lim ∫ g n ( x ) dx ∫ g ( x ) dx (d) divergent
(c) n →∞
1 0
n
167. Let g, h be continuous on a closed interval
1
a, b with h( x) 0 for x ∈ a, b suppose
(d) ∫ g ( x)dx 1
0

41
OB.18 Analysis

g n → g and hn → h as n → ∞ uniformly on (c) not differentiable at (0, 0)


a, b . Then (d) limit exist

172. xlim x log 1 x − log x


1 gn g 2 2
(a) h is defined for large n and h → h
→∞

n n (a) 2 (b) 1
(c) 0 (d) ∞
uniformly on a, b as n → ∞
π
2
1 gn g
(b) h is defined for large n and h → h
n n
173. ∫ log tan θ d θ
0

uniformly on a, b as n → ∞ (a) 1 (b) 0

gn g (c) π 2 (d) π 4
(c) h → h is continuous on a, b as n → ∞
n ∞ 1

gn g
174. The series ∑ (n
n 1
n
−1) n is
(d) h → h is unbounded on a, b
n (a) convergent
(b) absolutely convergent
168. The geometric series
(c) conditionally convergent

1 (d) divergent
∑x
k 0
k

1− x

an
(a) converges on a, b 175. The series ∑n 1 n
is
(b) converges uniformly on any closed interval (a) convergent
a, b ⊂ (−1,1) (b) absolutely convergent
(c) diverges (c) conditionally convergent
(d) divergent
(d) converges on a, b ⊂ (−1,1)

n
sin x sin y 176. The series ∑ (−1) n

n 1
is
169. ( x , ylim
n 1
)→ (0,0) x 2 y 2 is (a) convergent
(a) 0 (b) 1 (b) absolutely convergent
(c) 2 (d) does not exist (c) conditionally convergent

x2 y4 (d) divergent
170. lim is
(x, y)→(0,0) x2 2y4

177. If α 1
(a) 0 (b)1 2 then
(c) 2 (d) limit does not exist ⎧⎪ xy α
⎪⎨ log( x 2 y 2 ) , ( x, y ) ≠ (0, 0)
g ( x, y )
⎧⎪ x 2 y 2 ⎪⎪⎩0 ( x, y ) (0, 0)
⎪⎪ ,0 ( x, y) π
g ( x, y) ⎪⎨ sin x2 y 2 is
171. ⎪⎪ is
⎪⎪0 ( x, y ) (0,0) (a) differentiable at (0, 0)

(b) not differentiable at (0, 0)
(a) differentiable at (0, 0)
(c) limit does not exist
(b) continuous at (0, 0) (d) none of the above

42
Analysis OB.19

(c) g has three fixed points


178. g ( x, y ) xy is
(d) g has infinitely many fixed points.
(a) differentiable at (0, 0)
184. Let a1 , a2 ,... be an enumeration of Q. For
(b) not differentiable at (0, 0)
each n ∈  , define
(c) limit exist at (0, 0)
⎧⎪ 1
(d) continuous at (0, 0) ⎪⎪ 2 n if x an
f n ( x) ⎨
⎪⎪0 if x ≤ an

⎪⎩
1
179. ∑n 3 is ∞
n 1

(a) converges (b)diverges


and g ( x) ∑fn 1
n ( x). Then which of the

(c) not bounded (d) decreasing sequence following imply true?


180. Let f : A → R and assume that g is bounded (a) g ( x) is uniformly convergent on R
function on A ⊆ R. (b) g ( x) is not monotone in R
(a) If lim
x→c
f ( x) g ( x) 0 then lim g ( x)
x→c
0 (c) g ( x) is continuous for x ∈ Q

(b) If lim f ( x) 0 then lim f ( x) g ( x) 0 (d) g ( x) is divergent.


x→ c x→ c

185. Find te coefficient x7 in the Taylor series


(c) If lim
x→c
f ( x) g ( x ) then lim g ( x )
x→ c
c
expansion of the function
(d) If lim f ( x) ≠ g ( x ) then lim g ( x) 0
x→c x→c
f ( x) log x 1 x 2 about the origin
181. g ( x) 1 is (a) -5/112 (b) 1/10
x
(c) 1 (d) 104
(a) uniformly continuous on 0,1
186. Let f : R → R be differentiable at x a.
(b) not uniformly continuous on (0,1]
Evaluate nlim f a 1 f a 2 ...
(c) uniformly continuous on (0,1] →∞ n2 n2
(d) continuous on 0,1
f a n − nf (a )
182. Suppose that g is an infinitely differentiable n2
function which satisfies g ′′ g′− g 0. If
(b) 1 2 f (a )
1
(a) f (0)
g (0) g (a) 0, then
(c) f 1 ( a ) (d) 1
(a) g ( x) 0 for all x ∈ 0, a
187. Let f : R → R be continuously differentiable
(b) g ( x) ≠ 0 for all x ∈ 0, a
1 n 1 k
(c) g ( x) 0 for x ∈ 0, a Evaluate: nlim
→∞
∑f n .
nk 1
(d) g ( x) 0 for x ∈ 0, a (a) f (1) (b) f (1) − f (0)
183. If g is differentiable on an interval where (c) 1 (d) ∞
g 1 ( x) ≠ 1 then 188. Assume that b 0. Let x1 b and let
(a) g has two fixed point
(b) g have at most one fixed point xn 1 b xn for all n ≥ 1. Then

43
OB.20 Analysis

(b) discontinuous
(a) xn 1 b and that xn is increasing
(c) uniformly continuous
(b) xn is diverges (d) jump continuous
1
(c) limit of xn is 0 ⎛ sin x ⎞⎟ x2
193. lim ⎜⎜ ⎟
(d) xn is decreasing
x→ 0 ⎜⎝ x ⎠⎟
(a) log 6 (b) 1
189. Let a, b ∈ R with a b. Let g n be sequence
−1

of continuous real valued functions that (c) 0 (d) e 6

converges uniformly on a, b . For every n ∈ w 1 n

x
194. nlim
→∞ n2

k 1
n2 − k 2

let hn ( x) ∫ gn
(a) π 2 (b) π 4
a

(a) hn converges uniformly on a, b (c) 1 (d) 0

(b) hn diverges 195. Compute f ( x ), f n ( x ) lim n 2 x (1 − x 2 ) n where


n →∞

(c) hn converges pointwise on a, b 0 ≤ x ≤1


(a) 1 (b) 0
(d) hn converges on a, b (c) 2 (d) ∞
190. For every n ∈ w and every x ≥ 0 but 196. The interval of convergence of the series

log(n 1)
g n ( x)
1
n
. ∑ n 1
( x − 5) n is
1 x n 1

(a) 4 ≤ x 6 (b) 2 ≤ x ≤ 6
(a) g n converges pointwise but not uniformly
(c) 5 ≤ x ≤ 6 (d) 4 ≤ x ≤ 6
on 0,∞
π
2
(b) g n converges uniformly but not poinwise sin 2 xdx
197. Evaluate ∫ sin x cos x
0
on 0,∞
1
(c) g n does not converges (a) 1 (b) log 2 1
2
(d) g n converges on 0,∞ (c) 0 (d) 2π
191. a ∈ A is isolated iff 198. If g and h are measurable functions then
(a) if there is ε 0 ∋ B(a; ε) ∩ A a (a) the set x : g ( x) ≠ h( x) is measurable.

(b) if there is ε 0 ∋ B(a; ε) ∪ A A (b) the set x : g ( x) h( x) is measurable

(c) if there is δ 0 ∋ B(a; δ ) \ A a (c) x : g ( x) h ( x ) , x : g ( x ) ≥ h( x ) ,

(d) if there is δ 0 ∋ B(a; δ ) a x : g ( x) h( x) are measurable


192. A differentiable real valued function on R with (d) the set x : g ( x) h( x) is not measurable
bounded derivative is
(a) continuous 199. Let g n be a nonegative measurable functions

44
Analysis OB.21

(a) If g n → g and g n ≤ g for all n then A


(a) f′ 1 ≤ and f′ x ≤A
2 for
2 4
∫ gn → ∫ g
0 x 1
(b) If g n → g and g n ≥ g for all n then
A
(b) f 1 2 ≤ and f′ x ≤A
2 for
4
∫ gn → ∫ g
0 x 1
(c) If ∫ g n → ∫ g then g n → g and g n ≥ g
A
(c) f′ x ≤ and f ′ 1 ≤ A for 0 x 1
4
(d) If ∫g n → ∫ g then g n → g and g n ≥ 0 for
all n (d) f ′′ x ≤ f ′ x − f ′ 1 2
200. Let S and T be nonempty bounded subsets of R
with S ⊂ T . Then 204. lim(1 − sin x cos x) co sec 2 x =
x→0

(a) inf T ≤ inf S ≤ sup T ≤ sup S 1


(a) (b) log 1 2
(b) inf T ≤ inf S ≤ sup S ≤ sup T e
(c) sup T ≤ sup S ≤ inf T ≤ inf S (c) 0 (d) ∞

(d) sup T ≤ sup S ≤ inf S ≤ inf T 1 n

205. nlim
→∞ n6
∑k
k 1
5

201. Let f ( x) be a periodic function.

(a) If xlim f ( x) exists then f ( x) is a constant (a) 6 (b) 1 6


→∞

function. (c) 0 (d) 1



(b) If xlim f ( x) does not exists then f ( x) is a cos nx
→∞ 206. The series ∑ (−1) n

n2
, x ∈ R is
constant function n 1

(a) convergent
(c) xlim
→∞
f ( x) does not exist
(b) absolutely convergent
(d) If xlim f ( x) exists, then f ( x) is a non (c) conditionally convergent
→∞
(d) divergent
constant function

n
202.Which of the following function is uniformly 207. ∑ (−1) n

n 2
continuous n 1

(a) convergent
(a) f ( x) e x on [0, ∞)
(b) absolutely convergent
(b) f ( x) x sin x on [0, ∞) (c) conditionally convergent
(d) divergent
(c) f ( x) Inx on (0,1)
⎪⎧⎪0 if x ∈[a,b]∩Q
(d) f ( x) x on [0, ∞) 208. Let 0 b and f (x) ⎨
a ⎪⎪⎩x if x ∈[a,b] is irrational
203. Consider a function f(x) whose second derivative Find the upper and lower Riemann integrals of
f ′′ ( x ) exists and is continuous on 0,1 . f ( x) over [a, b]
Assume that f (0) f (1) 0 and suppose that (a) L( P, f ) 0 and U ( P, f ) b−a
there exists A 0 ∋ f ′′( x) ≤ A for x ∈ 0,1 . (b) L( P, f ) a and U ( P, f ) b

45
OB.22 Analysis

(c) L( P, f ) b and U ( P, f ) a −b In(n 1)


(d) S n
(d) L( P, f ) 0 and U ( P, f ) a n −1
212. Suppose that ∑ xn is a series of positive terms
209. Let f :[a, b] → R be a Riemann integrable
which is convergent. Then
function. Let g :[a, b] → R be function ∋ 2
(a) ∑ x n and ∑ xn xn 1 are convergent
x ∈ [a, b], f ( x) ≠ g ( x) is finite. Then which 2
(b) ∑ x n and ∑ xn xn are divergent
of the following is true? 1

2
b b (c) ∑ x n is convergent and ∑ xn xn 1 is
(a) ∫ f ( x)dx ≠ ∫ g ( x)dx
divergent.
a a
2
b b
(d) ∑ xn xn 1 is convergent ∑ x n is divergent
(b) ∫ f ( x)dx ∫ g ( x)dx 213. Every convergent sequence in a metric space.
a a
(X,d) is
b b (a) cauchy sequence
(b) convergent sequence
(c) ∫ f ( x)dx ≤ ∫ g ( x)dx
(c) divergent sequence
a a
(d) bounded sequence
b b
214. For what values of P does the following series
(d) ∫ f ( x)dx ≥ ∫ g ( x)dx
converge?
a a

1 1 1
⎪⎧⎪1 0 ≤ x ≤ 1 1− − p ...
210. Consider the function g ( x) ⎨ 2p 3 p
4
⎪⎪⎩0 1 x ≤ 2
(a) P 0 (b) P 0
x (c) P 0 (d) P 1
G ( x) ∫ g (t )dt on 0, 2 .Then ∞
cos nα
0
215. The series ∑ (−1)
n 1
n

n2
where α ∈ R is a
(a) G ( x) is continuous
fixed real number
(b) G ′( x ) ≠ ( x ) for x ∈ (0, 2) with x ≠ 1 (a) absolutely convergent
(c) G ( x) is continuous and differentiable expect (b) convergent
(c) uniformly convergent
at 1
(d) divergent
(d) G ( x) is dicontinuous ∞
n log n
⎛ K K 1 ⎞⎟ 216. Th series ∑ (−1) n

en
is
⎜⎜ ⎟ n 1
∞ ⎜⎝ ( K 1) k ⎠⎟⎟ (a) convergent
211. Find the sum of the series ∑ In
K 1 K K 1 (b) uniformly convergent
(c) absolutely convergent
In(n 1) (d) divergent
(a) Sn −
n+1 217. Consider the sequence fn defined by
In(n −1)
(b) S n nx
n-1 f n ( x) for x ∈ [0, 2]
e nx
In( n)
(c) S n −
n

46
Analysis OB.23

(a) The converges is uniform on [0, 2] ∞


sin(nx)
(b) The converges is not uniform on [0, 2]
(d) ∑
n 1 n
0, 2π

(c) nlim
→∞
fn 1 for x ∈ [0, 2] 220. Let g n : 0,1 → R be continuous ∋ g n are

(d) The convergence is uniform on 0,1 unfiromly bounded on 0,1 and the derivatives

218. Suppose that gn is a sequence of funtions gn′ exist and are uniformly bounded on (0,1).
Then
defined on D and M n is a sequence of
(a) g n is a divergent subsequence
nonnegative numbers such that g n ( x) ≤ M n (b) g n has a uniformly convergent subsequence
for every x ∈ D for every n ∈ N . Then
(c) g n has convergent subsequence
∞ ∞
(d) None of the above
(a) if ∑M
n 0
n converges then ∑g
n 0
n ( x)
221. For each n ≥ 1, let f n be a monotonic increasing
converges uniformly on D.
∞ ∞ real valued function on 0,1 ∋ the sequence of
(b) If ∑g
n 0
n ( x) converges then ∑M
n 0
n
functions fn converges pointwise to the
converges function f ≡ 0. Pick out the true statements from
∞ ∞
the following
(c) If ∑M
n 0
n diverges then ∑g
n 0
n ( x)
(a) If the functions f n are also non-negative
converges uniformly on D. then f n must be continuous for sufficiently
∞ ∞
large n.
(d) If ∑ g n ( x) diverges then ∑M n converges
n 0 n 0 (b) f n converges to f uniformly..
219. Which of the following is uniform convergence (c) f n diverges
(a) g n ( x ) nx (1− x) on 0,1
n (d) None of the above
2 2 −nx
222. The sequence of functions n x e over the

cos 2 (nx)
(b) ∑ on R
n 1 n2 interval 0,∞
(a) convergent

xn
(c) ∑ on R (b) uniformly convergent
n 1 n! (c) not uniformly convergent
(d) None of the above

47
OB.24 Analysis

ANSWERS

1. (c) 2. (a) 3. (a) 4. (a) 5. (a) 6. (d) 7. (a) 8. (b) 9. (c) 10. (d)

11. (b) 12. (a) 13. (b) 14. (c) 15. (c) 16. (b) 17. (b) 18. (c) 19. (a) 20. (d)

21. (a) 22. (c) 23. (b) 24. (a) 25. (d) 26. (b) 27. (c) 28. (c) 29. (a) 30. (b)

31. (a) 32. (b) 33. (c) 34. (b) 35. (a) 36. (b) 37. (a) 38. (a) 39. (c) 40. (b)

41. (c) 42. (b) 43. (b) 44. (a) 45. (c) 46. (a) 47. (b) 48. (a) 49. (b) 50. (c)

51. (a) 52. (a) 53. (c) 54. (c) 55. (b) 56. (a) 57. (c) 58. (d) 59. (c) 60. (a)

61. (a) 62. (d) 63. (a) 64. (c) 65. (b) 66. (b) 67. (a) 68. (d) 69. (a) 70. (d)

71. (c) 72. (b) 73. (a) 74. (b) 75. (b) 76. (d) 77. (a) 78. (b) 79. (b) 80. (c)

81. (c) 82. (a,b,c) 83. (a) 84. (a) 85. (b) 86. (a) 87. (a) 88. (b) 89. (d) 90. (b)

91. (b) 92. (c) 93. (a) 94. (a,c) 95. (a) 96. (a) 97. (b) 98. (c) 99. (c) 100.(d)

101.(b) 102.(c) 103.(b) 104.(a) 105.(a) 106.(b) 107.(b) 108.(a) 109.(b) 110.(b)

111.(a) 112.(c) 113.(c) 114.(a) 115.(c) 116.(b) 117.(c) 118.(b) 119.(a) 120.(c)

121.(a) 122.(a) 123.(b) 124.(b) 125.(b) 126.(c) 127.(d) 128.(d) 129.(b) 130.(b)

131.(b) 132.(b) 133.(a) 134.(d) 135.(b) 136.(c) 137.(a) 138.(a) 139.(a) 140.(d)

141.(c) 142.(a) 143.(a) 144.(b) 145.(c) 146.(a) 147.(a) 148.(b) 149.(b) 150.(b)

151.(c) 152.(c) 153.(c) 154.(b) 155.(b) 156.(c) 157.(c) 158.(a) 159.(a) 160.(a)

161.(a) 162.(a) 163.(b) 164.(a) 165.(a,b) 166.(a) 167.(b) 168.(b) 169.(d) 170.(d)

171.(c) 172.(a) 173.(b) 174.(a) 175.(a) 176.(d) 177.(a) 178.(b) 179.(a) 180.(b)

181.(b) 182.(a) 183.(b) 184.(a) 185.(a) 186.(b) 187.(b) 188.(a) 189.(a) 190.(a)

191.(a) 192.(a,c) 193.(d) 194.(b) 195.(b) 196.(a) 197.(b) 198.(c) 199.(a) 200.(b)

201.(b) 202.(d) 203.(a) 204.(a) 205.(b) 206.(b) 207.(d) 208.(a) 209.(a) 210.(c)

211.(a) 212.(a) 213.(a) 214.(c) 215.(a) 216.(c) 217.(b) 218.(a) 219.(b) 220.(b)

221.(b) 222.(c)

48
EXPLANATIONS
sin(nt ) 1 1
1. (c) fn ∑n . As sin(nt) ≤1, an ≤ 2 .
∵ ∑ n 2 converges. So ∑ n an converges by
n2 n n
sin(nt ) 2

comparison test ∵ f n ∑n converges absolutely.. g n ∑(−1)n 2−n . For n ∈ N,−n2 ≤−n


n2 n

⇒0 2 −n2
≤ 2 and so
−n
∑ (−1) 2
n 1
n −n2
converges by comparison with the convergent geometric

series ∑2
−n
∴ g n converges absolutely..
n 1

2. (a) Let ε 0 , choose δ ε . Given x, y ∈ X with d ( x, y ) δ we can find z ∈ M so th at


2

d ( x, z) dist ( x, M ) ε . ∵ dist ( y, M ) ≤ d ( y, z ) .
2

dist ( y, M ) ≤ d ( y, z ) ≤ d ( x, y ) d ( x, z ) δ dist ( x, M ) ε dist ( x, M ) ε.


2
Thus dist ( y, M ) − dist ( x, M ) ε. Reversing the roles of x, y in the above argument gives
dist ( x, M ) − dist ( y, M ) ε ⇒ dist ( y, M ) − dist ( x, M ) ε, So, the function is continuous.
zn
3. (a) Consider f ( x) h( x)e j ( x ) , f (a) f (b) 0 . So by Rolle’s theorem f ′( x) 0 for some x ∈ (a, b).

But f ′( x) h( x)e j ( x) j ′( x) h′( x)e j ( x) e j ( x ) ⎡⎣ h( x) j ′( x) h ′( x)⎤⎦ , e j ( x) ≠ 0

1 1
4. (a) Cosider the function g . Rewrite this as a composite function h and g g ( x) then h  g 1/ g
x

h is continuous on every point except 0 and ∵ g ( x) ≥ k 0 we know h is continuous on the range c, d

where g a, b ⊂ c, d . ∵ The composite of a continuous and integrable function is integrable.

∴ h  g is integrable on a, b which means 1/g is integrable on a, b .


5. (a) The graph g is a sawtooth which is1 at each integer, a narrow triangle near each integer, and 0
∞ ∞
1
otherwise. The sum ∑ g diverges; yet each triangle has are 2−n , so ∫ g ( x)dx ∑ 2 n 1
n
1.
1

6. (d) The ball is clearly closed and bounded, but not compact, consider the sequence xn in B where in
each xn is a sequence of all zeros, except for a 1 in the nth position. Whenever n ≠ m we have
xn − xm ∞ 1, so that no subsequence can be cauchy, let alone convergent.

7. (a) ∵ xn ≤ zn for every n .Then lim inf zn ≤ lim sup zn ≤ lim inf xn ≤ lim inf zn .Hence
limsup zn liminf zn and so lim zn exists.Similarly lim sup xn ≥ lim inf xn ≥ lim sup zn ≥ lim sup xn .

49
BE.2 Analysis

Hence lim sup xn lim inf xn and so lim xn exists. Moreover lim xn =limsup xn =limsup zn =lim zn

By squeeze theorem we get the result lim xn lim yn lim zn

8. (b) Fix ε 0 and find N ∋ n ≥ N . ⇒ na2n na2n ε . Now for n 2N we can find
2

m ≥ N ∋ n ≤ m ≤ n . Hence ma2m ε . Since x is a decreasing sequence th en


3 2 3 K

ε . Hence 1
man ≤ ma2m nan ≤ 3man ε, ∵ n ≤ 3m. Thus lim nan 0 Suppose that ∑
3 ns
⎛1⎞
converges. Then 0 lim n ⎜⎜ s ⎟⎟⎟ 1
lim n1−s . This is false if 0 ≤ S ≤ 1. Hence ∑ n s diverges.
⎜⎝ n ⎠

12. (a) The function g is uniformly continuous on [−1,1] because it is continuous and [−1,1] is a compact
set, so it is uniformly continuous. g does not satisfy a Lipschitz condition, suppose for a contradiction
that there is some M ∋ x − y ≤ M x − y for all x, y ∈ [−1,1]. Take y 0 and x 0, so

x ≤ Mx for all x 0. But then M ≥ 1/ x for all x 0, which is impossible.

⎪⎧⎪1 x ∈ Q
13. (a) g ( x) ⎨
⎪⎪⎩0 x ∉ Q , If P x0 , x1 ,..., xn is a partition of [a, b] then V ( g , P ) ∑ g xi − g xi−1 . In

particular, if the xi are taken to be alternately rational and irrational, then V ( g , P ) ≥ n − 2. For any
δ 0, take a partition P with P δ and V ( g , P) ≥ N for any given natural number N. It is clear that no
value of L can exist. ∴ g ( x) is not a bounded variation.

17. (b) If g is Lipschitz, then we have some M 0 ∋ for all x, y. g ( x) − g ( y ) ≤ M x − y . Then we have

g ( x) − g ( y )
that g ′( x) lim ≤ M.
y→ x x− y

20. (d) Let y be a limit point of x : g ( x) 0 . So there is a sequence yn ∋ yn ∈ x : g ( x) 0 for all n and
lim yn y. Since g is continuous. ∴ g( y) lim g( yn ) lim 0 0. Hence, y ∈ x : g ( x) 0 , so
n →∞ n→∞ n→∞

x : g ( x) 0 contains all its limit points and is closed subset of R.

21. (a) By Jordan’s decomposition theorem, the function g can be represented as the difference of the non-
decreasing functions i and h. (i.e) g i − h. Since i and h are non-decreasing, their derivatives exist
almost everywhere and are non-negative.
Using the formula Vab g Vab i Vab h
b b b b

∫ g ′ dx ∫ i ′ − h ′ dx ≤ ∫ i ′ dx ∫ h ′ dx
a a a a

50
Analysis BE.3

b b

∫ i ′dx ∫ h ′dx i (b) − i ( a ) h(b) − h(a )


a a

Vab i Vab h
Vab g

g ( x) − g ( y )
22. (c) Fix y ∈ R. For x ≠ y ≤ x− y
x− y

g ( x) − g ( y )
So, − x− y ≤ ≤ x− y
x− y

∵ lim − x − y 0 and lim x − y 0 . The squeeze theorem for limits implies that
x→ y x→ y

g ( x) − g ( y )
lim 0.
x→ y x− y

Thus, g is differentiable at y and g ′( y ) 0. Since this holds for all y ∈ R , g must be constant.

25. (d) For any x, y we have g ( x) 2 − g ( y )2 g ( x) g ( y ) g ( x) − g ( y ) ≤ 2 M g ( x) g ( y)

Let P a x0 x1 .... xn b be a partition of a, b . For any x, y ∈ ⎡⎣ xi , xi 1 ⎤⎦

⎛ ⎞
then g ( x) 2 − g ( y ) 2 ≤ 2M g ( x)

g ( y ) ≤ 2M ⎜⎜ sup g (t ) − t∈inf g (t )⎟⎟⎟ denote the number on
⎝t ∈ xi , xi−1
xi , xi−1 ⎟ ⎠

the right by c. Then for any x ∈ ⎡⎣ xi , xi 1 ⎤⎦

g ( x) 2 − c ≤ g ( y ) 2 for all y ∈ ⎡⎣ xi , xi 1 ⎤⎦

⇒ g ( x ) 2 − c ≤ inf g (t ) 2 for all x ∈ ⎡ x , x ⎤


t ∈ ⎡⎣ xi , xi 1 ⎦⎤ ⎣ i i 1⎦
⇒ g ( x) 2 ≤ c inf g (t ) 2 for all x ∈ ⎡ x , x ⎤
t ∈ ⎣⎡ xi , xi 1 ⎦⎤ ⎣ i i 1⎦
sup g (t ) 2 ≤ c inf g (t ) 2
t ∈ ⎣⎡ xi , xi 1 ⎦⎤ t ∈ ⎡⎣ xi , xi 1 ⎤⎦

⎛ ⎞
sup g (t ) 2 − inf g (t ) 2 ≤ 2 M ⎜⎜ sup g (t ) − inf g (t )⎟⎟
and thus ⎜ ⎟
⎠⎟
t ∈ ⎡ x , x ⎤
t ∈ ⎣⎡ xi , xi 1 ⎦⎤ ⎣ i i 1⎦ ⎝ t ∈ ⎣⎡ xi , xi 1 ⎦⎤ t ∈ ⎣⎡ xi , xi 1 ⎦⎤

⎛ ⎞
2 2
We conclude that U g , P − L g , P ∑ ⎜⎜⎜ sup g (t ) 2 − inf g (t ) 2 ⎟⎟⎟ (length of ⎡ x , x ⎤ )
⎝t ∈⎡⎣ xi , xi 1 ⎤⎦ t ∈ ⎣⎡ xi , xi 1 ⎦⎤ ⎠ ⎣ i i 1⎦

⎛ ⎞
≤ 2 M ⎜⎜⎜ sup g (t ) − inf g (t )⎟⎟⎟ (length of ⎡⎣ xi , xi 1 ⎤⎦ )
⎝t ∈⎡⎣ xi , xi 1 ⎤⎦ ⎡ ⎤
t ∈ ⎣ xi , xi 1 ⎦ ⎠⎟

2 M U ( g , P ) − L( g , P )

51
BE.4 Analysis

g ( x) g1 ( x)
28. (c) Let g : (−1,1) → R , lim L ∈ R , lim g ( x) 0. ∴ By L’Hospital rule, lim 2L
x→ 0 x2 x→0 x→0 x
g ( x ) − g (0) g ( x)
g 1 ( x) → 0 as x → 0 , g ′(0) lim lim , because lim g ( x) 0 and g is continuous.
x→ 0 x x→0 x x→ 0

g ( x) g ′( x)
This limit exists since g is differentiable. By L’Hospital rule we get g ′(0) lim lim 0
x →0 x x →0 1
g ′( x) − g ′(0) g ′( x)
So, g ′(0) 0. ∴ g ′′(0) lim lim 2L.
x →0 x x →0 x

29. (a)
sup ∫ hn dx ≤ ∫ g dx
. To show that
lim
α →∞ ∫ g dx 0.
This is simply a consequence of
hn ≥ α g ≥α g ≥α

the Lebesgue dominated convergence theorem applied to gα 1 g which is dominated by g .


g ≥α

Consider hn (x) 1 . This is clearly uniformly integrable. (Choose α 1) but if g is a function


[ n, n 1]

∋ g ≥ hn for all n then g ≥ 1 on [1, ∞). Hence g is not integrable.

g ′′(c )
31. (a) Taylor’s theorem gives g ( y ) g ( x) g ′( x )( y − x ) ( y − x ) 2 for some c between x and y.
2
g ′′(c) 2 g ′′(c ) 2
Let y x h , g(x h) g ( x) g ′( x )h h ; − g ′( x ) h g ( x) − g ( x h) h
2 2

g ′′(c) 2
Taking absolute values g ′( x)h ≤ g ( x) g( x h) h . If Mk sup g ′( x) : x ∈ a, ∞
2

1 M2
then g ( x)h ≤ 2 M 0 h 2 ; b 2 − 4ac ≤ 0. 4 g ′( x ) 2 −16 M 0 M 2 ≤ 0 . Thus
2
g ′( x ) 2 ≤ 4 M 0 M 2 ∵ This is true for any x ∈ a, ∞ we can take the supremum over x on the left and
2
we get M 1 ≤ 4M 0 M 2 . Hence the result.
36. (b) Let x 1. Then 0 K e0 c 1 . So, K −1 .To find f ( x) take the derivative of both sides and
2
use the Fundamental Theorem of calculus f ( x) 2( x −1)e( x−1)

f (1) − f (0)
37. (a) By the Mean Value Theorem there are points a ∈ 0,1 and b ∈ 1,3 ∋ f ′(a ) −4
1− 0

f (3) − f (1) 6
and f ′(b) 3 . Therefore applying the Mean Value Theorem, f ′ we find there
3 −1 2

f ′(b) − f ′(a) 7 7
is a point c ∈ a, b ∋ f ′′(c) 0.
b−a b−a 3

52
Analysis BE.5
2
38. (a) φ ( y ) y 3 is continuous on R, so f 2 φ( f 3 ) is Riemann integrable because it is a continuous
function of a Riemann integrable function.

40. (b) Since K is C′ then ∃ M 0 ∋ K ( x) − K ( y ) ≤ M x − y for all x, y ∈ [−1,1] .Take


ake M sup K ′( x) .
Hence for any partition x0 x1 ... xm . Then we have ∑ K ( xi ) − K xi−1 ≤ M ∑ xi − xi−1 ≤ 2M
Hence K is of bounded variation on [−1,1]
k

41. (c) Being by fixing a partition Γ xi


k
of interval [a, b] , V [ g n ; a, b] sup ∑ g n ( xi ) − g n ( xi−1 ) ≤ M
i 0 Γ i 1

for all n. Further more g k → g pointwise,


k k
V [ g ; a, b] sup ∑ g ( xi ) − g ( xi−1 ) sup lim ∑ g n ( xi ) − g n ( xi−1 ) ≤ M
Γ Γ n→∞
i 1 i 1

45. (c) Let ε 0. Since ( fn ) converges to 0 pointwise ∃ N , M ∋ fn (1) ε for n ≥ N and fn (3) ε for
n ≥ M .Now, since each fn is decreasing, we have fn (1) ≥ fn ( x ) ≥ fn (3) for all n and x ∈ 1,3 . Thus for

n ≥ max N , M , we have for any x ∈ 1,3 −ε fn (3) ≤ fn ( x) ≤ fn (1) ε . So fn ( x) ε.

∴ fn converges uniformly to 0.

⎛1⎞
x 2 sin ⎜⎜ ⎟⎟⎟
g ( x) − g (0) ⎜⎝ x ⎠ lim x 2 sin ⎛⎜ 1 ⎞⎟⎟
48. (a) lim exists and is finite, lim g ( x) − g (0) lim = x→0 ⎜⎜ ⎟ ,
x→0 x−0 ⎝ x⎠
x→0 x−0 x→0 x

⎛1⎞ ⎛1⎞
− x ≤ x sin ⎜⎜ ⎟⎟⎟ ≤ x . Also lim − x lim x 0. By the squeeze theorem lim x sin ⎜⎜ ⎟⎟⎟ 0.
⎜⎝ x ⎠ x→ 0 x→ 0 x→0 ⎜⎝ x ⎠

g ( x)
∴ g is differentiable at x 0 and g ′(0) lim 0.
x→0 x

e y for y ∈ 0, x .
2
49. (b) If x 0; clearly 1 0 ≤ e0 1 assume x ≠ 0. Let g ( y ) 1 y 2 and h( y )
g and h are continuous and differentiable on 0, x , so by Cauchy’s mean value thorem, there is some

g ( x) − g (0) g ′( x) 1 x 2 −1 2c 1 1
c between 0 and x such that ≤ 1
h ′( x) , thus e x −1 e0
2
c2 c2
h( x) − h(0) e (2c) e
2 2
It is true for any x ≠ 0. ∴ x2 ≤ ex −1 ⇒1 x2 ≤ ex for all x ∈ R.

g ( x) − g ( y )
51. (a) For x ≠ y , ≤ x− y . Taking limit as x → y yielding g ′( y ) 0. This holds
x− y
for every y ∈ R, so that g must be constant.

53. (c) Since g (a ) 0, W is a bounded non-empty subset of a, b . We have w sup W with x ≤ w ≤ b for

53
BE.6 Analysis

all x ∈ W . To show that g ( w) 0 . Suppose g ( w) ≠ 0 then either g ( w) 0 or g ( w) 0


If g ( w) 0 then ∃ a neighborhood Vδ (w) (w − δ, w δ ) ∩ a, b ∋ g ( x) 0 for all x ∈ Vδ ( w).
In this case x ≤ w − δ w if x ∈ W and thus w is not the least upper bound of W. We get a
contradiction. If g ( x ) 0 then by a similar argument, we can find a neighborhood
Vδ ( w) ( w − δ , w δ ) ∩ a, b ∋ g ( x ) 0 for all x ∈ Vδ ( w). In this case we have Vδ ( w) ⊆ W and thus w
is not an upper bound of W. We also get a contradiction. This shows that g (w) 0.

56. (a) Let X 1 x ∈ N,r 0 and let x1 min X 1 . Let X 2 n ∈ N;n n, and rn rn1 1 and let
n2 min X 2 . Continue in this way to get n3 , n4 ,... so in general

ns min n ∈ N ; S ns−1 , and rns rns−1 1 . Then rns is an increasing sequence. Also for

ε 1 , rnp − rnq ε for all p , q ⇒ rns is not cauchy, so it diverges and rns 0 for all ns . Thus,
2
lim rns ∞.

57. (c) ∵ g ( x) ≠ 0 on a, b the function 1/g is defined and continuous on a, b . Here there is M > 0 so that
b
1 1
M for all x. g ( x) 0 for all x in a, b . Let D a, b . Then L( f , D) ≤ ∫ f .
g ( x) M a

b
1
However L( f , D) b−a 0 . ∴ ∫ g ( x)dx 0.
M a

60. (a) Given ε 0, let N be ∋ n N ⇒ xn − 0 ε 2 (such an N exists since lim Sn 0 ). But then

n N , ⇒ xn ε 2 , Hence xn ε , hence xn − 0 ε ⇒ lim xn 0

61. (a) Given M 0, let N1 be ∋ n N1 ⇒ an b such as N1 exists since lim an ∞.

Let N max N 0 , N1 . Then n N⇒M an ≤ bn so lim bn ∞.

62. (d) Suppose that the sequence is equicontinuous ∃ a δ so that for any n and x, y ∈ R with x − y δ

π π ⎛ π⎞
sin nx − sin ny δ . Then
1 . Let n be so large that −0 δ , sin ⎜⎜⎜n. ⎟⎟⎟ − sin( n.0) 1
2n 2n ⎝ 2n ⎠
a contradiction. The family is not equicontinuous.
x

63. (a) Let x0 ∈ (0,1) , hn ( x) hn ( x0 ) ∫ h′ (u)du for x ∈ (0,1) .


n
x0

x x

Take the limit as n → ∞ h( x) − h( x0 ) lim(hn ( x) − hn ( x0 )) lim ∫ hn′ (u )du = ∫ f (u )du .


n→∞ n→∞
x0 x0

Because uniform convergence allow for interchaning the limit with the integral.

54
Analysis BE.7

The equation h( x) − h( x0 ) ∫ f (u ) du .
x0

⇒ h ′( x) f ( x) for any x ∈ (0,1).

67. Let f− ( x ) lim f ( x − δ ),


δ→ 0

f ( x) lim f ( x δ)
δ→ 0

These one-sided limits exist because f is monotone and lence, f ( x − δ ) ≤ f ( x ) ≤ f ( x δ ) Moreover,,


these limits are finite, because f ( x − δ ) ≤ f− ( x) ≤ f ( x) ≤ f ( x) ≤ f ( x δ ) . f is discontinuous at a point

x iff f− ( x) f ( x). Since rational points are dense in R, there is a point qx ∈ f− ( x) f ( x) ∩ Q. The
mapping x → qx is injective; since if x1 x2 are two different points of discontinuity, then
⎛x x2 ⎞⎟
qx1 f− x1 ≤ f ⎜⎜ 1 ≤f x2 qx2 and hence qx1 ≠ qx2 . Thus we constructed an injective
⎝ 2 ⎠⎟

mapping from the set R f of all discontinuties of f to Q and Card R f ≤ Card Q .


⇒ f cannot have more than countable number of discontinuities.

log x (log x)2


70. (d) We can integrate ∫ x
dx
2
c use comparison test. Notice that for n ≥ 3,

log n 1
log(n) 1 Hence 0 for n ≥ 3.
n n
1 log n
Since ∑ diverges ⇒ ∑ diverges
n n

71. (c) For ε0 1 and any δ 0 there exist x, y ∈ R ∋ x − y δ and x 3 − y 3 ≤ 1 . To find x and y, let

δ .Then x 3 − y 3 δ
3 3 2 3 2 δ2 3 2
y x x3 − x xδ xδ x δ which is equal to 1 if
2 2 2 4 8 2

2 2 δ δ
x . For any δ 0, let x and let y x . Then x− y δ and
3δ 3δ 2 2
3δ 2
x3 − y 3 x 1 ξ0 so f is not uniformly continuous.
2
73. (a) Claim that n3 n ! for n ≥ 6. Let n 6. Then 63 216 720 6!. Hence this case is true.
3
Assume that K3 K ! for some K ≥ 6. Then (K 1) K3 3K 2 3K 1 K !3K 2
3K 1 K ! K !... K ! Where there are K + 1summands. Then ( K 1)3 (K 1) K ! (K 1)! ,
∴ n3 n ! for all n ≥ 6 .
n2 n2 n2 1 n2
So, n ! . lim 1 0 ⇒ lim 0
n! n3 n n→∞ n n →∞ n

55
BE.8 Analysis

1− a n 1 lim 1− a n 1
1− lim a n 1
n
74. (b) nlim (1 a ... a ) lim = .Since a 1, lim a n 0. So
→∞ n→∞ 1− a lim 1− a 1− a

1
lim (1 a ... an )
n →∞ 1− a

87. (a) To show that for any ε 0 there is δ 0 ∋ ∑ L G(ai ) − L G(bi ) ε if ∑ ai − bi δ. Fix ε 0.

0 ∋ ∑ G(ai ) −G(bi )
ε
Since G is absolutely continuous, there is δ M . ∑ ai − bi δ. For this

choice of δ and ∑ ai − bi δ one can estimate

∑ L G(ai ) − L G (bi ) ∑M G(ai ) − G (bi ) ε.

1
88. (b) Given x ∈ X , if ∃ N ∈  ∋ g n ( x) − g n 1 ( x) for all n ≥ N , then g n ( x) is cauchy, ∵ for
2n
1
all M ∈ M ∋ M ≥ N , n, m ≥ M ⇒ g n ( x) − g m ( x) m−1 .Let En {x ∈ X : gn (x)−gn 1(x) 1/2n}
2

Then ∑μ E
N ∈
n 1 ∞and so the Borel Cantelli lemma implies that μ almost every
x ∈ X lies in at

1
most a finite number of the En ’s. ∴ for μ almost every x ∈ X ∃ N ∋ g n ( x) − g n 1 ( x) ≤ for all
2n
n ≥ N . ∴ g n ( x) is cauchy and convergent. The sequence g n ( x) converges for μ almost all x ∈ X .

89. (d) To find a Cauchy sequence that does not converges with respect to i 1

⎧⎪0 x ∈ a, (b − a) / 2
⎪⎪
g n ( x) ⎪nx − n(b − a) / 2 x ∈ (b − a) / 2, (b − a) / 2 1/ n

⎪⎪ .
⎪⎪1 x ∈ (b − a ) / 2 1/ n, b

⎪⎧⎪0 x ∈ [a, (b − a ) / 2)
This converges to the step function g ( x) ⎨
⎪⎪⎩1 x ∈ [(b − a ) / 2, b)
which is not continuous. The

sequence gn − g 1 , then this is a Cauchy sequence but it converges to a function outside of C a, b .

So a, b , i 1 is incomplete.
9 9 1 ε2 1 ε
90. (b) Given ε 0, Let N . Let x ∈ R and n N 2 . Then
, So 3
ε2 ε n 9 n
3 1 2 cos 2 (nx) 3
ε , g n ( x) − 0 −0 ≤ ε . Thus g → g uniformly on R.
n n n n

95. (a) Consider the function f ( x) − g ( x) which is also differentiable. It derivative f ′( x) − g ′( x) 0.


Thus f ( x) − g ( x) is constant. So f ( x ) − g ( x) k for some k ∈ R . Thus f ( x) g ( x) k

56
Analysis BE.9

96. (a) Using 2 cases if  0 or if  ≠ 0. If  0 the . f n 0 is the constant 0 sequence. Thus, it converges
to 0 .a as desired. Assume that l ≠ 0, given ε 0. since f n → a then ∃ N ∋ for all n N

fn − a ξ
 . So for all n N . f n − .a  fn − a  . fn − a ξ. So . f n → .a

97. (b) To prove that 0 is not an isolated point. Let ξ 0. By the Archimedean principle, there exists an
1 1
n∈z ∋ 1 nε and thus 0 ε. Thus ∈ B 0; ε ∩ T . So it is not true that
n n
B 0; ε ∩ T 0 . Thus, 0 is not isolated and therefore T is not discrete.

115. (c) The idea is that high p requires functions diverge slowly, but does allowas them to decay slowly as well.
1 ∞

∫x ∫x
−a −a
For p a , dx ∞ . while p a, dx ∞ . So, x− p χ 0,1 is not in q for q p
L
0 1

while x− p χ[1, ∞) is not in Lq for q p . ∴ Lp ⊂ Lq .

116. (b) The function d is bounded on M M if ∃ k 0 ∋ d(x, y) ≤ k for all ( x, y ) ∈ M M (i.e) for all

x, y ∈ M . Hence, diam M sup d ( x, y ) ≤ k


x , y∈M

118. (b) Suppose the series ∑n 1


an converges. Then ∃ a natural number N with an 1 for all

n N . It follows that an an 1 for all n N and ∴ ∑


n N 1
an converges. Hence, adding the

finitely many terms a1 , a2 ,..., aN to this sum we obtain that ∑a


n 1
n converges which is a contradiction.

119. (a) Given any x ∈ R by the density of rational numbers we can find a sequence of rationals
rn ∋ rn → x. Then by continuity of g , g ( x) lim g rn 0.
1 1 1

→∞ ∫ ∫ ∫ 0dx
138. (a) nlim sin n xdx lim sin n xdx 0
n→∞
−1 −1 −1

1
3 3 3
139. (a) The sum of this geometrical series is 1− 1 1 − ...
. 1
2 2 4
2 2

143. (a) Fix N ∋ n ≥ N ⇒ X n ≤ 1. Then if n ≥ N we have X n2 ≤ X n . Hence ∑X


n N
2
n must converge since

∑ X n converge. Hence ∑X
n N
2
n converges as well. Since we are only adding finitely many terms to

the series.

57
BE.10 Analysis

145. (c) Every one-point subset y of a metric space is closed, because the only possible sequence of points
in this subset is the constant sequence yn y and this has limit y. Every subset X of M is the

complement of its complement X X ′ ′ . But since M is finite, X 1 is closed. ∴ X ′ ′ is open

146. (a) an − bn ≤ 2 for every n (by the triangle inequality). Hence the nth element in the series is dominated

an − bn
by 2−(n−1). Since ∑ 2−( n−1) 2 the series converges. ∴ ∑ converges
2n

147. (a)  2 is a normed vector space over R with norm xn 2 ∑ z n2 . The Cauchy-Schwarz inequality for

any yn ∈  2 g yn ≤ xn 2
yn 2
. Hence, g is bounded by the constant xn and so is
2

continuous.
149. (b) Fix ε 0 . Define G : 0, 2 p → M as G ( x) g ( x). ∵ 0, 2 p is compact then ∃ δ ′ 0 ∋ if

x, y ∈ 0, 2 p and x − y δ ′ , d G( x), G ( y) ε. Let δ min δ ′, p . Now fix any a, b ∈ R and


suppose that a − b δ . Then ∃ x, y ∈ 0,2 p and k ∈ z ∋ x kp a and y kp b.
∵ g is periodic then g ( x) g (a) g ( y) g (b). ∵ x − y a −b δ ≤ δ ′ then we have that
d g (a), g (b) d G( x), G( y ) ε. Hence g is uniformly continuous

⎪⎧⎪0 if x ∈ [0,1)
153. (c) hn ( x) converges to the function h( x) ⎨
⎪⎪⎩1 if x 0 pointwise. To prove this convergence is not
uniform. Since if the convergence were uniform then h would be continuous. This is not true. Notice that
g n ( x) converges to the function g ( x) 0 pointwise. If a ∈ 0,1 then nlim na n 0. The convergence is
→∞

not uniform. Let xn 1/(n 1) and notice that g n ( xn ) ( n /( n 1)) n 1 . Hence,

g n ( xn ) (1−1/( n 1)) n 1
→ 1/ e. Hence, lim sup g n − g c [0,1]
≥ 1/ e, where

. c 0,1 is the sup norm. Since convergence in the sup norm is the same as uniform convergence, then

g n does not converge to g.

154. Let M be ∋ g ′( x ) M for all x. We claim that g ( x) − g ( y) ≤ M x − y for all x, y ∈ R. If it is true then

g would be uniformly continuous (since given ε 0, the corresponding δ would be ε M ) Suppose


that ∃ x y ∋ g(x) −g( y) M( y −x). Then by the Mean value theorem ∃ z ∈ (x, y)

g ( y) − g ( x)
∋ g ′( z) . It follows that the g ′( z ) M . This is a contradiction. Similarly
y−x

g ( y ) − g ( x) −M ( y − x). Hence g ( x) − g ( y ) ≤ M x − y for all x, y. ∴ g is uniformly continuous.

58
Analysis BE.11

x2 x2 1
160. (a) For any x ∈ −1,1 , e
n
−1 e n
−1 ≤ e n −1 → 0 as n → ∞. Given ε 0. ∃ N ∈ ∋

1 x2 1
n ≥ N ⇒ e n −1 ε . Hence n ≥ N ⇒ e n
− 1 ≤ e n −1 ε. For all x ∈ −1,1 . Since each x 2 n
e
1 1 1
x2 x2

→∞ ∫
is integrable on −1,1 . nlim ∫ lim e ∫ 1.dx
is continuous on −1,1 . e x
2
n
e n
dx n
dx 2.
n→∞
−1 −1 −1

nx 2 3 3 − x4 3− x4 3 − 34 84
161. (a) For any x ∈ 1,3 , x3 nx − x 3
x nx
≤ 3 ≤ 3
x nx 1 n 1
→ 0 as n → ∞. Given any
n 1

84 nx 2 3 84
ε 0 ∃ n∈ ∋ n ≥ N ⇒ ε . Hence n ≥ N ⇒ −x ≤ ε for all x ∈ 1,3 .
n 1 x3 nx n 1

nx 2 3 nx 2 3 nx 2 3
∴ → x uniformly on 1,3 . Since each is continuous on 1,3 . ∴ is integrable on
x3 nx x 3 nx x3 nx
3
3
nx 2 3
3
nx 2 3
3
⎡ x2 ⎤
1,3 . nlim
→∞ ∫ x3 nx
dx ∫ nlim
→∞ x 3 nx
dx ∫ xdx ⎢ ⎥
⎢2⎥
4
−1 −1 1 ⎣ ⎦1

167. (b) Since h is continuous on a, b . By extreme value theorem, ∃ x0 ∈ a,b ∋ h( x) ≥ h( x0 ) 0 for all

x ∈ a, b . Let c h( x0 ) . Since hn → h as n → ∞ uniformly on

a, b , ∃ N1 ∈  ∋ n ≥ N ⇒ hn ( x) − h( x) c
2 for all x ∈ a, b . So

1
hn ( x ) ≥ h( x) − c ≥ h ( x0 ) − c c 0 . for all n ≥ N 1 and x ∈ a, b . Hence
2 2 2 hn is defined

c2
for large n. From the hypothesis, we know that for any ε 0, ∃ N 2 ∈  ∋ n ≥ N 2 ⇒ hn ( x ) − h( x ) ε
2
hn ( x) − h( x)
1 1 hn ( x) − h( x) ≤
Let N max N1 , N 2 . For any n ≥ N . − c2 ε.
hn ( x) h( x) hn ( x)h( x)
2

1 1 1
Hence h ( x) converges uniformly to h( x) as n → ∞ on a, b . Since g and are continuous on
n h
gn
a, b . By Extreme Value theorem, they are bounded. ⇒ →g
hn h uniformly on a, b .

168. (b) Let c max a , b . ∵ a, b ⊂(−1,1), 0 ≤c 1. x k ≤ c k for all k ∈  and all x ∈ a, b

59
BE.12 Analysis
∞ ∞
1 1
∵ ∑ ck ∞. By Weierstrass M-test ⇒ The uniformly convergence of ∑x k
on
k 0 1− c k 0 1− x

any closed interval a, b ⊂ (−1,1)

sin x sin y sin y sin x sin y


169. (d) For each x ≠ 0 , ≤ 2 . Thus lim lim 2 0 for each y≠0
x2 y2 x x→0 y→0 x y2

sin x sin y sin y sin x sin y sin x sin x 1 sin 2 x


≤ 2 . Thus lim lim 2 0 . Since f ( x, x) →1
x 2
y 2
y y → 0 x → 0 x y2 x2 x2 2 x2 2

sin x sin x 1 sin 2 x


as x → 0 , f (−x, x) − → − 1 as x → 0 . ∴ The limit does not exist.
x2 x2 2 x2 2

x2 y4 x2 x2 y 4
170. (d) For each x ≠ 0 , 2 → 1 as y → 0 . Thus lim lim 1 .For each y ≠ 0
x 2 y4 x2 x→0 y→0 x2 2 y 4

x2 y4 y4 1 x2 y4 1 .
→ lim lim
x2 2 y4 2 y4 2 as x → 0 . Thus y → 0 x→ 0 x 2 2 y 4 2 Hence the limit does not exist.

g (h, 0) − g (0, 0) h h
171. (c) For any h ≠ 0 sgn( h)
h sin h sin h
∵ The above limit does not exist as h → 0 g x (0, 0) does not exist. Hence g is not differentiable at (0, 0).

g (h, 0) − g (0, 0) 0
177.(a) Compute the partial derivatives at (0, 0) . g x (0, 0) lim = lim 0
h→ 0 h h → 0 h
and similarly g y (0, 0) 0. Thus, both first partials exist at (0, 0). To prove g is differentiable at (0, 0) .
a (0, 0) and B ∇g (a) ,
α
⎛ h 2 k 2 ⎞⎟
α ⎜⎜ ⎟ log(h 2 k2)
g (h, k ) − g (0, 0) −∇g (0, 0).(h, k ) hk log( h 2 k2) ⎜⎝ 2 ⎠⎟⎟
(h, k ) h2 k2

h2 k2

1 2 α− 1
h k2 2
log h 2 k 2 → 0 as (h, k ) → (0, 0) . That is f is differentiable at (0, 0)

g (h, 0) − g (0, 0) 0
178. (b) Compute the partial derivatives at (0, 0) . g x (0, 0) lim lim 0
h→ 0 h h→ 0 h

and similarly g x (0, 0) 0. Thus both partials exist at (0, 0).

g ( h, k ) − g (0, 0) −∇g (0, 0).( h, k ) hk


( h, k ) . ∵ The above limit does not exist as (h, k ) → (0, 0), g is not
h2 k2
differentiable at (0, 0)

60
Analysis BE.13

180. (b) By hypothesis on g, ∃ M 0 satisfying g ( x) ≤ M for all x ∈ A. Given ε 0 ∃

δ 0∋ x −c δ ⇒ f (x) −0 ε x−c δ ⇒ f ( x) g ( x) − 0 (ε / M ) M ε.
M . Thus
⎪⎧⎪ 1 ⎪⎫⎪
xn ⎪⎨ ⎪⎬ 1
181. (b) Let ε 1and define sequences in (0,1] ⎪⎪ n 2 1 ⎪⎪ and yn n
. lim xn − yn 0 but
⎩⎪ ⎪⎭
g ( xn ) − g ( yn ) n2 − n 1 n(n −1) 1 ≥ 1 ε for all n ∈ . So by the sequential criterion for

non-uniform continuity, g ( x) 1 is not uniformly continuous on (0,1].


x

182. (a) Suppose that g (c) ≠ 0 for some c ∈ 0, a . If g ( x) 0 apply Extreme Value theorem to x ∈ 0, a

⇒ that g attains its maximum value at some d ∈ 0, a . g (d ) 0, g ′(d ) 0 and g ′′(d ) > 0 .
We have ( g ′′ g ′ − g )( d ) 0 which contradicts. g ′′ g′− g 0. In conclusion g ( x) 0 for all
x ∈ 0, a

183. (b) Suppose g has two fixed points x y on an interval I where g ′ ( x ) ≠ 1. ∵ g is differentiable on I.

g ( x) − g ( y )
Apply the Mean value theorem to g on x, y to conclude that g ′(c) for some
x− y

c ∈ ( x, y ). However g ( x) x and g ( y ) y thus we have g ′(c) 1. Since this is a contradiction.


∴ f has no more than one fixed point.


1 1
184. (a) By Weierstrass M-test f n ( x) ≤
2n
for all x ∈ R and ∑2
n 1
n converges to 1. Thus g ( x) is uniformly

convergent on R.

188. (a) Clearly xn 0 assume that xn 1 b


2
x n2 1 b xn

b xn
≤1 b b
1 2 b b
2
1 b

Showing that xn 1 1 b.

∵ x1 1 b , it follows from the principle of mathematical induction that xn 1 b for all n.


Again using principle of mathematical induction and show that xn 1 xn for all n.

61
BE.14 Analysis

Clearly x2 b x1 b x1
Assuming that xn 1 xn

xn 2 − xn 1 b xn 1 − b xn

b xn 1 − a xn . a xn 1 − a xn
a xn 1 a xn

b xn 1 − b xn
b xn 1 b xn

xn 1 − xn
0
b xn 1 b xn

Hence, xn is increasing.

ε
189. (a) Let ε 0. There is N ∈ R ∋ if m, n N and x ∈ a, b then gm ( x) − gn ( x) (b − a) so if
x x x x

m, n N and x ∈ a, b then hm ( x) − hn ( x) ∫ gm − ∫ gn ∫ gm − gn ≤ ∫ gm − gn
a a a a

b b
ε ε
≤ ∫ gm − gn ≤ ∫ (b − a ) ε. So hn converges uniformly on a, b
a a
b−a b−a

190. (a) If 0 ≤ x 1 then nlim g n ( x) 1, lim g n (1) 1 x then nlim g n ( x) 0.


→∞ n →∞ 2 and if 1 →∞

⎧⎪1 0 ≤ x 1
⎪⎪
⎪⎨1/2 x 1
∴ g n converges pointwise to g ( x) ⎪⎪ . But each g n continuous at 1 and g is not, so
⎪⎩⎪0 1 x

g n does not converge uniformly..

191. (a) If B(a; ε) ∩ A a then in any sequence of distinct points of A, all but at most one member must be
at distance ≥ ε from a. Thus no such sequence can converge to A. Conversely, if every ball around
a meets A in a point other than a we can construct a sequence an of distinct points of A tending to a
by the induction argument. Hence the proof.

192. (c) g : R → R be differentiable ∋ g ′( x) ≤ M for all x ∈ R. For any ε 0 let δ ε .


M Then by the
g ( y) − g ( x)
Mean value theorem if y − x δ we have for some c between x and y g ′(c ) ≤ M
y−x
⇒ g ( y ) − g ( x) ≤ y − x M δM ε . ∵ δ didn’t depend on x or y. ∴g is uniformly continuous.

62
Analysis BE.15

198. (c) To show that x : g ( x) h( x) . Coniser the function f ( x) g ( x) − h( x). Then

x g ( x) h( x ) x f ( x) 0 f −1 (0) . This set is measurable because the function f is measurable.

To show that x : g ( x) ≥ h( x) consider the same definition of the function f ( x) . This function f(x)

is measurable; moreover x g ( x) ≥ h( x) x f ( x) ≥ 0 f −1 [0, ∞) . ∴ This set is measurable as

well as x : g ( x) h( x) following similar argument.

199. (a) Considering Fatou’s Lemma ∫ g ≤ lim inf ∫ g


i≥n i
n . Appling again the Fatou’s Lemma to the function

g − gn ≥ 0 , ∫ g−g ≤ lim inf ∫ g − g n . Then −∫ g ≤ liminf ⎡⎢−∫ g n ⎤⎥ , ∫ g ≥ lim inf ∫ g


n
⎣ ⎦ n

∴ lim inf ∫ g n ∫ g.
200. (b) It is always that inf S ≤ sup S for any bounded nonempty subset of R.
To prove inf T ≤ inf S and sup S ≤ sup T . Let x ∈ S . Then x ∈ T since S ⊂ T . So inf T ≤ x by
definition of inf T . ⇒ inf T is lower bound for S because x was taken arbitrary in S. Since inf S is the
greatest lower bound we get inf T ≤ inf S . Similarly sup S ≤ sup T .

201. (b) Assume that xlim f ( x)  exists. To show that f ( x)  for any x ∈ R. Let T 0 be period of
→∞

f (a) − 
f ( x). Let a ∈ R and assume that f (a) ≠  . Take ε ∵ lim f ( x)  then ∃ M 0 ∋ for
2 x →∞

any x M . We have f ( x ) −  ε. Since R is Archimedean, there exists n ∈ N ∋ n M −a /T


Then we have a nT M . Hence f (a nT ) −  ε. ∵ f (a nT ) f (a)
f (a) − 
we get f (a ) −  ε . Which is a contradiction, ∴ f (a ) . ∵ a was arbitrary..
2

202. (d) f ( x ) − f (c ) x− c ≤ x−c for x, c ∈ [0, ∞). ∴ given ε 0 choose δ ε2 then


x−c δ ⇒1 x − c ε. ∴ f ( x) x is uniformly continuous on [ 0, ∞).

209. (a) Consider the function h( x) g ( x) − f ( x). Fix ε 0. Then the set A x ∈ [a, b]; h( x) ≠ 0 is finite.

Assume that A x1 , x2 ,..., xn with a x1 x2 ... xn b , set M max h ( xi ) : i 1, 2,...n

We have M 0 choose δ 0 small enough to have a x1 − δ; xi δ xi 1 − δ , xn δ b and


b b b b

∫ g ( x)dx ∫ f ( x)dx ∫ h( x) dx ∫ f ( x) dx
a a a a

210. (c)It is clear that G ( x) is continuous and differentiable except at 1. (i.e) F 1 (1) does not exist.

63
BE.16 Analysis

⎛ K K 1 ⎞⎟
⎜⎜ ⎟
⎜⎝ ( K 1) K ⎠⎟⎟

K 1 InK − KIn( K 1)


211. (a) Using the properties of logarithms ∑ In
K 1 K K 1 K 1 K K 1

n
InK In( K 1) ⎛ In1 In1⎞⎟ ⎛ ⎞ ⎛ ⎞ In(n 1)
∴ Sn ∑ − ⎜⎜ − ⎟ ⎜⎜ In2 − In3 ⎟⎟ ... ⎜⎜ Inn − In(n 1) ⎟⎟ −
K 1 K K 1 ⎜⎝ 1 2 ⎠⎟ ⎜⎝ 2 3 ⎠⎟ ⎜⎝ n ( n 1) ⎠⎟⎟ (n 1)

⎛ In(n 1) ⎞⎟
⇒ lim S n lim ⎜⎜⎜− ⎟ 0.
n →∞ n →∞ ⎝ ( n 1) ⎠⎟⎟

nenx − nenx nx
∵ f n′ ( x) 0
217. (b) Let us find the maximum f n in [0, 2] enx
2 ; e nx (n − n 2 ) 0 ; n2 x n

⎛1⎞ 1
x 1 . 1 is the maximum with f n ⎜⎜ ⎟⎟ . Since nlim sup f n ( x) − 0 : x ∈ [0, 2] lim f n 1
n n ⎝ n ⎠⎟
⎜ e →∞ n →∞ n

1 . The convergence is not uniform on [0, 2]


e
n ∞

218. (a) Let Sn ( x) ∑g


k 0
k ( x) be the nth partial sum. ∵ ∑ M n converges for all ε
n 0
0 then there exists N ∋
if n ≥ m ≥ N , then M m Mm 1 ... Mn ε . Thus if n ≥ m ≥ N ,
( x)
S n ( x) − Sm ( x) f m 1 ( x) ... f n ( x) ≤ f m 1 ... f n ( x) ≤ M m 1 ... Mn ε for all
n

x ∈ D . ⇒ Sn converges uniformly on D. Hence ∑g


k 0
n also converges uniformly on D.

cos 2 ( nx)
219. (b) Let M n 1 ≤ Mn
n 2 for n ≥ 1 then n2

1
∵ cos(nx) ≤ 1. Because ∑n
n 1
2 is a p-series with p 2 1, it converges. Hence by the Weierstrass

M-test, convergence is uniform.

64
OBJECTIVETYPE
OBJECTIVE TYPEQUESTIONS
QUESTIONS(PART-B)
(PART-C)
1. Which of the following are true? (b) Every bounded closed subset of a metric space
(a) It is possible to place a linear order relation on is compact
the complex numbers C which
(c) Suppose {sn } & {tn } are sequences of complex
make (C , +, X , ≤) an ordered field.
(b)It is possible to place an linear order relation numbers ∋ nlim
→∞
sn = s and ∋ lim tn = t.
n →∞

≤ on the complex numbers C such that Then nlim sn tn = st .


→∞
(C , +, X , ≤) is not an ordered field
(d) There exists a bounded sequence of real
(c) Both (a) and (b) are true numbers with no convergent subsequence.
(d)Both (a) and (b) are false
6. Which of the following is true?
2. Which of the following statements is true? (a) Every complete space is compact
(a) Every bounded sequence of real numbers has (b) Every compact space is complete.
at least one subsequential limit. (c) both (a) and (b) are true
(b) The integral of the limit is equal to the limit of (d) both (a) and (b) are false
the integrals.
7. Which of the following are valid intervals of
(c) For all sequences of real numbers {Sn } we convergence for a power series
have lim inf S n ≤ lim sup S n (a) (−∞, 0] (b) (−∞, ∞)
(d) If the radius of convergence of a power series (c) [2, 2] ∪ [3,3] (d) [-1,3)
Σak x is R and if 0 < R < ∞, then the series
k
8. Which of the following statements is / are true?
Σak x converges uniformly on (− R, R) ∑a
k
(a) Suppose i diverges and Σai = 2. A
3. Which of the following series is convergent rearrangement aik of the terms Σaik = 4.
n (b) Suppose f : X → Y is continuous and X is
(a) ∑ (−1)
n

100n + 1000 compact then f must be uniformly continuous.


n (c) Metric spaces X and Y with X closed and
⎛ 7n ⎞
(b) ∑ ⎜⎝ 8n + 1 ⎟⎠ bounded and a continuous mapping
f : X → Y such that f ( x) is not closed and
1 bounded.
(c) ∑ n log n
(d) If f ( x), g ( x) : R → R are everywhere

( −1) n differentiable then f ( g ( x) ) must be


(d) ∑ n everywhere differentiable.
4. Which of the following is not always true for 9. Which of the following statements is / are false?
x, y , z ∈ R k ? (a) Suppose f :[a, b] is a function and suppose

(a) x. y ≤ x y (b) x = 0 iff x = 0 f is a local maximum. Then f ′( x) exist and


equal to 0.
(c) x ≥ 0 (d) x + y = x + y
(b) Suppose f ∈ R(α ).
5. Which of the following statements is / are true?
b b
(a) A metric space X with distinct open sets
Then ∫ fdα = ∫ f ( x)α ′( x)dx
Ui : i ∈ N ∋ ∩U i is open
a a

i∈N (c) Suppose { f n } is a sequence of continuous

65
OC.2 Analysis
13. Which of the following series is converges
functions on [a, b] such that ( V x ∈ [a, b])

∑(2 + n ) x


(10 x) n
lim f n ( x) = f ( x). Then f must be continuous.
n →∞
(a) ∑n
n=0
2
+1
(b)
n =0
n

(d) Suppose f ( x), g ( x) are real differentiable ∞ ∞


1
f ′( x) (c) ∑ nx n (d) ∑ n( x − 2) n

functions on (a, b) and ∋ lim =A n =1 n=0


x → c g ′( x )
14. The series
f ( x) sin(nt )
then lim
x →0 g ( x)
=A i) ∑n n2

∑ (−1)
2
10. Which of the following statements is / are ture? n
2− n
ii)
(a) Every continuous function f : (0,1) → (0,1) n

(a) (i) is converges absolutely and (ii) is diverges


must have a fixed point.
(b) (ii) is converges absolutely and (i) is diverges
(b) If A is connected then the closure of A is
(c) both (i) and (ii) are converges absolutely
connected.
(d) both (i) and (ii) are diverges
(c) If A ⊆ R is open and B = A closure of A,
15. Which of the following series is convergent
then interior ( B) = A ∞
1
(d) There is a non constant continuous maps from i) S =∑
n =1 f (n) where f (n) is positive and
R → Q.
≤ cn for some constant c > 0
11. Let S ′ = {( x, y ) ∈ R : x + y = 1} .
2 2 2

1
Given p, q ∈ S ′ define d s ( p, q) to the shortest ii) S = ∑ g (n) n where g (n) is increasing and
n =1
distance along S ′ from p to q. For example if unbounded
(a) (i) is converge and (ii) does not converge
p = (1, 0) and q = (0,1) then d s ( p, q ) = π 2 .
(b) (ii) is converge and (i) does not converge
(Note d s ( p, q ) is also the angle between the line (c) both (i) and (ii) are converge
(d) both (i) and (ii) are diverge
connecting (0, 0) and p and the line connecting
16. Which of the following statement is / are true?
(0, 0) and q). Then which of the following (a) Let S and T be non empty bounded subsets
statements is / are true? of R. If S ⊆ T then
(a) ( S ′, d s ) is a metric inf T ≤ inf S ≤ sup S ≤ sup T .
(b) S ′ is compat (b) sup( S ∪ T ) = max{sup S ,sup T }
(c) S ′ is connected (c) Let I be the set of real numbers that are not
(d) Every continuous function f : S ′ → R has rational.If a < b then ∃ x ∈ I ∋ a < x < b.
a maximum value. (d) An ordered field F is Archimedean iff satisfies
for every a ∈ F there exists n ∈ N when
12. Find the coefficient of ak for k = 0,1, 2,... such
n > a.

x2 + 1
that ∑a x
k =0
k
k
=
x −1
17. For points x, y ∈ R k , let
d1 ( x, y ) = max{ x j − y j : j = 1, 2,..., k} and
(a) -2 (b) 2
(c) -1 (d) 5 k
d 2 ( x, y ) = ∑ x j − y j
j =1

66
Analysis OC.3

i) d1 and d 2 are matrices for R k


⎧1 if x = 0
ii) d1 and d 2 are complete. ⎪1
(a) both (i) and (ii) are true ⎪ n if x = m n for some integers

(b) both (i) and (ii) are false (a) f ( x) = ⎨m, n with n ≠ 0 and reduced to
(c) (i) is true but (ii) is false ⎪lowest terms
(d) (ii) is true but (i) is false. ⎪
⎪0 if x ∉ Q
xn ⎩
18. The series ∑1+ x n
is continous only for x ∉ Q
(a) converges for x ∈ [0,1) (b) If f is continuous on [a, b] with f ( x) > 0 for
(b) converges uniformly on [0, a ) for each
all x ∈ [a, b] then 1 f is bounded on [ a, b]
a, 0 < a < 1.
(c) converges uniformly on [0,1) 1
(c) f ( x) = is not uniformly continuous on
(d) None of the above x
19. Suppose (0,1]
(d) None of the above
⎪⎧ ⎪⎫
1
K = ⎨ f ∈ c ([0,1]) : Lip( f ) ≤ 1 & ∫ f ( x) dx = 0 ⎬
⎩⎪ ⎭⎪ n
23. Define { f n ( x)} by f n ( x) =
0
for x ≠ 0
then 1 + ( nx ) 2

(a) K is equicontinuous in c ([0,1]) and f n (0) = 0 . Fixing n ∈ . Compute

(b) K is closed in c ([0,1]) ∞

∫ f n ( x)dx. What happens to this integral as


(c) K is bounded in c ([0,1]) −∞

n → ∞?
(d) K is compact in c ([0,1])
(a) π (b) 1 2
20. Which of the following statements is/are true
(a) given any two real numbers a < b ∃ an (c) 0 (d) ∞

irrational number t ∋ a < t < b 24. Compute xn = inf {an , an +1 ,...} and

(b) A = sup{r ∈ Q r < x} = x yn = sup {an , an +1 ,...} and lim inf an and

(c) Let a & b real numbers. If a ≤ b + 1 n for lim sup an where an = (−1) 1 + 1 n
n
( )
every n ∈ N then a ≤ b (a) lim inf an = −1 & lim sup an = 1
(d) None of the above
(b) lim inf an = 0 & lim sup an = 1
21. Which of the following series is convergent series
(c) lim inf an = −1 & lim sup an = 0
( )


5n
(a) ∑ n (b) ∑ In 1 + 1 n (d) lim inf an = lim sup an = 0
n =1 3 + 4
n
n =1

∞ ∞
25. i) A contraction mapping on a metric space is
sin n 1
(c) ∑ 2 (d) ∑ n(Inn) p if p > 1 continuous
n =1 n n=2 ii) Every closed subset of a compct metric space
22. Which of the following statements is / are true? is compact.

67
OC.4 Analysis
(a) (i) is true but (ii) is not true (b) inf = −1 and sup = 1
(b) (ii) is true but (i) is not true
(c) both (i) and (ii) are true (c) inf = −1 and sup = 0
(d) both (i) and (ii) are not true (d) inf = 0 and sup = 1
26. Find the radius of convergence of the
31. Which of the following sets is / are countable?
(−1)n x n
series ∑ (a) The set of functions from {0,1} to N
n +1 (b) The set of finite subsets of N
(a) 1 (b) 0 (c) The set consisting of all sequences of 0s and1s
(c) ∞ (d) -1 (d) The set of functions from N to {0,1}
27. Find the power series representation for
32. Which of the following statements is / are true?
1 (a) A cauchy sequence must be monotone
f ( x) =
(1 + x) 2 (b) Every monotone sequence is cauchy
(c) An unbounded sequence can contain a
∞ ∞
cauchy subsequence
(a) ∑ (−1)
n =1
n
xn (b) ∑ (−1)
n =1
n +1
nx 2 n
(d) A cauchy sequence can have a divergent
subsequence.
∞ ∞
nx 2 n
(c) ∑ (−1) nx (d) ∑
n +1 n −1
33. For each x ∈ R, determine whether the sequence
n =1 n =1 2

⎛ 1 ⎞
t ⎜ n ⎟ has a limit.
28. What is the radius of convergence of ∫ 1− t 8
dt ⎝ 1 + x ⎠n =1

(a) R = 1 (b) R = 0 (a) x = 0,lim = 1 (b) x < 1, lim = 1


(c) R = ∞ (d) R = −1 (c) x = −1, no limit (d) x = 0, lim = 0
29. i) Suppose ( sn ) and (tn ) are bounded 34. Match the following accumulation point
sequences. Then i) The interval [0,1] - The set of all real numbers
lim sup( sn + tn ) ≤ lim sup sn + lim sup tn ii) The set of natural- [0,1]
ii) Suppose ( sn ) and (tn ) are bounded numbers
sequences. Then iii) The set of all - Empty set
irrational numbers
lim sup( sn tn ) ≤ lim sup sn lim sup tn (a) (i) → (iii), (ii) → (i), (iii) → (ii)
iii) Suppose ( sn ) is a bounded but not convergent (b) (i) → (i), (ii) → (iii), (iii) → (ii)
(c) (i) → (iii), (ii) → (ii), (iii) → (i)
sequence and sn > 0. If (d) (i) → (ii), (ii) → (iii), (iii) → (i)
tn → t ≠ 0. Then ( sn tn ) is convergent.
35. Suppose that the random variables X n are
(a) (i) and (ii) are true but (iii) is not true.
defined on the same probability space and there
(b) (i) and (iii) are true but (ii) is not true.
(c) both (i), (ii) and (iii) are true. is a constant c ∋ X n converges in distribution
(d) (ii) is true but (i) and (ii) are not true. to the random variable c.
30. Find the infimum and supremum of the set i) X n converges to c in probability
⎧ n ⎫ ii) X n converges to c a.s
⎨ , with m, n ∈  ⎬
⎩m + n ⎭ (a) both (i) and (ii) are true
(b) (i) is true but (ii) is not true
(a) inf = 0 and sup = 1 (c) (ii) is true but (i) is not true
(d) both (i) and (ii) are not true

68
Analysis OC.5

36. Suppose f is continuous on [ a, b] and a σ - algebra.


(a) (i) is true but (ii) is not true
f (a) < 0 < f (b) (b) (ii) is true but (i) is not true
(c) both (i) and (ii) are true
⎛ ( a + b) ⎞
i) Either f ⎜ ⎟ = 0 or f has different signs (d) both (i) and (ii) are not true
⎝ 2 ⎠
42. Which of the following series is convergence
⎡ ( a + b) ⎤ 1 1
at the end points ⎢ a, or f has differnt ∞
e n ∞
(−1) n −1 e n
⎣ 2 ⎥⎦ (a) ∑ 2 (b) ∑
n =1 n n =1 n
⎡ ( a + b) ⎤
, b⎥
∑ n sin ( 1 n )
∞ ∞
signs at the end points of ⎢
∑n
1
⎣ 2 ⎦ (c)
n
(d)
n =1 n =1
ii) There is a point x in (a, b) where f ( x) = 0
(a) (i) is true but (ii) is not true 43. Define a sequence {an } by letting a1 = 2 and
(b) (ii) is true but (i) is not true
1
(c) both (i) and (ii) are true an +1 = 4 −
(d) both (i) and (ii) are not true an for each n ≥ 1 , 2 ≤ an ≤ an +1 ≤ 4
37. Which of the following statements is / are true. for each n ≥ 1 and find the limit of this sequence.
(a) f ( x) = x is uniformly continuous (a) L = 2 + 3 (b) L = 2 − 3

(b) f ( x) = x 2 is uniformly continuous on [0,3] (c) L = 2 (d) L = 0


44. Consider the sequence of partial sums
(c) If f is uniformly continuous on a bounded set
S, then f is a bounded function on S. n
1
Sn = ∑
(d) None of the above k =1 k
2


n2 − 1 (a) If n, m ∈ z+ with m > n then
38. The series ∑ (−1)
n3 + 1
n

n =1 n
1
(a) converges conditionally Sm − Sn = ∑
k = n +1 k 2

(b) converges
(c) diverges 1 1
(d) none of the above (b) k 2 < k (k − 1) for k ≥ 2


5n3 + 2n + cos 2 (e n ) m
1 1 1
39. The series ∑ (c) ∑ = −
k (k − 1) n m
n =1 n +n +2
8 5
k = n +1

(a) converges
1 1
(b) converges conditionally (d) S m − S n < +
(c) diverges m n
(d) none of the above 45. Let f , g and h be defined on [0,1] as follows:
40. Find the interval of convergence of the power
f ( x) = g ( x) = h( x) = 0 whenever x is irrational

( x + 1) n
series ∑ f ( x) = 1 and g ( x) = x whenever x is rational
n =1 n ( n + 1)
m
(a) [−2, 2] (b) [−2, 0] h( x) = 1 if x is the rational number , h(0) = 1
n n
(c) [−1,0] (d) (−2, 2)
(a) f is continuous in [0,1]
41. i) Every σ - finite measure is saturated (b) g is continuous only at x = 0
ii) The collection τ of locally measurable set is

69
OC.6 Analysis
(c) h is continuous only at the irrational points in 50. Suppose −1 < α n < β n < 1, αn → 0 and
[0,1]
β n → 0 as n → ∞. Define the differ ence
(d) f is not continuous anywhere in [0,1]
f ( β n ) − f (α n )
46. Define f n :[0,1] → [0,1] by f n ( x) = x n (1 − x) quotients Dn =
βn − αn
(a) f n converges uniformly to 0 Dn = f ′(0)
(a) If an < 0 < β n and nlim
→∞
(b) f n does not converges
⎧βn ⎫
(c) f n converges to 1 (b) If 0 < α n < β n and ⎨ ( β − α ) ⎬ is bounded
⎩ n n ⎭
(d) none of the above
47. Which of the following statement is / are true? then nlim Dn = f ′(0)
→∞

(a) If { xn }n has an unbouned subsequence, then (c) If f ′ is continuous in ( −1,1) then

{ xn }n is unbounded lim Dn = f ′(0)


n →∞

(b) If { xn }n is unbounded then some subsequence (d) none of the above

is unbounded 51. Let f :[a, b] → R be a function of bounded


(c) If { xn }n is unbounded, then every variation on [a, b ] and define a function V on
subsequence is unbounded. [a, b] by V (a) = 0 and V ( x) = V ( f ,[a, x]) for
(d) If { xn }n diverges then its every subsequence
all x ∈ (a, b].
diverges.
(a) V is increasing on [ a, b]
48. Let ( an )n∈N and ( bn )n∈N be sequence of real
(b) If V is continuous at c ∈ [a, b]. Then f is
numbers. Then continuous at c.
(a) nlim inf ( an + bn ) ≥ lim inf an + lim inf bn (c) If f is continuous at c ∈ [a, b] then V is
→∞ n →∞ n →∞

continuous at c.
(b) nlim sup ( an + bn ) ≤ lim sup an + lim sup bn
→∞ n →∞ n →∞
(d) a ≤ x < y ≥ b, V ( y ) − V ( x) = V ( f ,[ x, y ]
(c) nlim inf ( an + bn ) ≤ lim inf an + lim inf bn
→∞ n →∞ n →∞ 52. Which of the following statements is / are ture?
(d) nlim sup ( an + bn ) ≥ lim sup an + lim sup bn (a) A function that is Riemann integrable on [a , b ]
→∞ n →∞ n →∞

49. Which of the following functions is / are must be bounded on [ a, b]


differentiable at 0. (b) A continuously differentiable function on a
compact interval is Lipschitz on that interval
⎧0, x ∈ Q (c) A Lipschitz function is differentiable
(a) f ( x ) = ⎨ ,
⎩x x ∉ Q (d) Every continuous function has an anti-
derivatie.
⎧0, x ∈ Q
(b) f ( x) = ⎨ 2 53. Which of the following statement is / are true?
⎩x , x ∉ Q (a) Any union of sets of measure zero have
measure zero
⎧⎪0, x = 0 (b) If f :[a, b] → R is integrable and
(c) f ( x) = ⎨ x sin 1 , x ≠ 0
⎪⎩ x ( ) x
f ( x) = ∫ f (t )dt then F ′( x) = f ( x)
1
(d) f ( x) = x 3 a

(c) Given any two partitions P and Q of [ a, b]

70
Analysis OC.7

and any function f :[a, b] → R we must ii) f ( x) = 1 x is not uniformly continuous on


have L( f , P) ≤ U ( f , Q )
(0,1] .
(d) A function f :[a, b] → R is continuous at (a) (i) and (ii) are false
c ∈ [a, b] iff the oscillation of f at c is zero. (b) (i) is true but (ii) is not true
(c) (ii) is true but (i) is not true
54. A function f :[0,1] → R (d) both (i) and (ii) are true
(a) If f is bounded then f ′ is bounded 59. i) The sequence { f n ( x)} defined by
(b) If f ′ is bounded then f is bounded 2

2
f n ( x) = xe − nx is uniformly convergent on
(c) If f is Riemann integrable on [0,1] then f is
[0,1]
Riemann integrable on [0,1] ∞
cos(2n x)
2
(d) If f is Riemann integrable on [0,1] then f is ii) ∑
n =1 2n
is continuous on R.

Riemann integrable on [0,1] . (a) (i) is true but (ii) is not true
(b) (ii) is true but (i) is not true
55. Let X be a compact metric space. Then (c) (i) and (ii) both are true
(a) X is seperable (d) both (i) and (ii) are not true
(b)X is second countable
(c) C(X) is separable 60. For what values of a and b the series
(d)None of these ⎛a 1 b ⎞
56. i) Every nonempty subset of [0,1] is Lebesgue
∑ ⎜⎝ n + n + 1 + n + 2 ⎟⎠ converges?

measurable
ii) Suppose H is separable Hilbert space if (a) a = − 1 2 , 3a + b + 2 = 0
T :H →H is a linear mapping (b) a = 1, b = 2
∋ I − T < 1, where I is the identity map of (c) a = 0, b = −1
H to itself then T is invertible. (d) a = 2, b = 3
(a) (i) is true but (ii) is not true
(b) (ii) is true but (i) is not true 61. Which of the following is / are metric?
(c) (i) and (ii) both are true (a) d ( x, y ) = x − y
2 2
x ∈ R1 and y ∈ R
1

(d) (i) and (ii) both are false


x− y
xn (b) d ( x, y ) = 1+ x − y
∑ 1 + xn converges for x ∈ [0,1) x ∈ R1 and y ∈ R
1
57. i) The series

(c) d ( x, y ) = ( x, y ) 2
xn
ii) The series ∑ converges uniformly on
1 + xn (d) d ( x, y) = x − 2 y
[0,1) 62. Let X be the set of all x ∈ [0,1] whose decimal
(a) (i) is true but (ii) is not true expansion contains only the digits 4 and 7. Then
(b) (ii) is true but (i) is not true (a) X is countable
(c) both(i) and (ii) are true
(d) both(i) and (ii) are not true (b) X is dense in [0,1]
(c) X is compact
58. i) If f is continuous on [a, b] with f ( x) > 0 for
(d) X is perfect.
1
all x ∈ [a, b] then f is bounded on [a, b]

71
OC.8 Analysis

63. Let A and B be separated subsets of some R k , f ( x) = 0 if x ≠ x0 . Then f ∈ R(α ) and


suppose a ∈ A, b ∈ B and define ∫ fdα = 0.
q(t ) = (1 − t )a + tb
ii) Suppose f ≥ 0, is continuous on [ a, b] and
for t ∈ R1 . Put A0 = q −1 ( A), B0 = q −1 ( B ). b

(a) A0 and B0 are separated subsets of R 1 ∫ f ( x)dx = 0 .Then f ( x) = 0 for all x ∈ [a, b].
a

(b) There exists t0 ∈ (0,1) ∋ q ( t0 ) ∉ A ∪ B (a) (i) is true but (ii) is not true
(b) (ii) is true but (i) is not
(c) Every convex subset of R k is connected
(c) both (i) and (ii) are true
(d) None of the above
(d) both (i) and (ii) are false
64. Define f and g on R 2 by 67. Which of the following sequences are bounded
⎛ nπ ⎞
2
xy (−1) n
f (0, 0) = g (0, 0) = 0, f ( x, y ) = (a) (b) sin ⎜ ⎟
x + y4
2
n2 ⎝ 7 ⎠

xy 2 n
g ( x, y ) = (c) (d) n5
(x 2
+y 6
) if ( x, y ) ≠ (0, 0) 3n
68. Which of the following statement is / are true?
(a) f is bounded on R 2
(b) g is unbounded in every neighborhood of (a) cos x − cos y ≤ x − y
(0, 0) (b) ex ≤ e x for all x ∈ R
(c) f is not continuous at (0, 0) (c) sin x ≤ x for all x ≥ 0
(d) restriction of both f and g to every straight (d) f ( x) = Inx is uniformly continuous on [1, ∞)
line in R 2 are continuous
69. i) Let f be integrable on [a, b] and suppose that
65. Suppose f is defined in (−1,1) and f ′(0) exists. g is a function on [ a, b ] ∋ g ( x ) = f ( x )
Suppose −1 < α n < β n < 1, α n → 0 and except for finitely many x ∈ [a, b]. Then g is
β n → 0 as n → ∞. Define the difference b b

f ( β n ) − f (α n ) integrable and ∫ f = ∫ g.
quotients Dn = βn − αn
a a

ii) If f is integrable on [a, b] then f is not


(a) If α n < 0 < β n then lim Dn ≠ f ′(0)
integrable on every interval [c, d ] ⊆ [a, b] .
⎧βn ⎫ (a) (i) is true but (ii) is not true
(b) If 0 < αn < βn and ⎨ ( β − α )⎬ is (b) (ii) is true but (i) is not true
⎩ n n ⎭
(c) both (i) and (ii) are true
bounded then lim Dn = f ′(0) (d) both (i) and (ii) are false
(c) If f′ is continuous in (−1,1) then x
70. Let f n ( x) = for x ≥ 0
lim Dn = f ′(0) x+n
(d) None of the above (a) f ( x) = lim f n ( x) = 0 for all x ≥ 0
66. i) Suppose α increases on [ a, b] , a ≤ x0 ≤ b, (b) If t > 0, the convergence is uniform on [0, t ]
α is continuous at x0 , f ( x0 ) = 1 and (c) The convergence is uniform on [0, ∞)
(d) None of the above

72
Analysis OC.9

71. Suppose f :R→R satisfies b


d ( x, y ) = ∫ x(t ) − y (t ) dt
f ( x + y ) = f ( x) + f ( y ) for each x, y ∈ R. Then a

(a) f (nx) = nf ( x) for all x ∈ R, n ∈ N


75. Consider the sequence { f n } defined by
(b) f is continuous at a single point iff f is
continuous on R nx
f n ( x) = for x ≥ 0. Then
(c) f is continuous iff f ( x) = mx for some m ∈ R 1 + nx
(d) None of the above (a) nlim f n ( x) = 1 , for x > 0.
→∞

72. Let T : ( C[0,1], . ) → ( C[0,1], . ) be a function (b) { fn } does not converge uniformly to f on
x
[0, ∞)
defined as Tf ( x) = ∫ f (t )dt where by C [ 0,1]
0 (c) for a > 0, { f n } converges uniformly to f on
mean the vector space of all continuous real valued
[ a, ∞ )
functions defined on [0,1] and f = 0sup f (t )
≤ t ≤1
(d) nlim f n ( x) = 0
→∞
then which of the following is true?
(a) T has a unique fixed point 76. A sequence of functions { gn } are defined as
(b) T is a contraction
(c) T is not a contraction x
g n ( x ) = 2 x + , x ∈ [ 0,1]
(d) T 2 is a contraction n
73. Which of the following integrals is / are (a) g = nlim
→∞
gn = 0
converges?
∞ ∞
sin x dx (b) g is continuous on [ 0,1]
(a) ∫
x
dx (b) ∫
1 1 + x3
1

∞ (c) ⎡ lim g n ( x) ⎤ = lim g n′ ( x) for x ∈ [ 0,1]
x ⎣ n →∞ ⎦ n →∞
(c) ∫ dx (d) None of these
0 1 + x sin x
2 2
1 1

74. Which of the following is / are metric space (d) ∫ nlim


→∞
g n ( x) dx = lim ∫ g n ( x)dx
n →∞
0 0
(a) X = R 2 for x = ( x1 , x2 ), y = ( y1 , y2 ) ∈ X
77. Suppose g is n times continuously differentiable
the function
on some interval (a, b) that contains c.
⎧ x1 − y1 if x2 = y2
d ( x, y ) = ⎨
⎩ x1 + x2 − y2 + y1 if x2 ≠ y2 If g ′(c) = g ′′(c ) = ... = g n −1 (c ) = 0
(b) Let X be the set of continuous functions from and g n (c) ≠ 0. Then
[a, b] to R . For all x, y ∈ X . (a) If n is odd then g has neither a relative minimum
d ( x, y ) = max { x(t ) − y(t ) : t ∈ [a, b]} nor a relative maximum at c.

(c) Let X be the set of bounded functions from (b) If n is even and g n (c) > 0, then g has a relative
some set A to R. For x, y ∈ X . minimum at c.
(c) If n is even g n (c) < 0 then g has a relative
d ( x, y ) = sup { x(t ) − y (t ) : t ∈ A}
maximum at c.
(d) Let X be the set of continuous functions from
(d) If n is odd g n (c) < 0 then g has a relative
[a, b] into R. For all x, y ∈ X .
maximum at c.

73
OC.10 Analysis
78. Which of the following statements is / are true? ∞

(a) If ( g ) is a sequence of functions, each one


n onto E, then ∑x = ∑x
x∈E n=0
n

bounded on a set and A ⊆ R and ( g )


n
(c) If E is countable and xn is a one-to-one
converges uniformly to g on A, then g is also ∞
bounded mapping of N onto E, then ∑x = ∑x
x∈E n =1
n
(b) A subset of R is closed iff, it contains all of its
boundary points.
(d) If E is countable then ∑ x > ∞.
( nxe )
− nx ∞ x∈E
(c) The sequence of functions n =1
82. Let f be a real valued function defined on [a, b].
converges uniformly on [0, ∞)
We define the lower envelope g of f to be the
( sin( x) ) = 0
x function g defined by
(d) xlim
→ 0+
g ( y ) = sup inf f ( x)
79. Which of the following limits does not exists δ > 0 x − y <δ

e x Iny and the upper envelope h by


(a) lim
( x , y ) → (1,1) x + 2 y 2
2
h( y ) = inf sup f ( x )
δ > 0 x − y <δ

2 2
x y-z Then which of the following is true?
(b) lim
( x , y , z ) → (0,0,0) x + y2 + z2
2
(a) If f is bounded, the function g is lower semi
continuous, while h is upper semi continuous.
x sin y (b) If φ is any lower semi continuous function
(c) lim
( x , y ) → (0,0) x2 + 2 y 2
∋ φ ( x) ≤ f ( x) for all x ∈ [a, b] then
x 2 sin 2 y φ ( x) ≤ g ( x) for all x ∈ [a, b]
lim
(d) ( x , y ) → (0,0) 2
x + 2 y2
(c) For each x ∈ [a, b], g ( x) ≤ f ( x) ≤ h( x) iff f
80. Let g n ( x ) = nx for x ∈ [ 0,1] and n ∈ . Then
n is lower semi continuous at x, wh ile
g ( x) = h( x) iff f is continuous at x.
(a) lim g n ( x) = 0 for x ∈ [0,1)
(d) If f is bounded the function g is lower semi
1
continuous while h is not continuous.
→∞ ∫
(b) nlim g n ( x )dx = 0
0
83. Let f be a non-negative measurable function.
Then
1

→∞ ∫ ∫f
(c) nlim g n ( x)dx = 1 (a) = 0 implies f = 0 almost everywhere.
0

(d) lim g n ( x) = 1 for x ∈ [ 0,1]


(b) ∫f = sup ∫ φ over all simple functions φ ≤ f

81. Let E be a set of positive real numbers. We define (c) There is an increasing sequence φn of non-
negative simple functions each of which
∑ x to be supF S
x∈E F∈
F , where
F is the collection of vanishes outside a set of finite measure such
finite subsets of E and S F is the (finite) sum of that f = lim φn
the elements of F. Then (d) ∫f = 0 implies f ≠ 0.

(a) ∑x <∞
x∈E
only if E is countable 84. Let f be defined on [a, b] and g be a continuous
(b) If E is countable and xn is onto maping N function on [α, β ] that is differentiable at γ with

74
Analysis OC.11

g (γ ) c ∈ [ a, b] xn xn
(a) If g ′(γ ) > 0 then D ( f  g )(γ ) = D f (c).g ′(γ )
+ + (b) rn
2 rn − rn 1 and ∑ rn
(b) If g ′(γ ) < 0 then D + ( f  g )(γ ) = D− f (c).g ′(γ ) converges
(c) If g ′(γ ) = 0 and all derivatives of f are finite xn
(c) ∑ r is converges
at c then D + ( f  g )(γ ) = 0 n

xn
(d) If g ′(γ ) < 0 then D + ( f  g )(γ ) = D + f (c).g (γ )
(d) ∑ r is diverges
n
85. Let E be a subset of a complete metric space
(a) If E is dense and F a closed set contained in 88. Fix a positive number α. Choose b1 α and
E, then F is nowhere dense
(b) If E and E are both dense, then at most one of define b2 , b3 , b4 ,... by recursion formula

them is an Fσ 1 ⎛⎜ α ⎞⎟
bn ⎜bn ⎟⎟
2 ⎜⎜⎝ bn ⎠⎟ . Then
1
(c) The set of rational numbers in [0,1] is not a
Gδ (a) bn decreases monotonically and that limit
(d) If E and E are not dense, then F is nowhere
dense bn α

86. Let A1 , A2 , A3 ,... be subsets of a metric space. ε n2 ε n2


(b) If εn bn − α then ε n +1 = < so
Then 2bn 2 α
n n

(a) If Bn ∪A i then Bn ∪A i for ⎛ ε1 ⎞


2n
i 1 i 1 that setting β 2 α then ε n +1 < β ⎜ ⎟
n 1, 2, 3,... ⎝β⎠


∞ (n 1, 2,3,...)
(b) If B ∩ Ai then B ⊃ ∪ Ai
i 1 i 1 ε1 1
(c) If α 3 and b1 2 then < and

∞ β 10
(c) If B ∪ Ai then B ⊃ ∪ Ai
i 1 i 1 ∴ ε 5 < 4.10−16 , ε 6 < 4.10−32
n n
(d) bn increases monotonically and that
(d) If Bn ∩A
i 1
i then Bn ∪A
i 1
i for
lim bn 1
n 1, 2, 3,... α

87. Suppose xn 0 and ∑ xn converges. Put 89. If xn is a complex sequence, define its

arithmetic means σn by
rn ∑x
m n
m then
s0 s1 .... sn
σn (n 0,1, 2,...)
xm xn r n 1
(a) r ... 1− n
m rn rm Then which of the following imply true?
xn (a) If lim sn s then lim σn s
if m n then ∑r diverges
(b) If xn sn − sn−1 for n ≥ 1. Then
n

75
OC.12 Analysis

1 n Suppose −1 αn βn 1, αn → 0 and
sn − σn
n 1
∑ kx k 1
k
βn → 0 as n → ∞ .Define the differ ence
And assume that if lim(nxn ) 0 and that g β n − g αn
quotients Dn . Then
σn converges. Then sn is converges. β n − αn

(c) If sn 0 for all n then lim sup sn 0 and (a) If αn 0 βn then lim Dn g 1 (0)
lim σn ∞ ⎪⎧⎪ βn ⎪⎫⎪
(d) If M ∞ nxn ≤ M for all n and (b) If 0 αn βn and ⎨⎪ β − α ⎬ is bounded
⎪⎪
⎩⎪ n n ⎭
lim σn σ. Then lim sn 0
then lim Dn g 1 (0)
90. Suppose xn 0, sn x1 x2 ... xn an d (c) If g 1 is continuous in (−1,1) then
∑ xn diverges. Then
lim Dn g 1 (0)
xn (d) If αn 0 βn then lim Dn g 1 (0)
(a) ∑ 1 xn diverges
93. Suppose a and c are real numbers, c 0 and g is
xN 1 xN k sN defined on [−1,1] by
(b) s ... ≥ 1−
N 1 sN k sN k
⎧⎪ x a sin( x−c ) if x ≠ 0
xn g ( x) ⎪⎨
and that ∑ s diverges. ⎪⎪⎩0 if x 0
n
Then
xn 1 1 xn (a) g is continuous iff a
(c) s 2 ≤ s − s and that ∑ s 2 converges.
0
n n−1 n n (b) g ′(0) exists iff a 1
xn (c) g is not continuous iff a 0
(d) ∑1 n 2 xn diverges (d) g ′(0) does not exists.
91. If E is a non empty subset of a metric space X, 94. Fix a 1
define the distance from x ∈ X to E by (a) If m, n, p, q are integers; n 0, q 0 and
ρE x inf d ( x, z ) m p 1 1
r m
ap
q then a
z ∈E n q
n
Then
(b) If x is real, define B ( x ) to be the set of all
(a) ρE x 0 iff x ∈ E
numbers a t , where t is rational and t ≤ x,
(b) If ρE x is a uniformly continuous function
then a r sup B ( r ) where r is rational.
on X then ρ E x − ρ E y ≤ d ( x, y ) for all (c) a x y
ax a y for all real x and y
x ∈ X , y ∈ X. (d) a x y
a x a y for all real x and y
(c) ρE x 0 iff x ∉ E 95. Let g be a continuous function from [0,1] to R
(d) ρE x is not a uniformly continuous on X. 1 n ⎛⎜ k ⎞⎟
and Sn ∑ g⎜ ⎟
n k 1 ⎜⎝ n ⎠⎟
n 1, 2, 3, 4,...
92. Suppose g is defined in (−1,1) and g 1 (0) exists.
Then

76
Analysis OC.13

(a) S n

is a divergent sequence (d) If f is differentiable and f ′(t ) ≠ 1 for every
n 1


real t then f has two fixed point.
(b) S n n 1
is a convergent sequence
x 1

(c) Suppose g ( x) ≥ 0 for all x ∈ [0,1] and ∫ sin(t


2
100. Define g ( x) )dt. Then
x
g ( x0 ) ≥ 0 for some x0 ∈ [0,1]. Also g is
(a) g ( x) 1
continuous on [0,1]. Then nlim
→∞
Sn 0. x if x 0

(d) If g is continuous on [0,1] then g is not (b) 2 xg ( x) cos( x 2 ) − cos( x 1) 2 r ( x)


uniformly continuous on [0,1] . where r ( x) c
x and c is a constant

1 1
96. If 0 ≤ x 1 then ∑x
n 0
n

1− x
suppose if (c) g ( x)
x if x 0

x ≥ 1, the series
(d) 2 xg ( x) cos( x 2 ) − cos( x 1) 2
(a) diverges 101. Which of the following sets are bounded?
(b) converges
(c) uniformly converges ⎧ 1 1 1 ⎫
(a) A = ⎨1, , ....., ,....⎬
(d) absolutely converges ⎩ 2 3 n ⎭
97. Let E 0 denote the set of all interior points of a set (b) C = {..... − 5, −3, −1}
E. Then
(c) B = {2, 4, 6,.....}
(a) E 0 is always open
(b) E is open iff E 0 E (d) z = {..., −2, −1, 0,1, 2,....}
(c) If G is contained in E and G is open, then G is
102. Which of the following is/are true?
contained in E 0 (a) An arbitrary union of open sets is open
(d) E and E always have the same interiors. (b) A finite union of closed sets is closed
(c) An arbitrary intersection of closed set is
98. Let E be the set of all x ∈ [0,1] whose decimal closed
expansion contains only the digit 4 and 7. (d) A finite union of open sets is open
(a) E is countable
103. A set is said to be nowhere dense if
(b) E is dence in [0,1]
( )
0
(c) E is compact (a) S =φ (b) S = S ′
(d) E is perfect
(c) S ⊂ S ′ (d) S ∈ S ′
99. Suppose f is a real function on (−∞, ∞). 104. If C is cantour set then
(a) If f is differentiable and f ′(t ) ≠ 1 for every (a) C is closed
real t then f has atmost one fixed point. (b) C is nowhere dense
(c) The Lebesgue measure of the set C is zero
(b) The function f defined by f (t) t (1 et )−1 (d) C is countable
has no fixed point, although 0 f ′(t ) 1 for 105. 1) Every open cover of a set in R has a count
all real t able subcover
(c) The function f defined by 2) Every bounded infinite set of R has a limit
point
f (t ) t (1 et )−1 has one fixed point
3) The set [0,1] is countable
although 0 f ′ (t ) 1 for all real t. (a) 1&2 are true but 3 is false

77
OC.14 Analysis
(b) 1&3 are true but 2 is false
(a) Then there exists a point c ∈ [ a, b ] such
(c) 2&3 are true but 1 is false
(d) 1,2&3 all are true that f ( c ) = 0
106. Which of the following is/are incorrect?
(a) Real line is seperable (ie) it has a countable (b) Then there exists a point c ∈ [ a, b ]
dense subset b
(b) Every nonempty perfect set is uncountable
such that f ( c ) < 0 then ∫ f <0
⎧ r n ⎫ a
(c) The set ⎨ n , r = 0,1..., 2 , n ∈ N ⎬ is
⎩2 ⎭ (c) Then there exists a point c ∈ (a, b)
dense in (0,1) b

(d) Cantour set is compact such that f ( c ) < 0 then ∫ f >0


a
107. How many disjoint set can be identified in the
universal set displaying Venn-diagram of 5 (d) Then there exists a point c ∈ (a, b) such
distinct sets b

(a) 5 2 (b) 25
that f ( c ) > 0 then ∫ f <0
a
(c) 35 (d) 53
108. Which of the following statement is/are true? ⎧⎪ 2 x + 5 if x ≤ 1
(a) Let f : D → R and c ∈ D ′ ∩ D then f 112. Let f ( x ) = ⎨ 2
⎪⎩9 x − 2 if x > 1
has a limit at c iff f is continuous at c
(a) f ( x) is continuous
(b) Let f : N → R , then f is continuous on
N (b) f ( x) is differentiable at 1
(c) Every continuous function on a bounded (c) f ( x) is continuous but not differentiable
interval is bounded
at 1
(d) Suppose f and g are continuous on
(d) f ( x) is discontinuous
[0,10] and differentiable on (0,10) . If
113. Let F be a collection of pairwise disjoint open
f ′ ( x ) = g ′ ( x ) for all x ∈ ( 0,10 ) and
subset of Rn for some fixed n ∈ N .
f (π ) = g (π ) then f ( x ) = g ( x ) (a) F is either a finite or a countably infinite
(b) F is uncountable
⎛1⎞
109. Lim x sin ⎜ ⎟ = (c) F is unbounded
x →0 ⎝x⎠ (d) None of these
(a) 0 (b) 1 114. Which of the following is/are true?
(c) π (d) nπ (a) Every deleted neighbourhood of x ∈ R is
an open set
x2 + 3 − 2 x (b) If a set has a maximum and a minimum,
110. Lim
x →1 x2 − 1 then it is compact
(c) No infinite set is compact
1
(a) − (b) -1 (d) Let S ⊆ R then int S ∩ bdS = φ
4
(c) 1 (d) 0
115. Then sequence ( Sn ) deifined by
111. Let f be continuous on [a, b] and suppose
Sn = cos ( nπ ) is
that f ( x ) ≤ 0 for all x ∈ [ a, b]

78
Analysis OC.15
(a) Convergent (b) Divergent 121. 1)Between any two distinct real numbers there
(c) Continuous (d) Not continuous always lies a rational number and therefore
116. Find the limit of infinitely many rational numbers
2)Between any two distinct real numbers.
Lim
n →∞
( n 2 + 2n − n 2 − 3n ) There always lies an irrational number and
therefore infinitely many irrational numbers
(a) 1 is true but 2 is not true
5
(a) (b) 1 (b) 2 is true but 1 is not true
2 (c) Both 1 & 2 are true
1 (d) Both 1 & 2 are false
(c) 0 (d)
2
122. If f is defined by f ( x ) = x + x − 1
2

117. Let ( Sn ) be defined by S1 = 2, ∀ x ∈ R then


Sn +1 = 2 + Sn for all n ∈ N 1
(a) f has local minima at x = ±
(a) Monotone 2
(b) Bounded 1
(c) Monotone & bounded (b) f has local maxima at x′ = ±
2
(d) Unbounded
(c) f has local maxima at x′ ≤ 1
118. 1) A sequence is bounded and monotone iff
it is a cauchy sequence (d) f has no local extrema
2) If for every ε > 0 there exists N ∈ R 123. Which of the following statements is/are true?
such that for every n > N we have (a) The set Z of all integers is not a
neighbourhood of any of its points
Sn < ε then Lim Sn = 0 (b) The set of all rational numbers Q is not a
n →∞
neighbourhood of any of its points
(a) 1 is correct but 2 is incorrect
(c) The set of all irrational number is not a
(b) 2 is correct but 1 is incorrect
neighbourhood of any of its points
(c) Both 1 & 2 are correct
(d) None of these
(d) Both 1 & 2 are incorrect
x
n2
119. Lim
n →∞ 1.01n
= 124. If f is integrable on [ a, b] and ∫ f (t ) dt = 0
a
(a) 0 (b) 1
(c) ∞ (d) n for all x ∈ [ a, b] then

120. Find the infimum and supremum of (a) f (t ) = 0 nowhere in [ a, b]


(b) f (t ) = 0 almost every where in [ a, b]
⎧ n
⎨x ∈ Q : x = ,n∈ N
(c) f (t ) ≠ 0 nowhere in [ a, b]
⎩ n +1
(d) f (t ) > 0 nowhere in [ a, b]
1
(a) inf = & sup = 1
2 125. Which of the following subsets of R are
(b) inf = 2 & sup = 1 neighbourhoods of 3
(a) (2, 4) (b) [2, 4]
1
(c) inf = 1 & sup =
2 ⎧ 1⎫
(c) (2, 7) (d) [2, 4] − ⎨3 ⎬
(d) inf = 1 & sup = 3 ⎩ 4⎭

79
OC.16 Analysis
(a) Converges to 1
126. Find the derived set of the set S = {1,3, 7,11}
(b) Neither converges nor diverges
(a) φ (b) 1 (c) Diverges to 1
(c) R (d) 3 (d) None of these
127. Find limit points of the set 134. The sequence Sn = sin nπθ and θ is a

⎧1 ⎫ ⎧ 1 1 1 ⎫ rational number such that


S = ⎨ : n ∈ N ⎬ = ⎨1, , ... ,...⎬
⎩ n ⎭ ⎩ 2 3 n ⎭ (a) 0 < θ < 1 is not convergent
(a) 0 (b) 1 (b) 0 < θ < 1 is convergent
(c) n (d) ∞ (c) 0 > θ > 1 is convergent
128. Find the limit point of the set (d) 0 > θ > 1 is not convergent

⎧ 3n + 2 ⎫ 1
S =⎨ :n∈ N⎬ 135. The sequerce log is
⎩ 2 n + 1 ⎭ n
(a) Converges to ∞
3 2 (b) Diverges to ∞
(a) (b)
2 3 (c) Diverges to −∞
(c) 2 (d) 1 (d) Converges to −∞
129. Which of the following is/are true?
(1 .2 .....n )
1n
136. The sequence e n 4 n ( )
− n +1 2 1 2 n
(a) A closed or semi-closed interval is not an
open set
(a) Converges to 0 as n → ∞
(b) Every non-empty finite set is not a open
(b) Converges to 1 as n → ∞
set
(c) Diverges
(c) The set of all real number R & the empty
(d) None of these
set φ are open
137. Let E be any bounded closed set in the
(d) The set of rational numbers Q is not an
open set complex plane containing an infinite number of
points, and let M n be the maximum of
⎧1 2 3 ⎫
130. The set A = ⎨ , , ....⎬ V ( x1 ,...., xn ) as the points x1 ,...., xn run
⎩2 3 4 ⎭
(a) Denumerable set (b) Countable through the set E where
(c) Uncountable set (d) Non-denumerable V ( x1 ,........, xn ) = Π
1≤i < j ≤ n
( xi − x j ) is the
131. A set E in R is open if
(a) It contains a neighbourhood of each of its Vandermonde determinant.
points (a) M n
2 / n( n −1)
converges as n → ∞
(b) It does not contains a neighbourhood
(c) It contains a neighbourhood at a (b) M 2 n / ( n −1) converges as n → ∞
(d) None of the above
(c) M n( n −1) converges as n → ∞
132. The set of all rational numbers is
(a) Countable (d) M 2n2 diverges
(b) Closed
(c) Neither open nor closed 138. Let f is differentiable and f ( x ) ≠ 1 for any
(d) Finite real x then
⎧⎪ ( −1) ⎫⎪
n (a) f is increasing function
(b) f is has at most one fixed point
133. The sequence ⎨1 + n : n ∈ N ⎬ is
⎪⎩ ⎪⎭ (c) f has at least two fixed point
(d) f has unique fixed point

80
Analysis OC.17

139. If xn is cauchy then 145. If a function f : C → C is entire and the real


(a) xn is bounded part of f is bounded function then
(a) f is a constant function
(b) xnk → x then xn → x (b) f is a continuous function
(c) [a, b] is complete (c) f is a onto function
(d) f is bijective function
(d) [0,1) is not complete
146. Let f n :[0,1] → R be defined for n ≥ 2
1
140. f ( x ) = on ( 0,1] and f ( x ) = x 2 on ⎧ n2 x 0≤ x≤ 1
x n

[1,∞ ) are ⎪
f n ( x ) = ⎨−n2 x − 2

n ( ) 1 <x<2
n n
(a) Uniformly continuous 2 ≤ x ≤1
⎪ 0
(b) Not uniformly continuous ⎩ n
(c) Continuous
then choose the correct statement
(d) Discontinuous
1 1
141. Let f : [ a, b ] → R be continuous function (a) Lim
n →∞ 0 ∫ fn ( x ) ≠ ∫
0
f ( x ) dx where
then
) f is bounded Lim f n ( x ) = f ( x)
( a
n →∞
(b) f is unbounded
(c) f assumes its max and min values (b) f n ( x ) is continuous ∀ n ≥ 2 and
(d) f (a) < p < f (b) ⇒ there exists x such x ∈ [0,1]
that f ( x) = p (c) f n ( x ) is uniformly continuous

142. f ( x ) = x n on [0,1] is (d) Lim f n ( x ) is continuous function


n →∞
(a) Convergent but not uniformly convergent
(b) Uniformly convergent 147. Let f n be a sequence of continuous real
(c) Divergent valued functions on [0,1] which converges
(d) None of these
uniformly to f.
143. Which of the following statement is/are
correct?
(a) Lim
n →∞
f n ( xn ) = f 1 ( )
2 for any sequence
(a) [a, b] is connected
(b) R is connected 1
{ xn } which converges to
2
(c) Let f : M → N is continuous and M is
connected. Then f ( M ) is connected (b) Lim f n ( xn ) = 1 for any sequence con
n →∞

(d) a, b ∈ I and a < c < b , c ∈ I (ie) I is an 1


interval iff I ⊆ R is connected. verges to
2
(c) Lim f n ( xn ) ≠ f ( x ) for any sequence
144. The subset {( a, b, c, d ) ∈ R 4
}
ad − bc ≠ 0 is n →∞

(a) Open and dense in R n diverges to f ( x )


(b) Compact
(d) Lim f n ( x n ) = f (0)
(c) Bounded n→ ∞

(d) Deneuramable

81
OC.18 Analysis

148. Let f : R → R be differentiable and assume φ  f is A-measurable


there is no x ∈ R . Such that
(b) If φ : R 2 → R1 is continuous then
f ( x ) = f ′ ( x ) = 0 . Then
φ ( f , g ) is A-measurable
(a) S = {x 0 ≤ x ≤ 1} is continuous
(c) All A-measurable functions form a linear
(b) S = {x 0 ≤ x ≤ 1, f ( x) = 0} is finite vector space over R
(d) Functions
(c) S = {x 0 ≥ x ≥ 1, f ( x) ≠ 0} is
sup f n , inf f n , lim sup f n , lim inf f n
infinite n n n n

(d) S = { f ( x ) 1 ≤ x ≤ ∞} is finite are A-measurable


149. If f is a bounded real valued function on [0,1]. 152. Let and be the norms of linear
1 2
f is Lebesgue measurable iff
vector space X. Then
(a) Sup ∫ψ dm = inf ∫ φdm , where m is the (a) 1
& 2
are equivalent norms
Lebesgue measure on [0,1] and (b) The relative induced distance are topologi
ψ ≤ f ≤φ cally equivalent
(c) For any {xn } ⊂ X , xn → 0 iff x n →0
(b) Sup ∫ψ dm > inf ∫ φdm ,where m is the (d) None of these
1 2

Lebesgue measure on [0,1] and


153. Let X & Y be the Banach Spaces with the norm
ψ ≤ f ≤φ
( x, y ) 1 = x + y
∫ψ dm < inf ∫ φdm , where m is the
X ×Y X Y
(c) Sup
Lebesgue measure on [0,1] and P P
(a) x, y P, X ×Y
=P x + y Y ,P ≥1
ψ ≤ f ≤φ X

(b) x, y = max x X , y ( )
(d) Sup ∫ψ dm ≠ inf ∫ φdm ,where m is the ∞, X ×Y Y

Lebesgue measure on [0,1] and (c) x, y P, X ×Y


= x + y
ψ ≤ f ≤φ
150. Which of the following need not be hold (d) x, y ∞, X ×Y
= min x X , y ( Y
)
always
154. The closed unit ball of a normed linear space X
(a) f is continuous and bounded on R then f
is compact iff
is uniformly continuous on R
(a) X is finite dimensional
(b) f is uniformly continuous on a bounded
(b) X is closed
interval I then f is bounded on I
(c) X is infinite dimensional
(c) If < f n ( x ) > and < g n ( x ) > con- (d) X is bounded

verges on I. Then f n ( x) + g n ( x) uni- 155. If 1 ≤ p < q < ∞ then


formly convergent (a)  p ( R ) ⊂  q ( R ) ⊂  ∞ ( R )
(d) All the above
151. Let f, g, f1,.... be A-measurable function for (b)  p (R ) is a proper subspace of  q (R )
some fixed σ - algebra A. Then
(c) The identity map Id :  p ( R ) →  q ( R )
(a) If φ : R1 → R1 is B1-measurable then is continuous

82
Analysis OC.19

(d)  1 ( R) is a dense subset of  q (R ) with (b) ρ ( f , g ) = inf{ d Y ( f ( x ), g ( x )) x ∈ X }

respect to the convergence in  q (R ) (c) ρ ( f , g ) = f ( x ), g ( x )


156. Let ( X , d X ) and (Y , dY ) be two metric (d) ρ ( f , g ) = d ( f ( x ), g ( x ))
space and let { f n } be a sequence of 160. Let E be a normed linear space. E is complete
iff
functions f n : X → Y . For any function
∑ ∑
∞ ∞
(a) xn < ∞ then xn converges to
f : X →Y then 1 1

an s ∈ E
(a) { f n } converges uniformly to f ∞ ∞

(b) { f n } diverges (b) ∑x


1
n = ∑ xn
1

sup{dY ( f n ( x), f ( x)) x ∈ X } → 0 ∞ ∞


(c)
as n → ∞ (c) ∑
1
xn < ∞ then ∑x
1
n diverges

(d) { f n } → 1 / 2 as n → ∞ ∞ ∞

[
157. The function f n : 0, ∞ ) → R is defined by
(d) ∑
1
xn = 0 then ∑x
1
n converges to s

ne where s ∈ E .
⎛x⎞
f n (x ) = e ⎜ ⎟
−x
161. { f n } is a sequence of Lebesgue measurable
⎝n⎠
1

(a) { f n } converges pointwise function on [0,1] such that Limn→∞ ∫ f n dx = 0


0

⎛1 ⎞ and there is an integrable function g on [0,1]


(b) { f n } converges on ⎜ ,1⎟
⎝n ⎠ such that f n
2
≤ g for each n. Then
(c) { f n } converges uniformly but not 1

pointwise converges
(a) Lim
n →∞ ∫ 0
f n dx = 0

(d) { f n } bounded 1
(b) Lim ∫0 f n dx = 0
2

n→∞
1
158. Let f n : R → R be given by f n ( x ) =
1
sin( nx ) (c) Lim ∫0 f n dx = 0
3
n n →∞

(a) { f n } converges uniformly to zero ∞


(d) Lim ∫0 fn dx = 0
n→∞
(b) { f n′} of derivates converges to zero 162. Let S be a Lebesgue measurable subset of R
(c) { f n′} of derivates does not converges and let f , g : S → R be measurable
(d) { f n } diverges function. Then
(a) f + g is measurable
159. Let ( X , d X ) and (Y , dY ) be two metric
(b) If φ ∈ C (R ) , then φ ( f ) is measurable
spaces, and assume that X is compact. Then
(c) fg is measurable
(a) ρ( f , g) = sup{dY ( f (x), g(x)) x ∈ X} (d) f  g is measurable

83
OC.20 Analysis
163. The total number of functions from the null set
to a finite set B having n elements is equal to
x ≠ 0 and that f ′( x) → 3 as x → 0 then
(a) n (b) 2 n (a) f ′(0) exists & equal to 3

(c) n 2 (d)1 (b) f ′(0) exists & equal to 0


(c) f ′(0) exists & equal to ∞
an b
164. If > n (an > 0, bn > 0) then (d) f ′(0) need not exists
an +1 bn +1
169. If f : S → R m be differentiable at
(a) Σan diverges ⇒ Σbn converges
c ∈ S ⊆ R n then
(b) Σbn diverges ⇒ Σan converges
(a) f is continuous at c
(c) Σbn converges ⇒ Σan converges
(b) f is constant at c
(d) Σan & Σbn diverges (c) f is discontinuous at c
(d) None of these
165. Take two arbitrary partitions P1 and P2 over
[a, b] then
170. If f ( x ) is differentiable function for ∀x ∈ R
such that f (1) = 5 and f ( x) < 5 ∀ x ≠ 1 .
(a) U ( f , P1 ) ≥ L( f , P2 ) ∀ P1 , P2
Then
(b) U ( f , P1 ) < L( f , P2 ) ∀ P1 , P2 (a) f ( x ) is monotonically decreasing
(c) U ( f , P1 ) ≤ L( f , P2 ) ∀ P1 , P2 (b) f ( x ) is bounded
(d) U ( f , P1 ) = L( f , P2 ) ∀ P1 , P2 (c) f ( x ) is not monotonic

166. For a function f : R → R , if f is monotone (d) f ( x ) is monotonically increasing


then
(a) f is Borel measurable ⎧ cos x x≥0
(b) f is Lebesgue measurable 171. The function f ( x) = ⎨ has
(c) f is Reimann integral
⎩− cos x x<0
(d) f is Improper Reimann integral (a) Continuity at x = 1
(b) Removable distcontinuity at x = 0
167. If < an > be a sequence such that
(c) Continuity at x = 1
Lim( an +1 − an ) =  then the sequence
n →∞
< bn > (d) Irremovable discontinuity at x = 0
172. For Lebesgue measure which property is not
an correct?
wher bn =
n (a) μ ( A + x0 ) = μ ( A) + μ ( x0 )
(a) Converges to 
(b) Converges to 0
(b) μ ( A ∪ B ) = μ ( A) + μ ( B)
(c) Diverges (c) 0 ≤ A ≤ μ ( B ) ≤ ∞
1
(d) Converges to (d) μ (I ) = l(I )
n
168. Let f be a continuous real function on R of ∞
1
which it is known that f ′( x ) exsist for all
173. The series ∑n
n =1
p converges if

84
Analysis OC.21

(a) p < 1 (b) p > 1 180. Which of the following inequality is true for

(c) p ≤ 0 (d) p = 1 x>0

174. If f is constant function on [a,b] then total x


(a) log (1 + x ) < <x
variation of f on [a,b] is 1+ x
(a) V f ( a, b) ≥ 0 (b) V f ( a, b) ≤ 0 x
(b) < log (1 + x ) < x
1+ x
(c) V f ( a , b ) = 0 (d) V f ( a , b) ≤ 1
x
175. The space C[a,b] of all continuous functions (c) x < log(1 + x ) <
on [a,b] is a normed linear space if 1+ x
(a) y ( x1 ) < Max y ′( x1 ) 1+ x
a ≤ x ≤b (d) < log x < x
x
(b) y ( x1 ) < Max y ( x1 )
a ≤ x ≤b 181. Which of the following series is absolutely
y ( x1 ) < Max y ( x1 ) + Max y ′( x1 ) convergent?
(c) a ≤ x ≤b a ≤ x ≤b
1 ( −1)n
(d) y ( x1 ) > aMax y ′( x1 ) (a) ∑ (b) ∑
≤ x ≤b n n3/2

cos x ( −1)n 1
(d) ∑ log n + 1
176. ∫
(c) ∑
dx is n ( )
2
log x
(a) Convergent 182. Let f : [ 0,10 ) → [ 0,10 ] be a continuous
(b) Conditionally convergent
mapping then
(c) Divergent
(d) Conditionally divergent (a) f has at least 9 fixed points


(b) f has at least one fixed point
177. If a power series ∑a z
n =0
n
n
converges for (c) f need not to have any fixed point
(d) f has at most one fixed point
z = a then it is converges absolutely for
183. If u n = n +1 − n and Vn = n 4 + 1 − n 2 then
(a) z <a (b) z >a
∞ ∞
(c) z = a (d) z ≥a (a) ∑ u n & ∑ Vn both converges
n =1 n =1

178. Which of the following is/are true? ∞ ∞


(a) The set [0,1] is countable (b) ∑ u n converges but ∑ Vn diverges
n =1 n =1
(b) The set [0,1] is not countable
(c) m*[0,1]=0 ∞ ∞

(d) m*[0,1]=1 (c) ∑ u n diverges but ∑ Vn converges


n =1 n =1

1 sin (1/ x ) ∞ ∞
179. ∫ 0
x
dx (d) ∑ u n & ∑ Vn both diverges
n =1 n =1

(a) Integral is convergent


184. If f n ( x ) = tan −1 nx then
(b) Integral is absolutely convergent
(c) Integral is divergent (a) f n (x ) is point wise convergent in [0,b],
(d) Integral is not absolutely convergent
b <π /2

85
OC.22 Analysis
which is continuous on R but no where
(b) f n (x ) is uniformly convergent in (0, b).
differntiable
(c) f n (x ) is uniformly convergent in [0,b], (d) The set of points of discontinuity of the
function defined by
π f (x) = Lim Lim cos2m (nπ x) is uncountable
b> n→∞ m→∞
2
⎛1 n +1⎞
(d) f n (x ) is uniformly convergent in (0,b), 189. The series ∑ ⎜⎝ n − log ⎟ is
n ⎠
π (a) Divergent
b> (b) Convergent
2
(c) Neither convergent nor divergent
185. Let F ( x) is defined on [a, b] such that (d) None of the above
1 190. The series 1 + x 2 + x 4 + x 6 + .... is a power
F ( x) = x + f (x) where f(x) is differentiable
20 series with radius of convergence is
(a) 0 (b)1
on [a, b] and and f ′( x ) < 10 then from the (c)2 (d)3
following statements select the correct
statement 191. A monotone function f on [a, b] , then which of
the following imply true?
(a) F ( x) is of bounded variation
(a) Any monotone increasing function is the
(b) F ( x) is one -one sum of an absolutely continuous function
and a singular function.
(c) F ( x) is decreasing
(d) None of these (b) Let fn be a sequence of non-decreasing
186. Select the correct statement? singular functions on a, b ∋ the function
1 a n +1 f ( x) ∑ f n ( x) is everywhere finite. Then
(a) Lim sup( a n )
n
≥ lim sup , an > 0
an
f is also singular.
1 a n +1 (c) There is a strictly increasing singular function
(b) Lim inf( a n )
n
≥ lim inf , an > 0
an on 0,1
(c) Lim inf(an + bn ) ≥ lim inf an + lim inf bn (d) Any monotone increasing function is the
sum of an discontinuous function and a
< an > & < bn > are bounded singular function
(d) Lim sup(an + bn ) ≤ lim sup an + lim sup bn 192. A function f satisfy a Lipschitz condition on an
interval then
< an > & < bn > are bounded (a) it is absolutely continuous
187. The limit superior & limit inferior of <(-1)n > is (b) an absolutely continuous function f satisfies
respectively given by a Lipschitz condition iff f ′ is bounded
(a) 1,-1 (b) -1,-1 (c) discontinuous
(c) 0,-1 (d) -1,0 (d) an absolutely continuous function f does
188. Which of the following is correct ? not satisfies a Lipschitz condition iff f ′ is
(a) The set of points of discontinuities
bounded.
of a function is countable
(b) The set of pionts of removable 193. Let A ⊂ M be connected and let f : A → R be
discontinuties of a function is countable
(c) There cannot be a function defined R continuous with f ( x) ≠ 0 for all x ∈ A then

86
Analysis OC.23

(a) f ( x) 0 for all x ∈ A 5


dx
(b) f ( x) 0 for all x ∈ A (a) ∫ 3
7x 2x4
0
(c) f is constant
5
(d) f : A → R be discontiuous at f ( x) 0 for dx
(b) ∫ 2
x − 5x 6
all x ∈ A 0

194. The radius of converges of the power series ∞


dx
(c) ∫ x 3
2x 2
2n n 1
∑ z is
n! (d) None of the above
(a) 0 (b) ∞ 198.Which of the following series converge uniformly
(c) 1 (d) e
2 x2 x2
195.Pick out the sequences which are uniformly (a) x ... over the interval
1 x2 (1 x 2 ) 2
convergent?
1 [−1,1]
(a) f n ( x ) 1 ( x − n) 2 on
0,∞ ∞

(b) ∑ (an cos nt bn sin nt ) over the interval


sin x on ⎡⎢⎣0, π 2
n n 1
(b) f n ( x )
−π, π where ∑ n an ∞ and
1
(c) f n ( x ) on −∞,0 ∑b ∞
1 ( x − n) 2 n
n

xn ∞
(d) f n ( x)
n
1 on 0,1 (c) ∑e
n 0
nx
cos nx over the interval 0,∞

196.Pick out the functions which are Riemann ∞


n
integrable on the interval 0,1 (d) ∑ (−1)
n 1
n

n 2
⎧⎪1 if x ∈ α1 , α2 ,..., αn
⎪⎨ 199.Let f ∈ C 0, π . Determine the cases where the
(a) f ( x) ⎪⎪0 otherwise
⎩ given condition implies that f 0
where α1 , α2 ,..., αn are fixed, but arbitrarily π

chosen numbers in 0,1 (a) ∫ f ( x) cos nxdx 0 for all integers n ≥ 0


0

⎧⎪0, if x is irrational or if x 0
⎪⎪ π

(b) f ( x) ⎨
⎪⎪sin qπ if x p , p and q (b) ∫ f ( x) sin nxdx 0 for all integers n ≥ 1
⎪⎩ q 0

are positive coprime integers π

∫x
n
(c) f ( x) dx 0 for all integers n ≥ 0
⎧1 if x is rational
(c) f ( x) = ⎨0 if x is irrational
0

⎩ (d) None of the above


(d) None of the above

x2 n
197.Which of the following improper integrals are 200.Consider the series ∑ (−1) n

n2
. Pick out
convergent n 1

the true statements


(a) The series converges uniformly on R

87
OC.24 Analysis
(b) the series does not converge absolutely for
1 22 33
any real value of x (c) 1 ...
(c) The series converges conditionally 22 33 44
(d) The series is divergent. (d) None of these
201.Let f : R → R be a continuous function. Which 205.Which of the following functions are continuous?

of the following imply that it is uniformly
(a) f ( x) ∑n x 2 n
, x ∈ ⎡⎢− 1 , 1 ⎤⎥
continuous n 1
⎣ 2 2⎦
(a) f is differentiable and its derivative is bounded
on R x2 x2
f ( x) x2 ..., x ∈ R
(b) f is 2π periodic (b) 1 x2 1 x2
2

(c) f is absolutely continuous


(d) None of the above ∞
cos nx
202.Let f be a continuously differentiable 2π periodic (c) f ( x) ∑ (−1) n
3
, x ∈ −π, π
n 1 n 2
real valued function on the real line. Let
(d) All the above
π
206.Which of the following functions are Riemann
an ∫ f (t ) cos ntdt where n is a non-negative
−π integrable on the interval 0,1 ?
integer. Pick out the true statements
⎧⎪⎪0, if x is rational
(a) an → 0 as n → ∞ (a) f ( x) f ( x) ⎨
⎪⎪⎩1 if x is irrational
1
(b) an ≤ c for all n, where c 0 is a constant (b) f ( x ) lim cos 2 n (24π x )
n n →∞

independent of n
⎧⎪cos x, if 0 ≤ x ≤ 1
(c) The deriative of f is also a 2π periodic ⎪⎪ 2
function. (c) f ( x) ⎨
⎪⎪sin x, if 1
⎪⎩ x ≤1
2
(d) an → n as n → ∞
(d) None of these
203.Let f n and f be continuous functions on an interval 207.Consider the polynomial
[a,b] and assume that f n → f uniformly on p ( x) a0 a1 x a2 x 2 ... an x n with real
a, b . Pick out the true statements. coefficients. Pick out the case(s) which ensure

(a) If f n are all continuously differentiable, then that the polynomial p (.) has a root in the interval
f is continuously differentiable 0,1
(b) If f n are all Riemann integrable, then f is
a1 an
Riemann integrable (a) ao ... 0
2 n 1
(c) If xn → x in a, b , then f n ( xn ) → f ( x)
a0 a1 an
(d) All the above (b) 1.2 ... 0
2.3 n 1 n 2
204.Pick out the convergent series
(c) a0 0 and a0 a1 ... an 0

(n 1) n
(a) ∑ n 3
(d) None of the above
n 1 n 2
208.Pick out the true statements

1 4 n (a) If a continuous function f : R → R is
(b) ∑
n 1 1 5n uniformly continuous, then it maps cauchy
sequences into cauchy sequences

88
Analysis OC.25

sin( x 2 ) 212.Let f : (0,1) → R be continuous. Pick out the


(b) The function f ( x) is uniformly statements which imply that f is uniformly
sin 2 x
continuous
continuous on the interval 0,1
1 1
(c) A continuous function f : R → R is (a) f ( x) x 2 sin
x3
uniformly continuous, if it maps cauchy
sequences into cauchy sequences (b) f 1 n → 1 2 and f 1 n 2 → 1 4
(d) All the above
(c) f ( x) − f ( y ) ≤ x − y for all x, y ∈ 0,1
209.Let f : −π, π → R be continuous. Pick out the
(d) All the above
cases which imply that f ≡ 0 213.Which of the following statements are true?
π
(a) Let amn , m, n ∈ N ; be a double sequence of
(a) ∫ f ( x) cos nxdx 0 for all n ≥ 0
amn ≤ m for
−π
real numbers ∋ all
π
n

(b) ∫ f ( x) sin nxdx 0 for all n ≥ 1 ∞ ∞


amn ∞ ∞
amn
−π m, n ∈ N . Then ∑∑ m n ∑∑ m n
m 1 n 1
2
n 1 m 1
2

(c) ∫x
n
f ( x)dx 0 for all n ≥ 0 (b) Let amn , m, n ∈ N , be an arbitrary double
−π sequence of real numbers.
(d) None of the above ∞ ∞ ∞ ∞

210.Let an be a sequence of positive terms. Pick


Then ∑∑ a
m 1 n 1
2
mn ∑∑ a
n 1 m 1
2
mn

out the cases which imply that ∑ an is convergent (c) Let amn , m, n ∈ N be an arbitrary double

an 1 ⎛ n ⎞⎟
2 sequence of real numbers.
⎜⎜
(a)
an ⎜⎝ n 1⎠⎟⎟ for all n ∞ ∞ ∞ ∞

Then ∑∑ a
m 1 n 1
3
mn ∑∑ a
n 1 m 1
3
mn
2 2
(b) ∑ n an ∞
(d) None of the above
3
(c) nlim n 2 an 3 214.Which of the following functions are uniformly
→∞ 2
continuous?
(d) None of thes above
(a) f ( x) sin( x sin x) on 0,∞
211.Pick out the true statements
x2 (b) f ( x) x sin 1
(a) cos x 1−
for all x 0 x on 0,1
2
(b) If p is a polynomial in one variable with the (c) f ( x) sin 2 x on 0,∞
real coefficients which has all its roots real, (d) All the above
then its derivative p1 has all its roots real aswell. 215.Pick out the functions which are continuous at
(c) The equation cos(sin x ) x has exactly one least one point in the real line
⎧⎪⎪sin x if x is rational
solution in the interval ⎡⎢⎣ 0, π 2 ⎤⎥⎦ (a) f ( x) ⎨
⎪⎪⎩0 if x is irrational
(d) sin x x 2 for all x 0
⎧⎪⎪ x if x is rational
(b) f ( x) ⎨
⎪⎪⎩0 if x is irrational

89
OC.26 Analysis

⎧⎪⎪1 if x is rational continuous at (0, 0)


(c) f ( x) ⎨
⎪⎪⎩0 if x is irrational ⎧⎪ 1

⎪⎨ x sin y y sin 1 if xy ≠ 0
(d) None of the above f ( x, y ) x
(a) ⎪⎪
216.Pick out the functions which are uniformly ⎪⎪⎩0 otherwise
continuous
sin x x2 y2
(a) f ( x) , x ∈ 0,1 (b) f ( x, y ) ( x, y ) ≠ 0, 0
x x2 y 2

(b) f ( x) sin 2 x, x ∈ R xy
(c) f ( x, y ) 2
, ( x, y ) ≠ 0, 0
x y2
1
(c) f ( x) , x ∈ 0,1 (d) None of the above
x
(d) All the above 221.Which of the following functions are uniformly
continuous?
217.Pick out the uniformly continuous function from
the following functions (a) f ( x) x2 , x ∈ R
(a) f ( x) sin 2 x, x ∈ R (b) f ( x) x sin 1 , x ∈ 0,1
x
(b) f ( x) x ,1 ≤ x ≤ 2
(c) f ( x ) sin 2 x, x ∈ R
3
(c) f ( x) x , x ∈ R
(d) f ( x) x3 , x ∈ R
(d) All the above
222.Which of the following maps are differentiable
218.Which of the following functions are differentiable
everywhere?
at x 0?
1
⎧⎪ x 2 sin 1 (a) f ( x) x3 sin when x ≠ 0 and f (0) 0
⎪ if x ≠ 0 1x1
x
(a) f ( x) ⎨
⎪⎪0 3
⎪⎩ if x 0 (b) f ( x) x x, x ∈ R

⎧⎪ x 2 if x is rational (c) f : R → R ∋ f ( x) − f ( y ) ≤ x − y 2
for all
⎪⎨
(b) f ( x) ⎪⎪⎩0 if x is irrational x and y ∈ R
(d) None of the above
(c) f ( x) x x
(d) None of the above 223.Pick out the true statements
219.Which of the following series are convergent? (a) If the series ∑
n
an convergent; a ≥ 0 for all
n


1 ⎛1⎞ n, and if the sequence bn is bounded, then
(a) ∑ n sin ⎜⎜⎜⎝ n ⎠⎟⎟⎟
n 1
∑ an bn is absolutely convergent
n
∞ 2
2n 3
(b) ∑
n 1 5n3 7
(b) If the series ∑
n
an is convergent and if ∑ bn is
n

absolutely convergent then ∑ an bn is



( n 1) n n

(c) ∑ n 3 absolutely convergent.


n 2
n 1

(d) None of the above (c) If the series ∑


n
an and ∑ bn are convergent,
n

220.Let f : R 2 → R be a mapping ∋ f (0, 0) 0. then ∑


n
an bn is also convergent
Determine which of the following are jointly (d) None of the above

90
Analysis OC.27
224.Which of the following functions are continuous? ∞
( n 1) n
1 (b) ∑ n 3
(a) f ( x ) lim log(e n n
x ); x ≥ 0 n 1 n 2
n →∞ n

⎛ 1 ⎞
(b) f ( x ) x x − x , x ≥ 1 where x
2
x
(c) ∑ ⎜⎝⎜ n
n 1
3
1 3
− n⎟⎟⎟

denotes the largest integer less than or equal (d) None of the above
to x. 229. Let {an } be the sequence of consecutive
xe x 2 nx positive solutions of the equation tan x = x
(c) f ( x) lim ,x∈R
e nx
1
n →∞ and let {bn} be the sequence of consecutive
(d) None of the above positive solutions of the equation tan x = x.
225.Pick out the uniformly continuous functions over Then
the interval 0,1 1 1
∑ ∑
∞ ∞
(a) n=1
an convergebut n=1
bn diverges
(a) f ( x) exp − 1
x2 1


(b) n=1
an diverges but
(b) f ( x) sin 1
x
1

(c) f ( x) x sin
1 n=1
bn converges
x
1 1
∑ ∑
∞ ∞
(d) All the above (c) Both n=1
an and n=1
bn converge
226.Pick out the true statements
1 1
∑ ∑
∞ ∞
(a) cos 2 x − cos 2 y ≤ x − y for all x, y ∈ R
(d) Both n=1
an and n=1
bn diverge
(b) If f : R → R satisfies
230. Let x0 = 0. Define xn+1 = cos xn for every n ≥ 0.
f ( x) − f ( y ) ≤ x − y 2 for all x, y ∈ R Then
then f must be a constant function. (a) {xn} is increasing and convergent
(c) Let f : R → R be continuously differentiable (b) {xn} is decreasing and convergent
and f ′( x ) ≤ 4 5 for all x ∈ R. The ∃ a (c) {xn} is convergent and x2n < limm→∞ xm <
x2n+1 for every n∈
unique x ∈ R ∋ f (x) x
(d) {xn} is not convergent
(d) None of the above
227.Pick out the uniformly continuous functions 231. Let f : 2 → 2 be defined by
sin 2 x, x ∈ 0, ∞
f(x, y) = (e x+y, ex–y). The area of the image of
(a) f ( x)
the region{(x, y) ∈2 :0 < x, y < 1} under the
π mapping f is
(b) f ( x) cos x cos , x ∈ 0,1
x (a) 1 (b) e – 1
(c) e 2
(d) e2 –1
(c) f ( x) sin x cos π , x ∈ 0,1
x
232. Let f, g : [0, 1] →  be defined by
(d) All the above
1
228.Pick out the convergent series ⎪⎧ x if x′=n for n ∈ 
and f(x) = ⎨

1 ⎪⎩0 otherwise
(a) ∑ 1 1
n 1 n n

91
OC.28 Analysis

⎧1if x ∈ ∩ [0,1] from the Banach Space X into the Banach


Then g(x) = ⎨ 0 otherwise space Y.

If f is continuous, then the graph of f is always
(a) Both f and g are Riemann integrable
compact.
(b) f is Riemann integrable and g is Lebesgue
The correct statements amongst the above
integrable
are :
(c) g is Riemann integrable and f is Lebesgue
integrable (a) P and R only
(d) Neither f nor g is Riemann integrable (b) Q and R only

233. Consider the following statements : (c) P and Q only


P : The family of subsets (d) R and S only
⎧ ⎛ 1 1⎞ ⎫ 235. The subspace P = {(x, y, z) ∈ R3 : z = x2 + y2
⎨A n = ⎜ − , ⎟ , n =1, 2,...⎬
⎩ ⎝ n n⎠ ⎭ +1} is
satisfies the finite intersection property. (a) compact and connected
Q : On an infinite set X, a metric d : X × X →
(b) compact but not connected
⎧0, x = y
R is defined as d(x, y) = ⎨ . (c) not compact but connected
⎩1, x ≠ y
(d) neither compact nor connected
The metric space (X, d) is compact.
236. Let P = (0, 1) Q = [0, 1]; U = (0, 1); S = [0,
R : In a Frechet (T1) topological space, every
1], T = R and A = {P, Q, U, S, T}. The
finite set is closed.
equivalence relation ‘homeomorphism’
S : If f : R → X is continuous, where R is induces which one of the following as the ∈
given the usual topology and (X, τ) is a partition of A?
Hausdorff (T2) space, then f is a one-one
(a) {P, Q, U, S}, {T}
function.
(b) {P, T}, {Q}, {U}, {S}
Which of the above statements are correct?
(c) {P, T}, {Q, U, S}
(a) P and R (b) P and S
(d) {P, T}, {Q, U}, {S}
(c) R and S (d) Q and S
237. Let x = (x1, x2,...) ∈ l4, x ≠ 0. For which one
234. Consider the statements ∞

P : If X is a normed linear space and M ⊆ X of the following values of p, the series ∑x y


i =1
i i

is a subspace, then the closure M is also


a subspace of X. converges for every y = (y1, y2,...) ∈ lp?
Q : If X is a Banach space and ∑ xn is an (a) 1 (b) 2
absolutely convergent scries in X, then (c) 3 (d) 4
∑ xn is convergent. 238. For 0 ≤ x ≤ 1, let
R : Let M1 and M2 be subspaces of an inner ⎧ n
⎪ , if x is irrational
product space such that M1 ∩ M2 = {0}. fn(x) = ⎨1 + n
Then ⎪⎩ 0, if x is rational
∀ m1 ∈ M1, m2 ∈ M2; ||m1 + m2||2 = ||m1||2
and f(x) = lim f (x). Then, on the interval [0,
+ ||m2||2. n→∞ n

S : Let f : X → Y be a linear transformation 1]

92
Analysis OC.29

(a) f is measurable and Riemann integrable ⎧ xy


⎪ 2 2 52 [1 − cos(x + y )], ( x, y) ≠ (0, 0)
2 2

(b) f is measurable and Lebesgue integrable f(x, y) = ⎨(x + y )



(c) f is not measurable ⎩k, (x, y) = (0, 0)
(d) f is not Lebesgue integrable Then the value of k for which f(x, y) is
continuous at (0, 0) is
x
239.Let fn(x) = {( n − 1) x + 1}{nx + 1} (a) 0 (b)
1
2
n
and sn(x) = ∑ f ( x)j for x ∈ [0, 1]. 3
j =1
(c) 1 (d)
2
The sequence {sn}
(a) converges uniformly on [0, 1] 243. Let A and B be disjoint subsets of  and let
m* denote the Lebesgue outer measure on
(b) converges pointwise on [0, 1] but not .
uniformly Consider the statements:
(c) converges pointwise for x = 0 but not for P : m* (A ∪ B) = m* (A) + m* (B)
x∈ (0, 1)
Q : Both A and B are Lebesgue measureable
(d) does not converge for x ∈ [0, 1]
R : One of A and B is Lebesgue measureable
240. If f : [1, 2] → R is a non-negative Riemann-
integrable function such that Which one of the following is correct?
∈ 2 2 (a) If P is true, then Q is true
f ( x)
∫1 x
dx = k ∫ f ( x) dx ≠ 0, then k belongs
1
to the interval (b) If P is NOT true, then R is true
(c) If R is true, then P is NOT true
⎡ 1⎤ ⎡1 2⎤
(a) ⎢ 0, ⎥ (b) ⎢ ,
⎣ 3⎦ ⎣3 3 ⎥⎦ (d) If R is true, then P is true

⎡2 ⎤ 244. Let f :  → [0, ∞) be a Lebesgue measurable


⎡ 4⎤
(c) ⎢ , 1⎥ (d) ⎢1, ⎥ function and E be Lebesgue measurable
⎣3 ⎦ ⎣ 3⎦
subset of  such that then ∫f
E
dm = 0 ,

241. The set X =  with metric where m is the Lebesgue measure on  .Then
x− y (a) m(E) = 0
d(x, y) = is
1+ x − y (b) {x ∈  : f(x) = 0} = E
(a) bounded but not compact (c) m({x ∈ E : f(x) ≠ 0}) = 0
(d) m({x ∈ E : f(x) = 0}) = 0
(b) bounded but not complete 245. Let {f n } be a sequence of real valued
(c) complete but not bounded differentiable functions on [a, b] such that
fn(x) → f(x) as n → ∞ for every x ∈ [a, b]
(d) compact but not complete and for some Riemann-integrable function f :
[a,b] → R. Consider the statements
242. Let

93
OC.30 Analysis

P1 : {fn} converges uniformly 248. Let X =  be equipped with the topology


P2 :{fn′} converges uniformly generated by the basis consisting of sets
b b An = {n, n + 1, n + 2, ...}, n ∈  . Then X is
P3 : ∫
a
f n ( x) dx → ∫ f ( x) dx
a
(a) Compact and connected
(b) Hausdorff and connected
P4 : f is differentiable (c) Hausdorff and compact
Then which one of the following need NOT (d) Neither compact nor connected
be true
249. Consider the metrics
(a) P1 implies P3 (b) P2 implies P1
F I
z
1/ 2
b
(c) P2 implies P4 (d) P3 implies P1 d2(f, g) = GG | f (t) JJ
g(t) |2 dt and d∞ (f, g)
n n
H a K
x x = sup ⏐f (t) g(t)⏐ on the space X = C[a, b]
246. Let fn(x) = and gn(x) = for x ∈
1+ x 1 + nx t ∈[ a , b ]

[0,1] and x ∈ N. Then on the interval [0,1], of all real valued continuous functions on
(a) both {fn} and {gn} converge uniformly [a, b]. Then which of the following is TRUE?
(b) neither {fn} nor {gn} converges uniformly (a) Both (X, d2) and (X, d∞) are complete.
(c) {fn} converges uniformly but {gn} does (b) (X, d2) is complete but (X, d∞) is NOT
not converge uniformly complete.
(d) {gn} converges uniformly but {fn} does (c) (X, d∞) is complete but (X, d2) is NOT
not converge uniformly complete.

xn ∞
xn (d) Both (X, d2) and (X, d∞) are NOT
247. Consider the power series ∑ and ∑ . complete.
n =1 n n =1 n
Then 250. A function f : R → R need NOT be Lebesgue
(a) both converge on (–1,1] measurable if
(b) both converge on [–1,1) (a) f is monotone
(c) exactly one of them converges on (–1,1] (b) {x ∈ R : f(x) ≥ α} is measurable for each
α∈Q
(d) none of them converges on [–1,1)
(c) {x ∈ R : f(x) = α} is measurable for each
α∈R
(d) For each open set G in R, f–1(G) is
measurable

94
Analysis OC.31

ANSWERS

1. (b) 2. (a,c) 3. (b,d) 4. (d) 5. (a,c) 6. (b)

7. (b,d) 8. (a,b,c,d) 9. (a,b,c,d) 10. (b) 11. (a,b,c,d) 12. (a,c)

13. (a,b,d) 14. (c) 15. (b) 16. (a,b,c,d) 17. (a) 18. (a,b)

19. (a,b,c,d) 20. (a,b,c) 21. (c,d) 22. (a,b,c) 23. (a) 24. (a)

25. (c) 26. (a) 27. (c) 28. (a) 29. (a) 30. (a)

31. (a,b) 32. (c) 33. (a,b,c) 34. (d) 35. (b) 36. (c)

37. (a,b,c) 38. (b) 39. (c) 40. (b) 41. (c) 42. (a,b)

43. (a) 44. (a,b,c,d) 45. (b,c,d) 46. (a) 47. (a,b) 48. (a,b)

49. (b) 50. (a,b,c) 51. (a,b,c,d) 52. (a,b,d) 53. (c,d) 54. (b,d)

55. (a,b,c) 56. (c) 57. (a) 58. (d) 59. (c) 60. (a,c)

61. (b) 62. (c,d) 63. (a,b,c) 64. (a,b,c,d) 65. (b,c) 66. (c)

67. (a,b,c) 68. (a,b,c,d) 69. (a) 70. (a,b) 71. (a,b,c) 72. (a,c,d)

73. (a,b) 74. (a,b,c,d) 75. (a,b,c) 76. (b,c,d) 77. (a,b,c) 78. (a,b)

79. (b,c) 80. (a,c) 81. (a,c) 82. (a,b,c) 83. (a,b,c) 84. (a,b,c)

85. (a,b,c) 86. (a,c) 87. (a,b) 88. (a,b,c) 89. (a,b) 90. (a,b,c)

91. (a,b) 92. (a,b,c) 93. (a,b) 94. (a,b,d) 95. (b,c) 96. (a)

97. (a,b,c) 98. (c,d) 99. (a,b) 100. (a,b) 101. (a) 102. (a,b,c,d)

103. (a) 104. (a,b,c) 105. (a) 106. (c) 107. (b) 18. (b,d)

109. (a) 110. (a) 111. (b) 112. (a,c) 113. (a) 114. (a,d)

115. (b) 116. (a) 117. (a,b,c) 118. (d) 119. (a) 120. (a)

121. (c) 122. (b) 123. (a,b,c) 124. (b) 125. (a,b,d) 126. (a)

95
OC.32 Analysis

127. (a) 128. (a) 129. (a,b,c,d) 130. (a,b) 131. (a) 132. (a,c)

133. (a) 134. (a) 135. (c) 136. (b) 137. (a) 138. (b)

139. (a,b,c,d) 140. (b,c) 141. (a,c,d) 142. (a) 143. (a,b,c,d) 144. (a)

145. (a,b) 146. (a,b,d) 147. (a) 148. (b) 149. (a) 150. (a)

151. (a,b,c,d) 152. (a,b,c) 153. (a,b) 154. (a) 155. (a,b,c,d) 156. (a,c)

157. (a) 158. (a,c) 159. (a) 160. (a) 161. (b) 162. (a,b,c)

163. (d) 164. (c) 165. (a) 166. (a) 167. (a) 168. (a)

169. (a) 170. (d) 171. (a,c,d) 172. (a) 173. (b) 174. (c)

175. (c) 176. (a,b) 177. (a) 178. (b,c) 179. (a,b) 180. (b)

181. (b) 182. (c) 183. (c) 184. (a) 185. (a,b) 186. (b,c,d)

187. (a) 188. (a,b,d) 189. (b) 190. (b) 191. (a,b,c) 192. (a,b)

193. (a,b) 194. (b) 195. (c,d) 196. (a,b) 197. (c) 198. (b)

199. (a,b,c) 200. (b,c) 201. (a,b,c) 202. (a,b,c) 203. (b,c) 204. (a,b)

205. (a,c) 206. (b,c) 207. (a,b,c) 208. (a,b) 209. (a,b,c) 210. (a,b,c)

211. (a,b,c) 212. (a,c) 213. (a,b) 214. (b,c) 215. (a,b) 216. (a,b)

217. (a,b) 218. (a,b,c) 219. (a,c) 220. (a,b) 221. (b,c) 222. (a,b,c)

223. (a,b) 224. (a,b,c) 225. (a,c) 226. (a,b,c) 227. (a,c) 228. (b,c)

229. (b) 230. (c) 231. (d) 232. (b) 233. (a) 234. (c)

235. (c) 236. (d) 237. (a) 238. (b) 239. (b) 240. (c)

241. (a) 242. (a) 243. (c) 244. (c) 245. (c) 246. (d)

247. (b) 248. (b) 249. (a) 250. (a)

96
EXPLANATIONS
1. It is possible to place a linear order on C so that we do not get an ordered field, and impossible to place
a linear order of C so that we do get an ordered field.

7n 7
3. (b) Convergent by the root test, → < 1. (d)Convergent by alternative series test.
8n + 1 8

4. x + y = x + y doesn’t have to happen. For example; let y = − x ≠ 0.

5. (a) True: There is a metric space X with open sets. U i ∋ ∩ U i is open. For example if X is any set and d
is the metric d ( x, x) = 0 and d ( x, y ) = 1 for x ≠ y.

(c) True. If Sn → S and tn → t then Sn tn → St .


7. Intervals of convergence for power series are always either a bounded connected interval or infinite on
both sides.
9. (a) False. If f ( x) = − x then f has a maximum at x = 0 but f 1 (0) ≠ 0. ∵ f 1 (0) doesn’tt exist

b b

∫ fdα cannot equal to ∫ f α dx


1
(b) False. It α is not differentiables then
a a

(c) False. Let [a, b] = [0,1] and let f n ( x) = x n . Then each f n is continuous.

⎧0 x≠0
However, nlim f n = f ( x) where f ( x) = ⎨ which is clearly not continuous.
→∞
⎩1 x =1

(d) False: For an explicit counter example let f ( x ) = x + 1, g ( x ) = x and c = 1.

f 1 ( x) f ( x)
Then lim 1
= 1 while lim = 2.
x →c g ( x) x → 0 g ( x)

x 2 + 1 ( x + 1)( x − 1) + 2 2
12. = = 1+ x − = 1 + x − 2(1 + x + x 2 + ...) . Thus a0 = a1 = −1 and ak = −2 for
x −1 ( x − 1) 1− x
all k ≥ 2

1
14. (i) As sin( nt ) ≤ 1, a n ≤
1
. Since ∑n 2 converges. So ∑ an is also converges by the comparison
n2 n n

test. ∴ ∑ an converges absolutely..


n


2

(ii)For n ∈ N , −n2 ≤ −n . 0 < 2 − n2 ≤ 2 − n and so (−1) n 2− n converges by comparison with the


n =1

∞ ∞

∑2 . ∴ ∑ (−1) n 2− n converges absolutely..


2
−n
covergent geometric series
n =1 n =1

97
CE.2 Analysis

x n +1
1 + x n +1 = x 1 + x → x.
n

18. (a) Use the ratio test x n 1 + x n +1 .The series converges for x < 1 and in particular on [0,1).
1+ x n

xn
(b) For 0 ≤ x ≤ a < 1. ≤ a n . ∵ a < 1 the geometric series Σa n converges. By the Weierstrass M--
1 + xn
test, the series converges uniformly on [0, a )

20. (a) By the density of Q, there is a rational number r ∋ a − 2 < r < b − 2.

Thus a < r + 2 < b. ∵ r + 2 is irrational.

(b) x is an upper bound for A. To show that x = sup A . Let ε > 0. Since x −ε < x, ∃ a rational number
r ∋ x−ε <r < x by the density of 1 rational numbers. ∵r ∈ A and x−ε <r. ⇒ x = sup A

(c) Suppose to the contrary that a>b. ∵ a − b > 0 ∃ n ∈ N ∋ a − b > 1 n by the Archimedean property..

This is equivalent to a > b + 1 n which is a contradiction.

23. ∵ f n has a jump discontinuity at x = 0 it does not affect the value of the integral. Thus, for any fixed n
∞ ∞ ∞

we have that ∫
−∞
f n ( x )dx = arctan( nx)
−∞
= π ⇒ lim ∫ fn (x)dx = π
n→∞
−∞

⎧ ⎛ 1 ⎞ ⎧ ⎛ 1⎞
⎪ an +1 = − ⎜ 1 + n + 1 ⎟ n is even ⎪ an = ⎜ 1 + n ⎟ n is even
⎪ ⎝ ⎠ ⎪ ⎝ ⎠
xn = ⎨ yn = ⎨ 1
24. ⎪a = − ⎛1 + 1 ⎞ , ⎪a = ⎛1 + 1 ⎞ .Hence −1 − ≤ xn ≤ − 1
⎜ ⎟ n is odd ⎜ ⎟ n is odd n
⎪⎩ n ⎩⎪
n
⎝ n⎠ ⎝ n +1⎠

and 1 ≤ yn ≤ 1 + yn .So xn → −1 and yn → 1 (i.e.) lim inf an = −1 and lim sup an = 1.

an + 1 x
(−1)n x n nlim = lim
an n →∞ ⎛ n + 2 ⎞
26. Here an =
→∞
, = x by Ratio test the series converges for x < 1 thus
n +1 ⎜ ⎟
⎝ n +1 ⎠

R = 1 the interval of convergence is ( −1,1).


∞ ∞
d ⎛ 1 ⎞ 1
27. We have f ( x) = − ⎜ ⎟ . The power series representation of
dx ⎝ 1 + x ⎠ 1+ x
is ∑ (− x) = ∑ (−1)
n =0
n

n=0
n
xn

d ∞ ∞ ∞

The representation of f ( x) is − ∑
dx n =0
(−1)n xn = −∑ (−1)n nxn −1 = ∑ (−1) n +1 nx n +1
n =1 n =1

98
Analysis CE.3

t
28. The power series of
1 − t8
is ∑ (t )
8 n
= Σt 8 n . Hence, the integral as a power series becomes

t t 8n + 2
∫ 1 − t8 ∫
dt = Σ t 8 n +1
dt = C + ∑ 8n + 2
∴ The radius of convergence is R = 1.

31. (a) Let F = { f :{0,1} → N }. There is a bijection g between F and all pairs ( m, n) ∈ N 2 , given by
g ( f ) = (m, n) where m = f (0) n = f (1). Hence F ∼ N 2 . But N 2 is countable. ∴ F is also countable.
(b) The set of finite subsets of N is countable, since it can be written as a countable union of countable
sets.

32. (c) True. Consider an example an = ⎡⎣1 + (−1) ⎤⎦ n, n ≥ 0 . The odd terms a2 n +1 = 0 for every n ≥ 0 and
n

the even terms a2 n = 2n for every n ≥ 0 . The sequence is unbounded and it contains the constant
subsequence a2 n +1 = 0 which is Cauchy..

1
33. Put an = , x is a fixed real number. If x = 0 then an = 1 for all n ≥ 1, so lim an = 1 . If x < 1 then
1 + xn

x n → 0 as n → ∞ so lim an = 1 . If x > 1 then x n → ±∞ so lim an = 0 . If x = 1, an = 1 2 for all n,

so lim an = 1 2 . If If x = 1, an is not well defined, so no limit.

36. (i) True. If f ( (a + b) / 2 ) ≠ 0 then this number is either > 0 (and f has different signs at the end
points of [a, (a + b) / 2] , or < 0 (and f has different signs at the points of ( [(a + b) / 2, b] .

(ii)True. Let x be in each I n . If f ( x) < 0 then there is some δ > 0 ∋ f ( y ) < 0 for all y in [a, b] with
x − y < δ . Let n be ∋ (b − a ) / 2n < δ . Since length I n = (b − a ) / 2 n all the points y in I n satisfy

x− y ≤ 1 < δ hence f ( y ) < 0 for all y in I n . Contradicting that f has opposite signs on the
2n
endpoints of I n . Similarly we cannot have f ( x ) > 0 thus f ( x) = 0 .

37. (a) True. For ε > 0 let δ =ε and for all x − y < δ we have x − y ≤ x − y < δ = ε .

∴ f ( x) is uniromly continuous (use the triangle inequality to for x − y ≤ x− y )

(b) True. x + y ≤ 6 for all x, y ∈ [0,3] by the triangle inequality. For ε > 0 let δ = ε 6 so that for all
x − y < δ and x, y ∈ [0,3]. we have f ( x) − f ( y ) = x 2 − y 2 = x − y ( x + y ) ≤ 6 x − y < 6δ = ε .

∴ f ( x) is uniformly continuous [0,3]

99
CE.4 Analysis

n2 − 1 x2 − 1
38. an = is positive and converges to 0 as n → ∞ . To show a is decreasing. Let f ( x ) =
n3 + 1 n
x3 + 1
− x4 + 3x2 + 2 x
f '( x ) = <0
(x 3
+ 1)
2 for x ≥ 3 . ∴ an is decreasing for n ≥ 3 and thus the series converges by

1 2
n 1 ∞
1 ∞
n2 − 1
− ∑n ∑n
2
n 1
the alternating series test. ∵ ≥ 2 3 ≥ 4n and diverges by the comparison test, 3
+1
n + 1 2n
3 n =1 n =1

diverges. ∴ The series converges conditionally..

5n 3 + 2 n + cos 2 (e n ) n3 1 1

1
39.
n +n +2
8 5

4 n 8
≥ . and
2 2
∑ n diverges, the series diverges by the comparison test.
n =1

( x + 1) n +1
an + 1 (n + 1)(n + 2)
( x + 1) n = n = x +1 an + 1
= x +1 < 1
a
40. Let n = ; an ( x + 1) n = x +1 2 ; nlim
n(n + 1) n+2 1 + →∞ an
n
n(n + 1)


( x + 1) n ∞
(−1) n
(−2, 0) ; x = −2 : ∑
n =1 n( n + 1)
=∑
n =1 n( n + 1)
Which implies that convergent by alternation series
x =−2


( x + 1) n ∞
1 ∞
1
x = 0:∑ =∑ . Which implies that convergent by comparison with ∑n
n =1 n( n + 1) n =1 n( n + 1)
2
x =0 n =1

∴ The interval of convergence is [−2,0].


1
e n e
42. (a) Use the comparison test ∵ n ≥ 1 we have 1 n ≤ 1 ; e 1n ≤ e ; ≤ .
n2 n2
Being smaller than a convergent p-series, the given series must be convergent itself.
1
e n
(b) by the alternating series test with an = . an is certainly non-negative for each n ≥ 1 and
n
1
e n e0 1
lim an = lim = lim = lim = 0. Moreover an is decreasing for each n ≥ 1
n →∞ n →∞ n n →∞ n n →∞ n

1
⎛ e 1n ⎞ e 1n (− n) −2 .n − e 1n 1
⎜ ⎟ = −e n −1
⎜ n ⎟ n2 = 2 (n +1) is negative for each n ≥ 1 . Thus the given series converges
ges
⎝ ⎠ n
by the alternating series test.

100
Analysis CE.5

1
43. The sequence an +1 = 4 − a for each n ≥1 is monotonic and bounded hence also convergent. Let us
n

denote its limit by L.

1 1 4 ± 16 − 4 4 ± 2 3
an +1 = 4 − L = 4− ; L2 − 4 L + 1 = 0 ; L = =
an ; L 2 2
= 2± 3

Since, an ≥ 2 for all n, L ≥ 2 and this implies L = 2 + 3

44. (a) Since all the terms in the sum are positive and m > n then Sm − Sn = Sm − Sn . The terms is

m
1
S m − S n are those terms up to m excluding the first n. Thus Sm − Sn = ∑
k = n +1 k
2

1 1
(b) k ( k − 1) = k 2 − k < k 2 . ∵ All trms are positive k 2 < k (k − 1)

m m
1 1 1 1 1 1 1 −1
(c) k (k − 1) = k − k − 1 . Then ∑
k = n +1 k ( k − 1)
= ∑ −
k = n +1 k k − 1 all cancel except for the term and
n m
.

m
1 1 1
Thus ∑
k = n +1 k ( k − 1)
= −
n m

m m
1 1 1 1 1 1
(d) Sm − S n = ∑
k = n +1 k 2
<∑
k = n +1 k ( k − 1)
< − < +
m n n m

46. f n ( x) − x n (1 − x ) . Note x n ≥ x n +1 on [0,1] if follows that f n +1 ( x) ≤ f n ( x). For 0 ≤ x < 1. x n → 0, also


f n ( x) → 0 for 0 ≤ x < 1. For x = 1, f n (1) = 0 for all n, lim f n ( x) = 0 for all x ∈ [0,1].

From Dini’s theorem it follows that f n converges uniformly to 0. ∵ the limit function is continuous,

( f n ) is monotone and [0,1] is compact. ∴ f n converges uniformly to 0.


47. (a) True. If the sequence is bounded then all of its subsequence are bounded
(b) True. If all subsequences are unbounded, then the sequence must be bounded. In particular, a sequence
can be regarded as s subsequence of itself.
49. (d) The function is differentiable at 0 with zero derivative. Its slope function is the fourth function which
is continuous at 0.
51. (a) For all x < y, V ( y ) − V ( x) = V ( f ,[ x, y ] because V ( f ,[ x, y ] is the supremum over all possible
partitions P and P = {x, y} is a partition. So V ( f ,[ x, y ] ≥ f ( y ) − f ( x) ≥ 0 V ( y ) − V ( x) ≥ 0

V ( y ) ≥ V ( x) for all y > x.

∴V is increasing on [a, b]

101
CE.6 Analysis

(b) f (c + ) − f (c − ) ≤ f (c + ) − f (c) + f (c) − f (c − ) ≤ V (c + ) − V (c − )



∵V is continuous, V (c + ) − V (c ) = 0

f (c+) − f (c−) ≤ 0

f (c+) = f (c−)
So, f is continuous at c.

(c) Given ξ > 0 by the approximation property of the supremum P = { x0 , x1 ,..., xn } a partition of

n
[a, b] ∋ V ( f ,[a, b]) − ε < ∑ f ( xi ) − f ( xi −1 )
2 i =1

Insert xn∈w into the partition ∋ 0 < xn∈w − x0 < δ f ( xn∈w ) − f ( x0 ) < ε
Because f is continuous.
n

Then V ( f ,[c, b]) − ε 2 < f ( xn∈w ) − f ( x0 ) + f ( x1 ) − f ( xn∈w ) +∑ f ( xi ) − f ( xi −1 )


i =2

n
V ( f ,[c, b]) − ε < ε + f ( x1 ) − f ( xn∈w ) +∑ f ( xi ) − f ( xi −1 )
2 2 i =2

V ( f ,[c, b]) −V ( f ( xn∈w , b)) < ε

V ( f , [ c, xn∈w ]) < ε , for all ξ > 0 we can find a δ ∋ if xn∈w − xc < δ then V ( xn∈w ) − V (c) < ε

So, V is continuous.
54. (b) Let K ∋ f '( x) < K for all x ∈ [a, b]. By mean value theorem , there is a number
c ∋ f ( x) − f (0) = f '(c) ( x − 0) < K .1 = K . So, f is bounded by f (0) − K and f (0) = K .
(d) Products of integrable functions are integrable.

x n +1
1 + x n +1 = x 1 + x → x
n

57. (i) By the ratio test xn 1 + x n +1 . The series converges for x < 1 and in particular on [0,1)
1+ x n

58. (i) By Extreme value function, f attains its minimum m for x ∈ [a, b]. By hypothesis, f ( x) ≥ m > 0 for

all x ∈ [a, b]. ∴0 ≤ 1 ≤1 ⇒ 1 is bounded on [ a, b]


f ( x) m for all x ∈ [ a, b] f

{ xn } = ⎧⎨ { yn } = { 1 n} ,
1 ⎫
(ii)Let ε = 1 define sequences in (0,1] . ⎬ and lim xn − yn = 0 but
⎩ n + 1⎭
2

f ( xn ) − f ( yn ) = n 2 − n + 1 = n(n −1) +1 ≥ 1 = ε for all n ∈  . So by the sequential crieterion for


Nonuniform continuity, the claim follows.

102
Analysis CE.7

59. (i) Fix n ∈ N , f n ( x) → f ( x) = 0 pointwise for all x by the Extreme value theorem f n ( x) = e 1 − 2nx
1 nx 2

( 2

)
1 1 ⎛ 1 ⎞
The critical points of f occur at x = ± The extrema of f on [0,1] are ± at ⎜ x = ± ⎟.
2n 2en ⎝ 2n ⎠

1
Thus M n = xsup
∈[0,1]
f n ( x) − f ( x) = , ∵ nlim Mn = 0 { f n ( x)} is uniformly convergent on [0,1]
2en →∞


cos(2n ( x) cos(2n ( x) 1
(ii)Let f n ( x) = ∑ n for all x ∈ R. for any fixed n ∈ N , f n is continuous on R. 2 n
≤ n
2
n =1 2
∞ ∞
1
for all x ∈ R and ∑2
n =1
n converges ∴ By the Weierstrass M test. ∑f
n =0
n ( x) converges uniformly to

f ( x) on R. Thus f is continuous on R.

b = ( a + b + 1) n + (3a + b + 2) n + 2a
2
a 1
60. An = + + n(n + 1)( n + 2)
n n +1 n + 2

a + b +1 2
If a + b + 1 ≠ 0 then ( a + b + 1) n 2 + (3a + b + 2) n + 2a ≥ n
2

a + b +1 n2 a + b +1 1
For large n so η n ≥ − ≥
2 ( n + 1)( n + 2) 8 n

∵ ( n + 1)( n + 2) < 4n 2 for n ≥ 1.

1
∵∑ diverges, ∑ An diverges.
n

Assume that a + b + 1 = 0. Then if 3a + b + 2 ≠ 0 then (3a + b + 2)n + 2a ≤ 3a + b + 2 n for large


ge n

2 3a + b + 2 1
so that An ≤ and the series converges since ∑
(n + 1)(n + 2) (n + 1)(n + 2) converges.

If 3a + b + 2 = 0 , then 2a = −1, and the series converges.

63. (a) Claim A0 ∩ B0 is empty. If not take x ∈: A0 ∩ B0 . x ∈ A0 and x ∈ B 0 x ∈ B0 or x is a limit point of


B0 . x ∈ B0 . x ∈: A0 ∩ B0 (i.e) q( x) ∈ A ∩ B a contradiction. ∵ A and B are separated.
Claim: x is a limit point of B0 ⇒ q( x) is a limit point of B. Take any neighborhood Nr of q( x) and
q(t ) lies in B for small enought t. More precisely..

r r
x− <t < x+
b−a b−a

103
CE.8 Analysis

⎛ r r ⎞
∵ x is a limit point of B0 and ⎜ x − ,x+ ⎟ is a neighborhood N of x, thus N contains a
⎝ b−a b−a ⎠
point y ≠ x ∋ y ∈ B0 , (i.e) q( y ) ∈ B. Also q( y) ∈ N r .

∴ q ( x) is a limit point of B. Hence, q ( x) ∈ A ∩ B a contradiction. Since A and B are separated.

Hence A0 ∩ B0 is empty. (i.e) A0 and B0 are separated subsets of R1 .

(b) Suppose not. For every to ∈ (0,1) neither q(t0 ) ∈ A nor q (t0 ) ∈ B (∵ A and B are separated). Also
q (t0 ) ∈ A ∪ B for all to ∈ (0,1). Hence (0,1) = A0 ∪ B0 a contradiction. Since (0,1) is connected.

(c) Let S be a convex subset of R k . If S is not conncted, then S is a union of two nonempty separated sets
A and B. There exists to ∈ (0,1) ∋ q(t0 ) ∉ A ∪ B. But S is convex, q (t0 ) must lie in A ∪ B, a
contradiction. Hence S is connected.
68. (a) True. Let x, y ∈ R and assume x < y. cosine is a differentiable function on R. So in particular it is
continuous on [ x, y ] and differentiable on [ x, y]. Thus by the mean value theorem there is some

cos x − cos y cos x − cos y


c ∈ [ x, y ] ∋ cos1 (c) = . But cos1 (c) = sin c and sin c ≤ 1 ; ≤1
x− y x− y

∴ cos x − cos y ≤ x − y .

(b) True. Let f ( x) = e x − ex. Then f (1) = 0 and f 1 ( x ) = e x − e

For x < 1; f 1 ( x) < 0 so f is a decreasing function on (−∞,1) . For x > 1; f 1 ( x) > 0 so f is an increasing
function on (1, ∞). Thus f ( x) ≥ 0 for all x ∈ R. ⇒ e x ≥ ex for all x ∈ R .

(c) True. Let g ( x) = x − sin x for x ≥ 0. Then g (0) = 0 and g 1 ( x) = 1 − cos x ≥ 0 . Thus g is always
nondecreasing on [0, ∞ ). Thus x − sin x ≥ 0 and x ≥ sin x for x ≥ 0 .

1
(d) True f ( x) is continuous on [1, ∞ ) and f is differentiable on (1, ∞). In fact f ( x ) = ≤ 1 for all
1

x
x ∈ (1, ∞ ). ∴ f is uniformly continuous on [1, ∞ ) .

70. (a) For fixed x ≥ 0 we have f ( x) = nlim


→∞
f n ( x) = 0

⎧t − ε t ⎫
(b) If x ∈ [0, t ] for some fixed t > 0, given ε > 0 let N = max ⎨ ,1⎬. Then if n > N and x ∈ [0, t ]
⎩ ε ⎭
x t t
we have f n ( x) − f ( x) = ≤ ≤ = ξ . ∴ ( f n ) converges uniformly to f on [0, t ].
x+n t+n t+n

104
Analysis CE.9

A A A
sin x ⎡ cos x ⎤ cos x
73. (a) Use the integration by parts to get ∫ dx = ⎢ − ⎥ − ∫ 2 dx for any A > 1.
1
x ⎣ x ⎦1` 1 x

A ∞
⎡ cos x ⎤ cos(1) cos x
∵ lim ⎢ −
A→∞


x ⎦1`

1
= cos(1) and the improper integral ∫
1 x
2
dx is absolutely convergent.


sin x
∴∫ dx is convergent
1
x

∞ ∞
1 1 dx 1
(b) The inequality
1+ x 3

x 3 for any x ≥ 1. ∵ ∫
1 x3
=∫
1 x
3
2
dx is convergent (because


1
3 > 1), by the comparison test
2 ∫
1 1 + x3
dx to be convergent

nx 1
75. (a) Since f n (0) = 0 for all n ∈ N . f (0) = 0 and for x > 0 we have lim = lim = 1.
n →∞ 1 + nx n →∞ 1 x
+
n

⇒ f ( x) = 1 for x>0

nx 1 1 1
(b) Let n ≥ 1, if 0 < x < 1 n then −1 = > =
1 + nx 1 + nx 1 + 1 2

⇒ { f n } does not converges uniformly to f on [0, ∞)

nx 1 1 1
(c) If x ≥ a then −1 = ≤ and thus nlim =0
1 + nx 1 + nx 1 + na →∞ 1 + na

⇒ { f n } conerges uniformly to f on [0, ∞)

x
76. lim = 2x + = 2 x (b) 2x is continuous on [ 0,1]
n →∞ n

1 1
(c) nlim g 1 ( x) = lim 2 + = 2 ; ∴ ⎡ lim g n ( x ) ⎤ = g 1 ( x ) = 2.
→∞ n n →∞ n ⎣ n →∞ ⎦

1 1
x 1
→∞ ∫ n →∞ ∫
(d) nlim g n ( x )dx = lim 2 x + dx = lim1 + =1
0 0
n n →∞ 2n

1 1 1

∫ g ( x)dx = 1 ; ∫ lim g n ( x )dx =


0 0
n→ ∞ ∫ g ( x )dx = 1
0

105
CE.10 Analysis

−z2
79. (b) Consider the limit along the curve x = y = 0 . Then lim = −1 . If we consider the limit along the
z →0 z2
0
curve x = z = 0 then lim = 0 . ∵ We get two different limits along two different curves. The limit
y→0 y 2

does not exist.

x sin x
(c) Consider the limit along the curve given by y = x then lim = 1 limit along the curve given
x→0 x2 + 2x2 3

− x sin x
by y = − x then lim = − 1 ∴ We get two different limits along two different curves and so
x→0 x2 + 2x2 3
the limit does not exist.

(n + 1)n n +1
80. (a) Fix x ∈ [0,1) then the sequence ( nx ) consists of non-zero terms and L = lim
n
nx n

⎛ (n +1) ⎞
= lim⎜ ⎟ x = 1. x = x < 1 . Hence lim nx n = 0 for x ∈ [0,1) for x = 0 g n ( x) = g n (0) = n.0n =0
⎝ n ⎠
for all n. So, lim g n ( x) = 0 for all x∈[0,1)

1 1
⎡ x n +1 ⎤
1 1
n = lim 1
n 1
(c) ∫ n ∫ . So, n →∞ ∫ g n ( x)dx = nlim
=
( )
n
g ( x )dx nx dx =⎢ n ⎥ = lim
→∞ n + 1
n →∞ 1
+ 1 = =1
0 0 ⎣ n + 1 ⎦0 n + 1 0 n 0 +1

81. (a) Suppose ∑ x > ∞.


x∈E
{ }
For each n. Let En = x ∈ E : x ≥ 1 n . Then each En is a finite subset of E.

Otherwise if Eno is an infinite set for some no , then letting Fk be a subset of Eno with K no elements for

each K ∈ N ; S FK ≥ K . Then ∑x ≥ S
x∈E
FK ≥ K for each K contradiction. Now E = ∪ E so E is countable.
n

(c) Clearly { x1 , x2 ,.....xn } ∈ F for all n. Thus sup S n ≤ sup


F ∈F
S F . On the other hand given
F ∈F

sup S ≤ sup S n . Hence,
∃n ∋ F ⊂ { x1 , x2 ,..., xn } so S F ≤ S n and F ∈F F ∑ x = sup.S
x∈E F ∈F
F = sup Sn = ∑ xn
n =1

85. (a) Let E be a subset of a complete metric space. Suppose E c is dense and F is a closed set contained in
E. Then F c ⊃ E c . So F c is dense and F is nowhere dense.
(b) Suppose E and E c are both dense in a complete metric space X. Also suppose that E and E c are both
Fσ1 s say E ∪ Fn and Ec ∪ Fn1 where each Fn and Fn1 is closed. By part (a) each Fn and

Fn1 is nowhere dense. Then X ∪ Fn ∪ ∪ Fn1 so X is of the first category, contradicting the Baire

category theorem. Hence at most one of the sets E and E c is an Fσ


(c) The set of rational numbers in [0,1] is an Fσ . Since the set of rational numbers in [0,1] and the set
of irrational numbers in [0,1] are both dense in the complete metric space [0,1] the set of irrational

106
Analysis CE.11

numbers in [0,1] is not an Fσ by part (b). Hence its complement, the set of rational numbers in [0,1]
is not a Gδ .

xm xn xm ... xn rm − rn rn
87. (a) r ... 1−
m rn rm rm rm

xn xm xn
If m n. If ∑ r converges, for ε 0∃N∋ r ...
rn
ε for all m, n wherever n m ≥ N.
n m

xm xn rn rn
Fix m N . Thus r ... 1− 1 −1 −
m rn rm rN for all n N.

xm xn
But rn → 0 as n → ∞, thus ... → 1 as n → ∞ .
rm rn

If we take ε 1
2 we will get a contradiction.
(b) rn 1 rn

rn 1 rn

rn rn 1 2 rn

rn rn 1
2
rn

⎛ rn 1 ⎞⎟⎟
⎜⎜ rn
rn − rn 1 ⎜⎜ ⎟⎟ rn − rn 1
⎜⎝ rn ⎠⎟

rn rn 1
2 rn − rn 1
rn

xn
2 rn − rn 1
rn

Since for all xn 0 for all n


k
xn k

Hence ∑n 1 rn
∑2n 1
rn − rn 1 2 r1 − rk 1

xn xn
rn → 0 as n → ∞ . Thus ∑ rn is bounded. Hence
∑ rn converges.

1⎛ α ⎞⎟ 1 ⎛⎜ α⎞ 1 ⎛⎜b2n −α⎞⎟
88. (a) bn − bn bn − ⎜⎜⎜bn ⎟⎟ ⎜⎜bn − ⎟⎟⎟ ⎜ ⎟⎟
2 ⎜⎝ bn ⎠⎟ 2 ⎜⎝ bn ⎠⎟ 2 ⎜⎜⎝ bn ⎠⎟ 0
1

107
CE.12 Analysis

Since, bn α : Hence bn decreaes monotonically. Also bn is bounded by 0, thus bn converges.


ges.

1⎛ α ⎞⎟ 1 ⎛⎜ α ⎞⎟
Let, lim bn b .Hence, lim bn lim ⎜⎜⎜bn ⎟⎟ ⎜b ⎟; 2 α , bn 0 for all n. Thus
1
2 ⎜⎝ bn ⎠⎟ 2 ⎜⎝ b ⎠⎟ b

b α .lim bn α.
2
1 ⎛⎜ α ⎞⎟ 1 ⎛⎜ α ⎞⎟ bn − α εn2 εn2
(b) bn ⎜bn ⎟⎟ bn 1 − α ⎜bn ⎟⎟ − α εn
2 ⎜⎜⎝ bn ⎠⎟ ; 2 ⎜⎜⎝ bn ⎠⎟ ;
1 1
2bn 2bn 2 α

2n
⎛ε ⎞
Hence εn β ⎜⎜ n ⎟⎟⎟ where β 2 α by induction.
⎝⎜ β ⎠⎟
1

ε1 2− 3 1 1 1
(c) 2 3 2 3 6 4 3 10
β 2 3

⎛ε ⎞ ⎛ε ⎞
Thus ε5 β ⎜⎜ 1 ⎟⎟⎟ 2 4 2 3.10−16 4.10−16 ; ε6 β ⎜⎜ 1 ⎟⎟⎟ 25 2 3.10−32 4.10−32
⎝⎜ β ⎠⎟ ⎝⎜ β ⎠⎟

t0 t1 ... tn
89. (a) Let tn Sn − S , τ n σn − S Then τ n
n 1
Choose M 0 ∋ tn ≤ M for all n. Gives ε 0, choose N so that n N implies tn ε. Taking

t0 t1 ... tn t0 ... t N tN 1 ... tn ( N 1) M


n N in τ n and then τn ≤ ξ
n 1 n 1 n 1 n 1

Hence, lim sup τn ε. since ξ is arbitrary, it follows that nlim τn 0 that is lim σn S
→∞
n→∞

n n n n n−1 n

(b) ∑ kx
k 1
k ∑k
k 1
Sk − Sk −1 ∑ ks − ∑ S
k 1
k
k 1
k −1 nSn ∑ Sk − S0
k 1
(n 1) Sn − ∑ Sk = ( n + 1 )
k 0

1 n ⎛ 1 n ⎞
(i.e) Sn − σn ∑ kx . Note that nxn is a complex sequence. nlim ⎜⎜
∑ kxk ⎟⎟⎟ lim nxn 0
n 1 k 1
k ⎜
→∞ ⎝ n 1k 1 ⎠⎟ n →∞

Also lim σn σ . Hence lim S σ


a −c
93. (a) Assume g is continuous iff for sequence xn → 0 with xn ≠ 0, x n sin x n → 0 as n → 0.
1
⎛ ⎞⎟ c
⎜⎜ 1 ⎟⎟
xn ⎜⎜ ⎟⎟ 0 and thus x a → 0 as n → ∞. Hence a
⎜⎜ 2nπ π ⎟ 0. (If not, then a 0 or a 0
⎠⎟
n
⎝ 2
a 1 → ∞ as n → ∞. It contradicts).Conversely assume g
0, x n
a
when a 0, x n 1 when a x−n a
a a −c a a
is continuous on [−1,1] − 0 . Clearly note that − x ≤ x sin x ≤ x and x → 0 as x → 0 since

108
Analysis CE.13

a 0. Thus g is continuous at x = 0, Hence g is continuous


a −1 −c
(b) g 1 (0) exists iff x sin x → 0 as x → 0 g 1 (0) exists ifff a −1 1
0. Also g (0) 0.

n
1− x n 1
96. If x ≠ 1 ; S n ∑x
k 0
k

1− x
.If we let n → ∞. For x 1 we get 1 1 1 ... Which evidently
diverges

99. (a) If not, then ∃ two distinct fixed points, say x and y of f. Thus f ( x) x and f ( y ) y.
Since f is differentiable, by applying mean value theorem f ( x) − f ( y ) f 1 (t )( x − y ) where t is
between x and y. Since x ≠ y , f 1 (t ) 1. It contradicts

et
(b) To show that 0 f 1 (t )
1
1 for all real t ; f (t ) 1 (−1)(1 et )−2 et 1−
(1 et )2

Since et 0; (1 et ) 2 (1 et )(1 et ) (1 et ) (1 et ) et 0
For all real t, thus (1 et ) 2 et 0; (1 et )−2 et 1 . For all real t. Hence 0 f 1 (t ) 1 for all real
t. Since f (t ) − t (1− et )−1 >0 for all real t , f (t ) has no fixed point.

101. (a) Bounded since A is a subset of the closed unit interval I = [0,1]

108. (b) True. That is because of all the points in N are isolated points of N.
(d) True. Let h ( x ) = f ( x ) − g ( x ) , then h is continuous on [0,10] & differentiable on (0,10) &

h′ ( x ) = 0 for all x ∈ ( 0,10 ) . h ( x ) = constant. Since h (π ) = 0 = constant. We have h ( x ) = 0

& ∴ f ( x) = g ( x)

1
109. (a) f ( x ) = x sin is well defined for all x ≠ 0 . For any ε > 0 . There exists δ = ε such that whenever
x

⎛1⎞ ⎛1⎞
0 < x < δ we have x sin ⎜ ⎟ − 0 ≤ x < ε . ∴ Lim x sin ⎜ ⎟ = 0
⎝ x⎠ x →0 ⎝x⎠

( x − 1)( x − 3) 1
Lim =−
(
110. (a) x →1 ( x − 1)( x + 1) x 2 + 3 + 2 x ) 4

112. (a), (c) Lim


x →1− x →1 + +
x →1 x →1
(
x →1
)
f ( x ) = Lim− ( 2 x + 5 ) = 7 ; Lim f ( x ) = Lim 9 x 2 − 2 = 7 .We have Lim f ( x ) = f (1) = 7 .

f ( x ) − f (1) 2x − 2
∴ f ( x ) is continuous at 1. f −′ (1) = Lim− = Lim− =2
x →1 x −1 x →1 x −1

f ( x ) − f (1) 9 x2 − 9
f +′ (1) = Lim+ = Lim+ = 18 . We have f −′ (1) ≠ f +′ (1) ⇒ f is not differentiable at 1
x →1 x −1 x →1 x −1

109
CE.14 Analysis

114. (a) True. Since any deleted neighbourhood of x is a union of two open intervals
(d) True. For x ∈ int S , which means N ( x, ε 0 ) ⊆ S for some ε 0 > 0 , we have N ( x, ε 0 ) ∩ ( R \ S ) = φ
and x ∉ bdS . ∴ int S ∩ bdS = φ .

115. (b) By choosing n = 2k , we have S2 k = 1 → 1 as k → ∞ and by choosing S2 k −1 = −1 → −1 as k → ∞ .


Therefore Sn = cos ( nπ ) is divergent.

( n 2 + 2n − n 2 − 3n )( n 2 + 2n + n 2 − 3n )
116. (a) Lim
n →∞
( n 2 + 2n − n 2 − 3n ) = Lim
n →∞
( n 2 + 2n + n 2 − 3n )
5n 5
= Lim =
n →∞
( 2
n + 2n + n − 3n 2
) 2

( −1)n
118. (d) 1) incorrect: Example let Sn = for n ∈ N , which is a cauchy sequence (since it is not
n
converges to 0) but it is not monotone
2) Incorrect: Example Let Sn = −1 for all n ∈ N , then every ε > 0 there exists N = 0 ∈ R such that

for every n > N we have S n < ε but nLim Sn = −1 ≠ 0


→∞

( n + 1)2
Sn +1 1.01n +1 ( n + 1)
2
1
= = → <1 n2
119. (a) Sn n 2
1.01n 2
1.01 as n→∞ ∴
. We have Lim =0
n →∞ 1.01
1.01n

⎧ n ⎫ ⎧1 2 3 ⎫
120. (a) Let S = ⎨ : n ∈ N ⎬ = ⎨ , , ,....⎬ . The set S is bounded below, 1 being a lower bound for S.
⎩ n + 1 ⎭ ⎩ 2 3 4 ⎭ 2

1 1 n
∈ S . glb of S (ie) inf S = . Since, < 1 ∀ n ∈ N , therefore 1 is an upperbound for S.
2 2 n +1
Any real number less than 1 cannot be as upper bound for S. ∴ 1 is as upper bound for S. sup S=1
123. (a) Let x ∈ Z . For any ε > 0 , x − ε & x + ε are two distinct real numbers and we know that
between any two distinct real numbers there lie infinite irrational numbers which are definitely not
members of Z.. ( x − ε, x + ε ) ⊄ Z, ∀ ε > 0 . ∴ Z is not a neighbourhood of x

(b) Let P ∈ Q ; ( P − ε , P + ε ) ⊄ Q, ∀ε > 0. ∴ Q is not a neighbourhood of x.


(c) Let R denote the set of all real numbers and Q denote the set of all rational numbers. Then
R-Q is the set of all irrational numbers. Let x ∈ ( R − Q ) . For any ε > 0 , x − ε and x + ε are two
distinct real number and between any two distinct real numbers there lie infinite rational number

110
Analysis CE.15

which are definitely not members of R-Q. Thus there exists no ε > 0 such that (x −ε, x +ε ) ⊂ R −Q
∴ R − Q is not a neighbourhood of x.
125. (a) Since (2,4) is an open interval 3 ∈ (2,4), (2,4) is neighbourhood of 3
(b) Since there exists an open interval (2,4) such that 3 ∈ (2,4) ⊂ [2,4]; ∴ [2,4] is a neighbourhood
of 3

⎧ 1⎫ ⎛ 1 1⎞
(d) [2,4]- ⎨3 ⎬ is a neighbourhood of 3. Since there exists an open interval ⎜ 3 − ,3 + ⎟ such
⎩ 4⎭ ⎝ 5 5⎠

⎛ 1 1⎞ ⎛ ⎧ 1 ⎫⎞
[ ]
that 3 ∈ ⎜ 3 − ,3 + ⎟ ⊂ ⎜ 2, 4 − ⎨3 ⎬ ⎟
⎝ 5 5⎠ ⎝ ⎩ 4 ⎭⎠
126. (a) We shall show that every real number P is not a limit point of the set S which is a finite set. since the
set S is finite, therefore if we take any ε > 0 , the open interval ( P − ε , P + ε ) contains at most a

finite number of points of the set S. Thus ( P − ε , P + ε ) is a neighbourhod of P which does


not contain infinitely many points of S and so P is not a limit point of S. Thus every real number P
is not a limit point of S and so the derived set of S is empty (ie) D(S)= φ .

n
130. (a),(b) A is denumerable because the mapping f : N → Α defined by f ( n ) = ∀ n ∈ N is
n +1
bijective (ie) one-one & onto

1 1
135. (c) S n = log . Take any given K < 0 . Then S n < K if log < K (ie) if − log n < K
n n
(ie) if log n > − K (ie) if n > e − K . If we take m ∈ Ν such that m > e − K then S n < K for all

n ≥ m . Hence S n → −∞ as n → ∞ .

143. (a) Let [a,b] = G ∪ H, Such that G ∪ H = φ . Let b ∈ H . Then claim G = φ . If not, Let C = Sup G.

Since G is closed C ∈ G . Since G is open Bξ (C ) ⊆ G ie [C , C + ξ ) ⊂ G . That contra


dicts C = sup G. Thus G = φ

(b) Let R = ∪ n∈N [ −n, n ] , Ο ∈ ∩ [ −n, n ] . Therefore, R is the union of connected subsets.
∴ R is connected.
(c) Say f ( M ) = G ∪ H , G, H ≠ φ, G , H open. Then M = f −1 ( G ) ∪ f −1 ( H ) where f −1 ( G )

and f −1 ( H ) are both open and nonempty, contradicts. Connectednees of M


(d) Assme I is a interval S = [a,b]; S = (a,b]; a ≥ −∞

111
CE.16 Analysis

⎡ 1⎤
S = [a,b); b ≤ ∞ ; S=(a,b), a ≥ −∞ , b ≤ ∞ , [ a, b ) = ∪ n≥ n 0 ⎢ a, b − n ⎥ , a ∈ ∩ [ a, b − 1/ n ] .
⎣ ⎦
Therefore I ⊆ R is connected.
148. (b) Consider f
−1
({0}) . Since {0} is closed and f continous, f ({0}) is closed. Therefore
−1

S = [0,1] ∩ f ({0}) is a closed and bounded subset of R . Hence S is compact.


−1

sin (1/ x )
179. (a), (b)Let f ( x) = . There is no neighbourhood of the point 0, in which f(x) constantly
x
1
sin1/ x sin1/ x 1 1
keeps the same sign. Now ∀ x ∈ ( 0,1] we have
x
=
x

x
also ∫
0 x
dx is

1 1
sin1/ x sin1/ x
convergent . ∫
0 x
dx is convergent ⇒ ∫
0 x
dx is absolutely convergent.

192.(a) Suppose f is Lipschitz. ∃ M ∋ f ( x) − f ( y ) ≤ M x − y for all x, y. Given ε > 0 , let δ ε


M
n

. For any finite collection xi , xi′ of nonoverlapping intervals with ∑ x′


1
i − xi δ we have

n n


1
f xi′ − f xi ≤ M ∑ x′
1
i − xi ε . Thus f is absolutely continuous.

f ( y ) − f ( x)
(b) Let f be absolutely continuous. Suppose f is Lipschitz. Now f ′( x) lim
y→ x y−x

f ( y ) − f ( x)
So, f ′( x) lim ≤ M , for all x.Conversely if f is not Lipschitz, then for any M, ∃ x
y→ x y−x

and y ∋ f ( x) − f ( y ) M x − y . Then f ′ (c ) M for some c ∈ ( x, y ) by the Mean value

theorem. Thus for any M , ∃ c ∋ f ′(c ) M so f ′ is unbounded.

193. (a),(b)Suppose ∃ points a, b of A for which f (a) 0 and f (b) 0. Then by the Intermediate value
theorem ∃ a point c ∈ (a, b) ∋ f (c ) 0. Contrary to the hypothesis on f. Thus either
f ( x) 0 for all x ∈ A or f ( x) 0 for all x ∈ A .

an 1 2n 1 z n 1n ! 2z 1
194. (b) 2 z .
an 2n z n (n 1)! n 1 n 1
By taking the limit we get 0, which is always less than 1. Thus, any z. The radius of convergence is
infinite.

112

You might also like